You are on page 1of 83

TOPNOTCH MEDICAL BOARD PREP SURGERY SUPPLEMENT HANDOUT - Jules Lopez,MD-MBA,Teddy Carpio,MD-MBA

For inquiries visit www.topnotchboardprep.com.ph or email us at topnotchmedicalboardprep@gmail.com

Table 1. Cytokines And Their Responses to Injury 


SURGERY REVIEW Cytokines and their Responses to Injury
 Among earliest responders after
injury
This review material is a synthesis of the first few chapters of Tumor Necrosis Factor
 Induces muscle breakdown and
Schwartz’s Principles of Surgery, 9th edition, Absite and Board Alpha (TNF-)
cachexia through increased
Review of Schwartz’s Principles of Surgery, 9th edition, 2009 Absite catabolism
Clinical Review of Surgery and RUSH integrated review of surgery.  Induces fever through
Integration with other basic subjects as well as review questions prostaglandin activity in anterior
per section are also included to facilitate mastery of the course. Interleukin 1 (IL-1) hypothalamus
 Promotes -endorphin release from
Ready? God bless and enjoy Surgery!  pituitary
 Promotes lymphocyte
- Dr. Jules Lopez and Dr. Teddy Carpio proliferation, immunoglobulin
production, gut barrier integrity
Interleukin 2 (IL-2)
P.S.  Attenuated production after major
blood loss leads to
Ophthalmology, ENT, Orthopedics, and Gynecology topics are not immunocompromise
included in this surgery handout.  Elicited by all immunogenic cells as
mediator of acute phase
Interleukin 6 (IL-6) response
Any statements, tables, figures marked with a  means that the
 Prolongs activated neutrophil
information highlighted was previously tested in previous board exams. survival
You should master/memorize those. Pay close attention to those in bold,
 Chemoattractant for neutrophils,
italicized, underlines as these are very important facts to remember for Interleukin 8 (IL-8)
basophils, eosinophils, lymphocytes
the subject. Master topics written in our quick review, subject cross overs
and end of review question boxes because they provide high yield  Activates macrophages via TH1
information, not just for surgery but for the rest of the other subjects as cells that demonstrate enhanced
well! Interferon (IFN-) phagocytosis and microbial killing
 Found in wounds 5-7 days after
injury
General Outline:
I. Basic Principles in Surgery
2. Heat Shock Proteins
a. Systemic Response to Injury and Metabolic support
 Group of intracellular proteins that are increasingly
b. Fluid and Electrolyte management of the Surgical
expressed during times of stress
Patient
 Bind both autologous and foreign proteins and thereby
c. Hemostasis, Surgical bleeding and transfusion
function as intracellular chaperones for ligands such as
d. Surgical Infections and Shock
bacterial DNA and endotoxin
e. Trauma
f. Burns  Protect cells from the deleterious effects of traumatic
g. Wound healing stress and, when released by damaged cells, alert the
II. Organ System Pathologies immune system of the tissue damage
a. Skin and soft tissues
b. Breast 3. Reactive Oxygen Species (ROS)
c. Head & Neck  Small molecules that are highly reactive due to the
d. Esophagus presence of unpaired outer orbit electrons
e. Stomach  Cause cellular injury to both host cells and invading
f. Small Intestine pathogens through the oxidation of unsaturated fatty
g. Appendix acids within cell membranes
h. Colon, Rectum & Anus
i. Abdominal Wall & Hernia 4. Eicosanoids (Refer to Figure 1 )
j. Liver, Portal Venous System & Gallbladder  Derived primarily by oxidation of membrane
phospholipid,arachidonic acid
 Composed of subgroups including prostaglandins,
PART I: BASIC PRINCIPLES IN SURGERY prostacyclins, hydroxyeicosatetraenoic acid,
thromboxanes, and leukotrienes
 Generate a proinflammatory response
SYSTEMIC RESPONSE TO INJURY AND
METABOLIC SUPPORT Figure 1. Arachidonic acid metabolism . Cycloxygenase catalyzes the
formation of PG and TXA2from arachidonic acid. LT = leukotriene;
A. Mediators of Inflammation PG = prostaglandin; TXA2 = thromboxane A2
B. Central Nervous System Regulation of Inflammation
C. Hormonal Response to Injury
D. Surgical Metabolism
E. Nutrition in the Surgical Patient

A. MEDIATORS OF INFLAMMATION
1. Cytokines(Refer to Table 1 )
 Protein signaling compounds that are essential for
both innate and adaptive immunity
 Mediate cellular responses, including cell migration and
turnover, DNA replication, and immunocyte
proliferation

TOPNOTCH MEDICAL BOARD PREP SURGERY SUPPLEMENT HANDOUT Page 1 of 83


For inquiries visit www.topnotchboardprep.com.ph or email us at topnotchmedicalboardprep@gmail.com
TOPNOTCH MEDICAL BOARD PREP SURGERY SUPPLEMENT HANDOUT - Jules Lopez,MD-MBA,Teddy Carpio,MD-MBA
For inquiries visit www.topnotchboardprep.com.ph or email us at topnotchmedicalboardprep@gmail.com
burn-injured patients may exhibit elevated levels for
 PHARMACOLOGY a 4 weeks)
 Wound healing is impaired because it reduces
 Aspirin (Acetylsalicylic acid, ASA) is one of the non- transforming growth factor-beta (TGF-B) and insulin-
steroidal anti-inflammatory drug (NSAID) prototypes like growth factor I (IGF-I) in the wound
 Mechanism of action ||
Figure 2. Steroid synthesis from cholesterol . ACTH is a principal
o Nonselective AND irreversible inhibitor of
regulator of steroid synthesis. The end products are mineralocorticoids,
the enzyme cyclooxygenase (COX), inhibiting glucocorticoids, and sex steroids.
both cylooxygenase-1 (COX-1) and
cyclooxygenase-2 (COX-2) isoenzymes
o Anti-inflammatory effect is mediated by COX-2
inhibition via decreasing platelet production of
TXA2, a potent stimulator of platelet
aggregation
 Side effect ||Gastrointestinal (GI) toxicity due to
inhibition of COX-1 and therebyPG synthesis
 Notes ||Uncoupler of oxidative phosphorylation and
is associated with Reye syndrome in children
 Ketorolac and Indomethacin are bothnonselective
BUT reversible COX inhibitors
 Celecoxib is a selective COX-2 inhibitor with a reduced
risk of GI toxicity

5. Fatty Acid Metabolites  BIOCHEMISTRY a


 Omega-3 fatty acids have anti-inflammatory effects,
including inhibition of TNF release from hepatic  Cholesterol is a steroid alcohol
Kupffer cells, leukocyte adhesion and migration  It is a precursor of the following:
6. Kallikrein-Kinin System 1. Cell membranes
 Group of proteins that contribute to inflammation, 2. Vitamin D (7-dehydrocholesterol)
BP control, coagulation, and pain responses 3. Bile salts (cholic and chenodeoxycholic acid)
 Kallikrein levels are increased during gram negative 4. Adrenal hormones (aldosterone and cortisol)
bacteremia, hypotension, hemorrhage, endotoxemia, 5. Sex hormones (testosterone and estradiol)
and tissue injury  It is very hydrophobic (which means, it can cross lipid
 Kinis mediate vasodilation, increased capillary predominant barriers), composed of 4 fused
permeability, tissue edema, pain pathway activation, hydrocarbon rings (A-D) and 8-membered branched
inhibition of gluconeogenesis, and increased hydrocarbon chain (20-27) attached to the D-ring
bronchoconstriction  It has a single hydroxyl group located at carbon 3 of the
 Elevated levels of both has been associated with the A-ring to which a fatty acid can be attached to form
magnitude of injury and mortality cholesterol esters

7. Serotonin
 Released at the site of injury, primarily by platelets
 Stimulates vasoconstriction, bronchoconstriction, and
platelet aggregation
 Ex vivo study showed that serotonin receptor blockade
is associated with decreased production of TNF andIL-1
in endotoxin-treated monocytes

8. Histamine (H4)
 Associated with eosinophil and mast cell chemotaxis
 Increased release has been documented in hemorrhagic
shock, trauma, thermal injury, endotoxemia, and sepsis

B. CNS REGULATION OF INFLAMMATION 2. Macrophage Migration-Inhibiting Factor


 Vagus nerve is highly influential in mediating afferent  Neurohormone that is stored and secreted by the
sensory input to the CNS anterior pituitary and by intracellular pools within
o Parasympathetic nervous system transmits macrophages
its efferent signals via acetylcholine  A counter regulatory mediator that potentially reverses
o Exerts homeostatic influences such as the anti-inflammatory effects of cortisol
enhancing gut motility, reducing heart rate,
and regulating inflammation 3. Growth Hormones (GH)
o Allows for a rapid response to inflammatory  Neurohormone expressed primarily by the pituitary
stimuli and also for the potential regulation of gland that has both metabolic and immunomodulatory
early proinflammatory mediator release, effects
specifically tumor necrosis factor (TNF)  Exerts its downstream effects through direct interaction
o Inhibit cytokine activity and reduce injury with GH receptors and secondarily through the
from disease process enhanced hepatic synthesis of IGF-I
 GH and IGF-I promote protein synthesis and insulin
C. HORMONAL RESPONSE TO INJURY resistance, and enhances mobilization of fat stores
1. Cortisol(Refer to Figure2  )
 Glucocorticoid steroid hormone released by the adrenal 4. Catecholamines
cortex in response to adrenocorticotropic hormone  Include epinephrine, norepinephrine, and dopamine,
(ACTH) which have metabolic, immunomodulatory, and
 Release is increased during times of stress and may be vasoactive effects
chronically elevated in certain disease processes (e.g.

TOPNOTCH MEDICAL BOARD PREP SURGERY SUPPLEMENT HANDOUT Page 2 of 83


For inquiries visit www.topnotchboardprep.com.ph or email us at topnotchmedicalboardprep@gmail.com
TOPNOTCH MEDICAL BOARD PREP SURGERY SUPPLEMENT HANDOUT - Jules Lopez,MD-MBA,Teddy Carpio,MD-MBA
For inquiries visit www.topnotchboardprep.com.ph or email us at topnotchmedicalboardprep@gmail.com
 After severe injury, plasma catecholamine levels are
increased threefold to fourfold, with elevations lasting
24 to 48 hours before returning to baseline levels

5. Insulin
 Mediates an overall host anabolic state  Lactate production is insufficient to maintain systemic
 Insulin resistance and hyperglycemia are hallmarks glucose needs during short-term fasting; therefore,
of critical illness due to the catabolic effects of significant amounts of protein must be degraded
circulating mediators, including catecholamines, cortisol, daily (75 g/d for a 70 kg adult) to provide the amino
glucagon, and GH acid substrate for hepatic gluconeogenesis
 Hyperglycemia during critical illness has  Proteolysis during starvation, which results from
immunosuppressive effects, and thus is associated decreased insulin and increased cortisol release, is
with an increased risk for infection associated with elevated urinary nitrogen excretion
 Insulin therapy (to manage hyperglycemia) decreased from the normal 7-10 g/day up to 30 g or more/day
mortality and reduced in infectious complications in
select patient populations 2. Metabolism During Prolonged Fasting
 Systemic proteolysis is reduced approximately 20 g/d
 QUICK REVIEW a and urinary nitrogen excretion stabilizes at 2 to 5 g/d
due to adaptation by vital organs (e.g. myocardium,
 Burn patients may exhibit elevated levels of cortisol for brain, renal cortex, and skeletal muscle) to using
4 weeks ketone bodies as their principal fuel source
 Plasma catecholamine levels are increased 3-4x lasting  Ketone bodies become an important fuel source for the
for 24 to 48 hours before returning to baseline brain after 2 days and gradually become the principal
fuel source by 24 days

D. SURGICAL METABOLISM 3. Metabolism After Injury


 To maintain basal metabolic needs (i.e. at rest and  Injuries or infections induce unique neuroendocrine
fasting), a normal healthy adult requires ~22 to 25 and immunologic responses that differentiate injury
kcal/kg/dayfrom carbohydrate, lipid, and protein metabolism from that of unstressed fasting
sources  Magnitude of metabolic expenditure appears to be
 Initial hours after surgical or traumatic injury are directly proportional to the severity of insult, with
metabolically associated with a reduced total body thermal injuries and severe infections having the
energy expenditure and urinary nitrogen wasting highest energy demands (Refer to Figure 4)

Figure 4. Influence of injury severity on resting metabolism


1. Metabolism During Short-term Fasting (<5 days)
(resting energy expenditure or REE)
 In the healthy adult, principal sources of fuel are
derived from muscle protein and lipids, with lipids
being the most abundant source of energy(40% or
more of caloric expenditure)
 Hepatic glycogen stores are rapidly and preferentially
depleted  fall of serum glucose concentration within
hours (<16 hours)
 Hepatic gluconeogenesis is then activated using lactate
from skeletal muscle as the main precursor(Refer to
Figure 3 )

Figure 3. The recycling of peripheral lactate and pyruvate for hepatic


gluconeogenesis is accomplished by the Cori cycle. Alanine within skeletal
muscles can also be used as a precursor for hepatic gluconeogenesis .

 Lipids(Triglyceride) become the primary source of


energy (50-80%) during critical illness and stressed
states
o Minimize protein catabolism in the injured
patient
o Lipolysis occurs mainly in response to
catecholamine stimulus of the hormone-
 BIOCHEMISTRY a sensitive triglyceride lipase
 Injury and severe infections induce a state of
 Lactate is generated from the skeletal muscle during peripheral glucose intolerance (insulin resistance),
anaerobic metabolism despite ample insulin production at levels several-fold
 Muscle CANNOT reconvert lactate to glucose above baseline
 Lactate MUST first be transported to the liver, where it is o Occur in part due to reduced skeletal muscle
converted to glucose via hepatic gluconeogenesis pyruvate dehydrogenase activity after injury,
 Glucose is then brought back to musclesfor usage which diminishes the conversion of pyruvate
 This metabolic pathway involving conversion of lactate to acetyl-CoA and subsequent entry into the
to glucose; which, in turn, is brought back to the muscle TCA cycle
for utilization is the Cori cycle o Increase in plasma glucose levels is
 Energy expense: 4 ATP molecules proportional to the severity of injury, and this
net hepatic gluconeogenic response is under
the influence of glucagon

TOPNOTCH MEDICAL BOARD PREP SURGERY SUPPLEMENT HANDOUT Page 3 of 83


For inquiries visit www.topnotchboardprep.com.ph or email us at topnotchmedicalboardprep@gmail.com
TOPNOTCH MEDICAL BOARD PREP SURGERY SUPPLEMENT HANDOUT - Jules Lopez,MD-MBA,Teddy Carpio,MD-MBA
For inquiries visit www.topnotchboardprep.com.ph or email us at topnotchmedicalboardprep@gmail.com
 After injury, the initial systemic proteolysis, mediated especially in special patients (e.g. critically ill
primarily by cortisol, increases urinary nitrogen with significant fluid overload and the obese)
excretion to levels in excess of 30 g/d, which roughly o Overfeeding may contribute to clinical
corresponds to a loss in lean body mass of 1.5%/d deterioration via the following: increased O2
(Refer to Figure 5) consumption, increased CO2 production and
prolonged need for ventilatory support,
Figure 5. Effect of injury severity on nitrogen wasting suppression of leukocyte function,
hyperglycemia, and increased risk of infection

1. Enteral Nutrition
 Generally preferred over parenteral nutrition due to:
o Lower cost
o Associated risks of the intravenous route
o Beneficial effects of luminal nutrient contact as
it reduces intestinal mucosal atrophy
 Initiation should occur immediately after adequate
resuscitation (adequate urine output)
 Presence of bowel sounds and the passage of flatus or
stool are NOT absolute prerequisites to start enteral
nutrition, EXCEPT in the setting of gastroparesis,
feedings should be administered distal to the pylorus
 Gastric residuals of 200 ml or more in a 4 to 6 hour
period or abdominal distention requires cessation of
feeding and adjustment of infusion rate
 QUICK REVIEW a  The following are options for enteral feeding access
(Refer to Table 3 ):
 Normal energy requirement: 22 to 25 kcal/kg/day
 Initial hours after surgical or traumatic injury results to a Table 3. Options for Enteral Feeding Access 
reduced total body energy expenditure and urinary Options for Enteral Feeding Access
nitrogen wasting  Short-term use
 Fat/lipid is the primary source of calories during acute Nasogastric tube  Aspiration risks
starvation (<5 days fasting) and after acute injury (NGT)  Nasopharyngeal trauma
 Frequent dislodgement
 Ketone bodies is the primary fuel source in prolonged
 Short-term use
starvation Nasoduodenal /  Lower aspiration risks in jejunum
 Ketone bodies becomes an important fuel source for Nasojejunal tube  Placement challenges (radiographic
brain after 2 days and eventually become the principal assistance often necessary)
fuel source by 24 days  Endoscopy skills required
 May be used for gastric decompression or
Percutaneous
bolus feeds
Endoscopic
E. NUTRITION IN THE SURGICAL PATIENT  Aspiration risks
Gastrostomy
 Goals of nutritional support in the surgical patient are  Can last 12-24 months
(PEG)
as follows:  Slightly higher complication rates with
placement and site leaks
o To meet the energy requirements for
 Requires general anesthesia and small
metabolic processes, core temperature
laporotomy
maintenance, and tissue repair Surgical
 Procedure may allow placement of
o To meet the substrate requirements for Gastrostomy
extended duodenal/jejunal feeding ports
protein synthesis  Laparoscopic placement possible
 Energy requirement may be measured by indirect  Commonly carried out during laparotomy
calorimetry and trends in serum markers (e.g.  General anesthesia, laparoscopic
prealbumin level) and estimation from urinary nitrogen Surgical placement usually requires assistant to
excretion, which is proportional to resting energy Jejunostomy thread catheter
expenditure  Laparoscopy offers direct visualization of
catheter placement
 Basal energy expenditure (BEE) may also be
 Jejunal placement with regular endoscope
estimated using Harris-Benedict equations, adjusted for
is operator dependent
the type of surgical stress (Refer to Table 2)  Jejunal tube often dislodges retrograde
o BEE (men) = 66.47 + 13.75 (weight in kg) + 5 PEG-jejunal tube  Two-stage procedure with PEG
(height in cm) – 6.76 (age in years) kcal/d placement, followed by fluoroscopic
o BEE (women) = 655.1 + 9.56 (weight in kg) + conversion with jejunal feeding tube
1.85 (height in cm) – 4.68 (age in years) kcal/d through PEG
 The BEE is then multiplied by the type of surgical stress
(Refer to Table 2 ) that the patient has to determine 2. Parenteral Nutrition
the total daily caloric need  Continuous infusion of hyperosmolar solution
containing carbohydrates, proteins, fat, and other
Table 2. Caloric Adjustment Above BEE in Hypermetabolic Conditions  necessary nutrients through an indwelling catheter
Caloric Adjustments Above Basal Energy Expenditures in inserted into the superior vena cava
Hypermetabolic Conditions  Principal indications include malnutrition, sepsis, or
Normal or Moderate Malnutrition 25-30 kcal/kg/day
surgical or traumatic injury in seriously ill patients for
Mild Stress 25-30
whom use of the gastrointestinal tract for feedings is
Moderate Stress 30
Severe Stress 30-35
not possible
Burns 35-40  Total (Central) Parenteral Nutrition (TPN)requires
access to a large-diameter vein to deliver the nutritional
 Provision of 30 kcal/kg/d will adequately meet energy requirements of the individual
requirements in most postsurgical patients, with low o Dextrose content of the solution is high (15-
risk of overfeeding 25%)
o Overfeedingusually results from o All other macronutrients and micronutrients
overestimation of caloric needs because actual are deliverable by this route
body weight is used to calculate BEE,

TOPNOTCH MEDICAL BOARD PREP SURGERY SUPPLEMENT HANDOUT Page 4 of 83


For inquiries visit www.topnotchboardprep.com.ph or email us at topnotchmedicalboardprep@gmail.com
TOPNOTCH MEDICAL BOARD PREP SURGERY SUPPLEMENT HANDOUT - Jules Lopez,MD-MBA,Teddy Carpio,MD-MBA
For inquiries visit www.topnotchboardprep.com.ph or email us at topnotchmedicalboardprep@gmail.com
 Peripheral Parenteral Nutrition (PPN) uses lower
osmolarity of the solution to allow its administration via 2. Sepsis increases metabolic needs by approximately
peripheral veins what percentage?
o Reduced levels of dextrose (5-10%) and a. 25%
protein (3%) b. 50%
o Some nutrients cannot be supplemented c. 75%
because they cannot be concentrated into d. 100%
small volumes
o Not appropriate for repleting patients with Answer: B
severe malnutrition
o Used for short periods (<2 weeks); beyond Sepsis increases metabolic needs to approximately
this, TPN should be instituted 150-160% of resting energy expenditure, or 50%
 Complications are as follows (Refer to Table 4 ): above normal (Refer to Figure 4). This is mediated in
part by sympathetic activation and catecholamine
Table 4. Complication of Parenteral Nutrition  release.
Complications of Parenteral Nutrition
 Rare occurrences if IV vitamin 3. Which of the following is the initial enteric formula for
preparations are used the majority of surgical patients?
 However, Vitamin K is not part of any a. Low-residue isotonic formula
Vitamin Deficiencies
commercially prepared vitamin
b. Elemental formula
solution so it should be supplemented
on a weekly basis c. Calorie dense formula
 Clinically apparent during prolonged d. High protein formula
parenteral nutrition with fat-free
solutions Answer: A
Essential Fatty Acid  Manifests as dry, scaly dermatitis and
(EFA) Deficiency loss of hair Most low-residue isotonic formulas provide a caloric
 Prevented by periodic infusion of a fat density of 1.0 kcal/ml, and approximately 1500 to
emulsion at a rate equivalent to 10 to 1800 ml are required to meet daily requirements.
15% of total calories These provide baseline carbohydrates, protein,
 Essential trace minerals may be
electrolytes, water, fat, and fat-soluble vitamins. These
required after prolonged TPN
 Zinc deficiency is the most common
solutions usually are considered to be the standard or
that manifests as diffuse eczematoid first-line formulas for stable patients with an intact GI
Trace Mineral tract.
rash at intertriginous areas
Deficiencies
 Copper deficiency is associated with
Microcytic anemia
 Chromium deficiency is associated with FLUID AND ELECTROLYTE MANAGEMENT
Glucose intolerance OF THE SURGICAL PATIENT
 May occur after initiation of parenteral
nutrition
 Manifests as glycosuria A. Body Fluids and Compartments
 If blood glucose levels remain elevated B. Body Fluid Changes
or glycosuria persists, dextrose C. Fluid Therapy
Relative Glucose
concentration may be decreased, D. Special Case: Refeeding Syndrome
Intolerance
infusion rate slowed, or regular insulin E. Electrolyte Abnormalities
added to each bottle F. Acid-Base Disorders
 Rise in blood glucose may be
temporary, as the normal pancreas
increases its output of insulin in
response to the continuous A. BODY FLUIDS AND COMPARTMENTS
carbohydrate infusion  Water constitutes ~50-60% of total body weight
 Due to large glucose infusion, a  Relationship between total body weight and total body
Hypokalemia
significant shift of potassium from water (TBW) is relatively constant for an individual
extracellular to intracellular space may and is primarily a reflection of body fat
(and Metabolic
take place o Lean tissues (e.g. muscle and solid organs)
Acidosis)
 Manifests as glycosuria, which is
have higher water content than fat and bone
treated with potassium, NOT insulin
o TBW of average young adult male and
 Lack of intestinal stimulation is
associated with intestinal mucosal female is 60% and 50%, respectively of
atrophy, diminished villous height, total body weight
Intestinal Atrophy bacterial overgrowth, reduced  Estimates of %TBW should be adjusted downward
lymphoid tissue size, reduced ~10-20% for obese individuals and upward by ~10%
immunoglobulin A production, and for malnourished individuals
impaired gut immunity
 Highest percentage of TBW is found in newborns
(~80%)
REVIEW QUESTIONS a
 QUICK REVIEW a
1. Prostacyclin has which of the following effects in
systemic inflammation?
 TBW is ~50-60% of total body weight
a. Inhibition of platelet aggregation
b. Vasoconstriction  TBW (Male): 60%of total body weight
c. Increased adhesion molecules  TBW (Female): 50%of total body weight
d. Decreased cardiac output  Young lean males have a higher proportion of TBW than
elderly or obese individuals
Answer: A  Lower percentage of TBW in females generally
correlates with a higher percentage of adipose tissue and
Prostacyclin is a member of the eicosanoid family and lower percentage of muscle mass
is primarily produced by endothelial cells. It is an
effective vasodilator and also inhibits platelet
aggregation. During systemic inflammation,  TBW is divided into 3 functional fluid compartments
prostacyclin expression is impaired and thus the (Refer to Table 5):
endothelium favors a more procoagulant profile. o Plasma (extracellular)
TOPNOTCH MEDICAL BOARD PREP SURGERY SUPPLEMENT HANDOUT Page 5 of 83
For inquiries visit www.topnotchboardprep.com.ph or email us at topnotchmedicalboardprep@gmail.com
TOPNOTCH MEDICAL BOARD PREP SURGERY SUPPLEMENT HANDOUT - Jules Lopez,MD-MBA,Teddy Carpio,MD-MBA
For inquiries visit www.topnotchboardprep.com.ph or email us at topnotchmedicalboardprep@gmail.com
o Interstitial fluid (extracellular)
o Intracellular fluid

Table 5. Functional Body Fluid Compartments  Table 6. Normal Fluid Balance


Extracellular fluid PLASMA (1) Water Gain Water Loss
Total (1/3 of TBW or (5% of total body weight) Urine
Body 20% of total body INTERSTITIAL FLUID (2) Oral fluids 800-1200 ml
Water weight) (15% of total body weight) 1,500 ml Stool
(TBW) INTRACELLULAR FLUID (3) Sensible
250 ml
(2/3 of TBW or 40% of total body weight) Solid fluids Sweat
500 ml 0 ml
Water of oxidation Skin
 Extracellular fluid compartment (ECF) is balanced Insensible
250 ml 450 ml
between sodium (Na2+), the principal cation, and Water of solution Lungs
chloride (Cl-) and bicarbonate (HCO3-), the principal 0 ml 150 ml
anions(Refer to Figure 6)
o Composition of the plasma and interstitial fluid
differs only slightly in ionic composition 1. Extracellular Volume Deficit
o Slightly higher protein content (anions) in  Most common fluid disorder in surgical patients 
plasma results in a higher plasma cation  Can either be acute or chronic (Refer to Table 7)
composition relative to the interstitial fluid o Acute volume deficit is associated with
 Intracellular fluid compartment (ICF) is comprised cardiovascular and central nervous system
ofcations, potassium (K+) and magnesium (Mg2+), and signs
theanions, phosphate(HPO4-) and proteins o Chronic deficit displays tissue signs such as
 Concentration gradient between compartments is decrease in skin turgor and sunken eyes, in
maintained by adenosine triphosphate (ATP) driven addition to acute signs
sodium-potassium pumps located with the cell
Table 7. Signs and Symptoms of Volume Disturbances
membranes
System Volume Deficit Volume Excess
Weight loss Weight gain
Figure 6. Chemical composition of body fluid compartments General
Decreased skin turgor Peripheral edema
Tachycardia Increased cardiac output
Orthostasis / Increased central venous
Cardio Hypotension pressure
Collapsed neck veins Distended neck veins
Murmur
Oliguria --
Renal
Azotemia
GI Ileus Bowel edema
Pulmo -- Pulmonary edema

 Most common cause of volume deficit in surgical


patients is a loss of GI fluids from nasogastric suction,
vomiting, diarrhea, or enterocutanous fistula
(Refer to Table 8 )
 Third-space or nonfunctional ECF losses that occur with
sequestration secondary to soft tissue
injuries/infections, burns, and intraabdominal
processes such as peritonitis, obstruction, or prolonged
surgery can also lead to massive volume deficits

Table 8. Composition of GI Secretions 


Volume Volume Na+ K+ Cl- HCO3
(ml/24h) (ml/24h) (mEq/L) (mEq/L) (mEq/L) (mEq/L)
Saliva 1000 10 26 10 30
 Water is freely diffusible and distributed evenly Stomach 1000-2000 60-90 10 130 0
throughout all fluid compartments of the body Duodenum 1500 120-140 5-10 90-120 0
 Sodiumis confined to ECF and is associated with water Ileum 3000 140 5 104 30
Colon 750 60 30 40 0
o Sodium-containing fluids are distributed
Pancreas 600-800 135-145 5-10 70-90 115
throughout the ECF and add to bothplasma
Bile 300-800 135-145 5-10 90-110 30-40
(intravascular) and interstitial spaces
o Sodium-containing fluids expand the
2. Extracellular Volume Excess
interstitial space by ~3x as much as the plasma
 May be iatrogenic or secondary to renal dysfunction,
congestive heart failure, or cirrhosis
B. BODY FLUID CHANGES
 Both plasma and interstitial volumes are increased
 A healthy person consumes water an average of 2L/d,
~75% from oral intake and the rest extracted from solid  Symptoms are primarily pulmonary and cardiovascular
foods (Refer to Table 6) (Refer to Table 7)
 Daily water losses include 800-1200 ml in urine, 250 ml  In healthy patients, edema and hyperdynamic
in stool, and 600 ml in insensible losses through both circulation are common and well tolerated
the skin (75%) and lungs (25%)  However, the elderly and patients with cardiac disease
 Sensible water losses such as sweating or pathologic may quickly develop congestive heart failure and
loss of GI fluids vary widely, but these include loss of pulmonary edema in response to only a moderate
electrolytes as well volume excess
o Sweat is hypotonic and sweating usually
C. FLUID THERAPY
results in only a small sodium loss
o Pathologic GI losses are isotonic to slightly  Most commonly used solutions are as follows:
hypotonic and contribute little to net gain or (Refer to Table 9)
loss of free water
TOPNOTCH MEDICAL BOARD PREP SURGERY SUPPLEMENT HANDOUT Page 6 of 83
For inquiries visit www.topnotchboardprep.com.ph or email us at topnotchmedicalboardprep@gmail.com
TOPNOTCH MEDICAL BOARD PREP SURGERY SUPPLEMENT HANDOUT - Jules Lopez,MD-MBA,Teddy Carpio,MD-MBA
For inquiries visit www.topnotchboardprep.com.ph or email us at topnotchmedicalboardprep@gmail.com
o Acute volume deficits should be corrected as
much as possible
o Once a volume deficit is diagnosed, prompt
fluid replacement should be instituted, usually
with an isotonic crystalloid
Table 9. Electrolyte Solutions for Parenteral Administration o Patients whose volume deficit is not corrected
Solution Na+ Cl- K+ Ca2+ Other mOsm after initial volume challenge and those with
ECF 142 103 4 27 -- 280 impaired renal function and the elderly should
Lactated Lactate be considered for more intensive monitoring
130 109 4 28 280
Ringer’s (LR) 28 mEq/l
of central venous pressure or cardiac output in
0.9% Sodium
chloride 154 154 0 0 -- 308
an ICU setting
(PNSS) o If symptomatic electrolyte abnormalities
Dextrose accompany volume deficit, the abnormality
D5 Lactated should be corrected to the point that the acute
50 g/l
Ringer’s 130 109 4 3 560
Lactate symptom is relieved before surgical
(D5LR)
28 mEq/l intervention.
D5 Sodium
Dextrose
chloride 154 154 0 0 588
(D5NS)
50 g/l  QUICK REVIEW a
D5 0.45%
Sodium 77 77 0 0
Dextrose
434  Extracellular volume deficit is the most common fluid
50 g/l disorder in surgical patients
chloride
D5 0.25%
Dextrose
 Most common cause of volume deficit in surgical
Sodium 34 34 0 0 357 patients is a loss of GI fluids
50 g/l
chloride  Both PLR and PNSS are considered isotonic and are
useful in replacing GI losses and correcting extracellular
volume deficits
 Type of fluid administered depends on the patient’s  Hypertonic saline solution is used as a treatment
volume status and the type of concentration or modality in patients with closed head injuries
composition abnormality present (Refer to Table 10)

Table 10. Fluid Therapy


2. Intraoperative Fluid Therapy
Solution Description
 Considered isotonic BUT it is slightly
 With the induction of anesthesia, compensatory
Lactated hypotonic due to lactate mechanisms are lost, and hypotension will develop if
Ringer’s (PLR)  Useful in replacing GI losses and correcting volume deficits are not appropriately managed
extracellular volume deficits  To avoid hemodynamic instability intraoperatively, the
 Considered isotonic BUT it is mildly following should be ensured:
0.9% Sodium
hypertonic o Known fluid losses corrected
 Also useful in replacing GI losses and preoperatively
chloride
correcting extracellular volume deficits, o Adequate maintenance fluid therapy
(PNSS)
especially those associated with hyponatremia, provided
hypochloremia, and metabolic alkalosis
o Ongoing losses replaced intraoperatively
 Useful for replacement of ongoing GI losses as
well as for maintenance fluid therapy in the  Among the ongoing losses during surgery include
D5 0.45% distributional shifts via third space or nonfunctional ECF
postoperative period
Sodium losses seen in the following:
 Provides sufficient free water for insensible
chloride
losses and enough sodium to aid the kidneys in o Major open abdominal surgeries in the form of
adjustment of serum sodium levels bowel wall edema, peritoneal fluid, and the
D5 3.5-5% wound edema during surgery
 Hypertonic saline solution
Sodium o Large soft tissue wounds, complex fractures
 Used for correction of severe sodium deficits
chloride with associated soft tissue injury, and burns
 Hypertonic saline solution
 Replacement of ECF losses during surgery often
 Used as a treatment modality in patients with
closed head injuries
requires 500 to 1000 ml/hr of a balanced salt solution
D5 7%  Shown to increase cerebral perfusion and to support homeostasis
Sodium decrease intracranial pressure, thus decreasing  Addition of albumin or other colloid-containing
chloride brain edema solutions to intraoperative fluid therapy is NOT
 However, there also have been concerns of necessary
increased bleeding, because hypertonic saline is
an arteriolar vasodilator 3. Postoperative Fluid Therapy
 Should be based on the patient’s current estimated
volume status and projected ongoing fluid losses
1. Preoperative Fluid Therapy  Any deficits from either preoperative or intraoperative
 Preoperative evaluation of a patient’s volume status and losses should be corrected and ongoing requirements
pre-existing electrolyte abnormalities is an important should be included along with maintenance fluids
part of overall preoperative care  In the initial postoperative period, an isotonic solution
 Administration of maintenance fluidsis required in an should be administered
otherwise healthy individual on NPO before surgery o Adequacy of resuscitation should be based on
 The following is the formula used for calculating vital signs and urine output
maintenance fluids in the absence of pre-existing o All measured losses, including losses through
abnormalities (Refer to Table 11): vomiting, NGT, drains, and urine output as
well as insensible losses should be replaced
Table 11. Maintenance Fluid Computation
 After the initial 24 to 48 hours, fluids can be changed to
First 0-10 kg Give 100 ml/kg/dor 4 ml/kg/hr
5% dextrose to 0.45% saline in patients unable to
Next 10-20 kg Give additional 50 ml/kg/dor 2 ml/kg/hr tolerate enteral nutrition
Weight >20 kg Give additional 20 ml/kg/dor 1 ml/kg/hr  If normal renal function and adequate urine output are
present, potassium may be added to the IV fluids
 However, may surgical patients have volume and/or
electrolyte abnormalities associated with their surgicalD. SPECIAL CASE: REFEEDING SYNDROME 
disease  Refeeding syndrome potentially lethal condition that
can occur with rapid and excessive feeding of
TOPNOTCH MEDICAL BOARD PREP SURGERY SUPPLEMENT HANDOUT Page 7 of 83
For inquiries visit www.topnotchboardprep.com.ph or email us at topnotchmedicalboardprep@gmail.com
TOPNOTCH MEDICAL BOARD PREP SURGERY SUPPLEMENT HANDOUT - Jules Lopez,MD-MBA,Teddy Carpio,MD-MBA
For inquiries visit www.topnotchboardprep.com.ph or email us at topnotchmedicalboardprep@gmail.com
patients with severe underlying malnutrition due to
starvation, alcoholism, delayed nutritional support,
anorexia nervosa, or massive weight loss in obese
patients
Figure 8. Etiology of Hyponatremia
 Shift in metabolism from fat to carbohydrate substrate
stimulates insulin release, which results in the cellular
uptake of electrolytes, particularly phosphate,
magnesium, potassium, and calcium
 Severe hyperglycemia may result from blunted basal
insulin secretion
 To prevent its development, the following measures
should be done:
o Underlying electrolyte and volume deficits
should be corrected
o Thiamine should be administered before the
initiation of feeding
o Caloric repletion should be instituted slowly,
at 20 kcal/kg per day, and should gradually
increase over the first week
 Treatment ||Water restriction and, if severe, the
E. ELECTROLYTE ABNORMALITIES
administration of sodium
1. Hypernatremia
 If symptomatic, 3% normal saline should be used to
 Results from either a loss of free water or a gain of
increase the sodium by no more than 1 mEq/l/huntil
sodium in excess of water
the serum sodium reaches 130 mEq/l or symptoms are
 Associated with either an increased, normal, or improved
decreased extracellular volume (Refer to Figure 7)
 If asymptomatic, correction should increase the sodium
 Symptoms are rare until serum sodium concentration level by no more than 0.5 mEq/l/hr to a maximum
exceeds 160 mEq/l increase of 12 mEq/l/d
 Clinical manifestations || Mostly central nervous
system in nature (restlessness, irritability, seizures,
coma) due tohyperosmolarity
INTERNAL MEDICINE a
 May lead to subarachnoid hemorrhage and death
 Central Pontine Myelinosisis a consequence of rapid
Figure 7. Etiology of Hypernatremia correction of hyponatremia
 Characterized with seizures, weakness, paresis, akinetic
movements, and unresponsiveness
 May result in permanent brain damage and death
 MRI may assist in the diagnosis

3. Hyperkalemia
 Serum K+ concentration above the normal range of
3.5-5 mEq/l
 Caused by excessive K+ intake, increased release of K+
from cells, or impaired K+ excretion by the kidneys
(Refer to Table 12)
 Clinical manifestations || Mostly GI (nausea/vomiting,
diarrhea), neuromuscular (weakness, paralysis), and
cardiovascular (arrhythmia, arrest)
 ECG changes ||High peaked T waves  (early),
widened QRS complex, flattened P wave, prolonged PR
interval (first-degree block), sine wave formation and
 Treatment ||Management of water deficit ventricular fibrillation
 In hypovolemic patients, volume should be restored  Treatment ||Reducing total body K+, shifting K+ from
with normal saline before concentration abnormality is extracellular to intracellular space, and protecting cells
addressed from the effects of increased K+
 Once adequate volume is achieved, water deficit is  Exogenous sources of potassium should be removed,
replaced using a hypotonic fluid including K+ supplementation in IV fluids
 Rate of fluid administration should be titrated to  K+ can be removed from the body using a cation-
achieve a decrease in serum sodium concentration exchange resin such as Kayexalate that binds K+ in
of no more than 1 mEq/l/h exchange for Na+
 Overly rapid correction can lead to cerebral edema  Immediate measures also should include attempts to
and herniation shift K+ intracellularly with glucose, insulin and
bicarbonate infusion and nebulized salbutamol(10-
2. Hyponatremia 20 mg)
 Occurs when there is an excess of extracellular water  When ECG changes are present, calcium chloride or
relative to sodium calcium gluconate (5-10 ml of 10% solution) should be
 Extracellular volume can be high, normal, or low (Refer administered immediately
to Figure 8)  All measures are temporary, lasting from 1 to 4 hours
 In most cases, sodium concentration is decreased as a  Dialysis should be considered in severe hyperkalemia
consequence of either sodium depletion or dilution when conservative measures fail
 Symptomatic hyponatremia does not occur until serum
sodium level is 20 mEq/l 4. Hypokalemia
 Clinical manifestations || Primarily central nervous  More common than hyperkalemia in the surgical patient
system in origin (headache, confusion, seizures, coma)  Caused by inadequate K+ intake, excessive renal K+
associated increases in intracranial pressure excretion, K+ loss in pathologic GI secretions, or
intracellular shifts from metabolic alkalosis or insulin
therapy (Refer to Table 12)
TOPNOTCH MEDICAL BOARD PREP SURGERY SUPPLEMENT HANDOUT Page 8 of 83
For inquiries visit www.topnotchboardprep.com.ph or email us at topnotchmedicalboardprep@gmail.com
TOPNOTCH MEDICAL BOARD PREP SURGERY SUPPLEMENT HANDOUT - Jules Lopez,MD-MBA,Teddy Carpio,MD-MBA
For inquiries visit www.topnotchboardprep.com.ph or email us at topnotchmedicalboardprep@gmail.com
 Clinical manifestations || Primarily related to failure of
normal contractility of GI smooth muscle (ileus,
constipation), skeletal muscle (decreased reflexes,
weakness, paralysis), and cardiac muscle (arrest)  QUICK REVIEW a
 ECG changes || U waves, T-wave flattening, ST-
segment changes, and arrhythmias (with digitalis  Normal Na+: 135-145 mEq/l
therapy)  Symptomatichypernatremia are rare until serum
sodium exceeds 160 mEq/l
Table 12. Etiology of Potassium Abnormalities  Symptomatic hyponatremia does not occur until serum
Etiology of Potassium Abnormalities sodium level is 20 mEq/l
Increased Intake
 Potassium supplementation  Normal K+: 3.5-5 mEq/l
 Blood transfusions  Peaked T waves are the first ECG change seen in most
 Endogenous load/destruction: hemodialysis,
patients with hyperkalemia
rhabdomyolysis, crush injury, GI hemorrhage
Increased Release  T-wave flattening is seen in hypokalemia
Hyperkalemia  Hypokalemia causes decreased deep tendon reflexes
 Acidosis
 Rapid rise of extracellular osmolality while hypomagnesemia and hypocalcemia causes
(hyperglycemia or mannitol) increased deep tendon reflexes
Impaired Excretion
 Potassium-sparing diuretics
 Renal insufficiency/failure 5. Hypercalcemia
Inadequate Intake  Serum calcium level above the normal range of 8.5-
 Dietary, potassium-free IV fluids 10.5 mEq/l or an increase in ionized calcium above
 Potassium-deficient TPN
4.2-4.8 mg/dl
Excessive Potassium Excretion
 Hyperaldosteronism  Caused by primary hyperparathyroidism in the
Hypokalemia outpatient setting and malignancy in hospitalized
 Medications (Non-K+ sparing diuretics)
GI losses patients
 Direct loss of potassium from GI fluid  Clinical manifestations || Neurologic impairment,
(diarrhea) musculoskeletal weakness and pain, renal dysfunction,
 Renal loss of potassium and GI symptoms (Refer to Table 13)
 ECG changes || Shortened QT interval, prolonged PR
 Treatment || Potassium repletion, the rate is and QRS intervals, increased QRS voltage, T-wave
determined by the symptoms flattening and widening, and atrioventricular block
 Mild, asymptomatic hypokalemia: oral repletion is  Treatment is required when hypercalcemia is
adequate (KCl 40 mEq per enteral access x 1 dose) symptomatic, which usually occurs when the serum
 Asymptomatic hypokalemia, not tolerating enteral level exceeds 12 mEq/l
nutrition: KCl 20 mEq IV q2h x 2 doses  Critical level for serum calcium is 15 mEq/l, when
 If IV repletion is required, usually no more than 10 symptoms noted earlier may rapidly progress to death
mEq/h is advisable in an unmonitored setting  Treatment || Aimed at repleting the associated volume
 K+ supplementation can be increased to 40 mEq/h deficit and then inducing a brisk diuresis with normal
when accompanied by continuous ECG monitoring, and saline
even more in the case of imminent cardiac arrest from a
malignant arrhythmia associated hypokalemia 6. Hypocalcemia
 Caution should be done when oliguria or impaired renal  Serum calcium level below 8.5 mEq/l or a decrease in
function is coexistent the ionized calcium level below 4.2 mg/dl
 Causes include pancreatitis, malignancies associated
PHARMACOLOGY a with increased osteoclastic activity (breast and prostate
cancer), massive soft tissue infections such as
 K+sparing Diuretics are competitive antagonists that necrotizing fasciitis, renal failure, pancreatic and small
either block the actions of aldosterone at the distal bowel fistulas, hypoparathyroidism, toxic shock
convoluted tubule, or directly inhibit sodium channels syndrome, and tumor lysis syndrome
o Aldosterone antagonists: Spironolactoneand  Transient hypocalcemia also occurs after removal of a
Eplerenone parathyroid adenoma due to atrophy of the remaining
o Epithelial sodium channel blockers: Amiloride gland and avid bone remineralization
and Triamterene  Neuromuscular and cardiac symptoms do not occur
 Non K+-sparing Diureticsinclude loop diuretics and until the ionized fraction falls below 2.5 mg/dl
thiazides, which both inhibit Na+ and Cl- reabsorption  Clinical manifestations || Neuromuscular symptoms
o Loop diuretics (Furosemide) inhibit the Na+- with decreased cardiac contractility (Refer to Table 13)
K+-2Cl- cotransporter in the thick ascending  ECG changes || Prolonged QT interval, T-wave
limb of the loop of Henle inversion, heart block and ventricular fibrillation
o Thiazidesinhibit the Na+-Cl-transporter in the
distal tubule MICROBIOLOGY a

 Toxic Shock Syndromeis due to the Staphylococcus


aureus toxin, Toxic shock syndrome toxin (TSST-1)
 Clinical manifestations ||Fever, hypotension, sloughing
of the filiform papillae (strawberry tongue),
desquamating rash, andmulti-organ involvement
 Usually no site of pyogenic inflammation blood CS
negative
 Common in tampon-using menstruating women or in
patients with nasal packing for epistaxis
 Treatment || Remove the offending agent and to start
antibiotics (Clindamycin and Vancomycin)

TOPNOTCH MEDICAL BOARD PREP SURGERY SUPPLEMENT HANDOUT Page 9 of 83


For inquiries visit www.topnotchboardprep.com.ph or email us at topnotchmedicalboardprep@gmail.com
TOPNOTCH MEDICAL BOARD PREP SURGERY SUPPLEMENT HANDOUT - Jules Lopez,MD-MBA,Teddy Carpio,MD-MBA
For inquiries visit www.topnotchboardprep.com.ph or email us at topnotchmedicalboardprep@gmail.com
Anorexia Nausea/vomiting
Gastrointestinal Nausea/vomiting
Abdominal pain
PATHOLOGY a
Weakness Weakness
Bone pain Lethargy
 Tumor Lysis Syndromeconsists of multiple electrolyte Neuromuscular
Confusion Decreased reflexes
abnormalities that may be seen after initiation of Coma
cancer treatment Hypertension Hypotension Arrest
 Chemotherapy causes release of break-down products of Cardiovascular
Arrhythmia
dying cancer cells Worsening of digitalis
toxicity
 Among the electrolyte abnormalities include
Renal Polyuria -
hyperkalemia, hyperphosphatemia, hyperuricemia,
Decreased Serum Levels
and hypocalcemia System Calcium Magnesium
 Clinical consequences are acute uric acid nephropathy Hyperactive reflexes Hyperactive reflexes
and acute renal failure Paresthesias Muscle tremors
Muscle cramps Tetany
Carpopedal spasm Positive Chvostek’s
Neuromuscular
 Treatment || Calcium supplementation and correction Seizures and Trousseau’s
of other metabolic derangements Tetany signs
Trousseau’s sign1 Delirium and
 Asymptomatic hypocalcemia can be treated with oral Chvostek’s sign2 seizures (severe)
or IV calcium Cardiovascular Heart failure Arrhythmia
 Acute symptomatic hypocalcemia should be treated 1Spasm resulting from pressure applied to the nerves and vessels of the
with IV 10% calcium gluconate to achieve a serum upper extremity with a blood pressure cuff
concentration of 7-9 mg/dl 2 Spasm resulting from tapping over the facial nerve
 Associated deficits in magnesium, potassium, and pH
must also be corrected  Treatment || Magnesium supplementation
 Hypocalcemia will be refractory to treatment if  Correction of magnesium depletion can be oral if
coexisting hypomagnesemia is not corrected first asymptomatic and mild or IV if symptomatic and severe
 For those with severe deficits (<1 mEq/L) or those who
7. Hypermagnesemia are symptomatic, 1 to 2 g of magnesium sulfate may
 Rare but can be seen with severe renal insufficiency and be administered IV over 15 minutes or 2 minutes if
parallel changes in potassium excretion under ECG monitoring to correct torsades
o Magnesium-containing antacids and laxatives  To counteract the adverse side effects of a rapidly rising
can produce toxic levels in patients with renal magnesium level and correct hypocalcemia (frequently
insufficiency/failure associated with hypomagnesemia), simultaneous
o Excess intake in conjunction with TPN, or administration of calcium gluconateis done
rarely massive trauma, thermal injury, and
severe acidosis, may be associated with
symptomatic hypermagnesemia
 QUICK REVIEW a
 Clinical manifestations || Mainly GI with
 Normal Ca2+: 8.5-10.5 mEq/l
neuromuscular dysfunction and impaired cardiac
conduction (Refer to Table 13)  Normal ionized Ca2+: 4.2-4.8 mg/dl
 ECG changes || (similar to hyperkalemia) Increased PR  Treatment is required when hypercalcemia is
interval, widened QRS complex, elevated T waves symptomatic, when the serum level exceeds 12 mEq/l
 Treatment || Eliminate exogenous sources of  Symptomatic hypocalcemia do not occur until the
magnesium, correct concurrent volume deficits and ionized fraction falls below 2.5 mg/dl
correct acidosis if present  Hypocalcemia will be refractory to treatment if
 To manage acute symptoms, calcium chloride(5-10ml) coexisting hypomagnesemia is not corrected first
should be administered to immediately antagonize the
cardiovascular effects F. ACID-BASE DISORDERS
 If persistently elevated or with symptoms, dialysis may 1. Metabolic Acidosis
be necessary  Results from an increased intake of acids, an increased
generation of acids, or an increased loss of bicarbonate
8. Hypomagnesemia  Body compensates by producing buffers (extracellular
 Magnesium depletion is a common problem in bicarbonate and intracellular buffers from bone and
hospitalized patients, particularly in the critically ill muscle), increasing ventilation (Kussmaul's
 Result from alterations of intake, renal excretion and respirations),increasing renal reabsorption and
pathologic losses generation of bicarbonate, and increasing renal secretion
o Poor intake may occur in cases of starvation, of hydrogen
alcoholism, prolonged IV fluid therapy, and TPN  Evaluation of a patient with metabolic acidosis includes
with inadequate supplementation of determination of the anion gap (AG), an index of
Magnesium unmeasured anions
o Losses are seen in cases of increased renal o AG = Na+ – (Cl- + HCO3-)
excretion from alcohol abuse, diuretic use, o Normal: <12 mmol/l
administration of amphotericin B, and primary  Etiology of metabolic acidosis is listed inTable 14 
aldosteronism, as well as GI losses from
diarrhea, malabsorption, and acute pancreatitis
Table 14. Etiology of Metabolic Acidosis 
 Clinical manifestations || Neuromuscular and central
High Anion Gap Metabolic Acidosis (HAGMA)
nervous system hyperactivity, similar to those of
Mnemonic: “MUDPILES”
calcium deficiency
Exogenous acid ingestion  Methanol
 ECG changes || Prolonged QT and PR intervals, ST-  Ethylene glycol  Uremia (Renal failure)
segment depression, flattening or inversion of P waves,  Salicylate  Diabetic ketoacidosis
torsades de pointes, and arrhythmias  Methanol  Propylene glycol
 Can produce hypocalcemia and lead to persistent Paraldehyde
hypokalemia 

Table 13. Clinical Manifestations of Abnormalities in Ca2+ and Mg+


Increased Serum Levels
System Calcium Magnesium
TOPNOTCH MEDICAL BOARD PREP SURGERY SUPPLEMENT HANDOUT Page 10 of 83
For inquiries visit www.topnotchboardprep.com.ph or email us at topnotchmedicalboardprep@gmail.com
TOPNOTCH MEDICAL BOARD PREP SURGERY SUPPLEMENT HANDOUT - Jules Lopez,MD-MBA,Teddy Carpio,MD-MBA
For inquiries visit www.topnotchboardprep.com.ph or email us at topnotchmedicalboardprep@gmail.com
 Infection, Iron, Isoniazid  Measures to ensure adequate ventilation through
Endogenous acid production  Lactic acidosis bilevel positive airway pressure or endotracheal
 Ketoacidosis  Ethylene glycol intubationare also initiated
 Lactic acidosis  Salicylates
 Renal insufficiency

Norma Anion Gap Metabolic Acidosis (NAGMA) Table 16. Etiology of Respiratory Acidosis
Etiology of Respiratory Acidosis
Acid administration (HCl)
Narcotics
Mnemonic: “HARD UP” Central nervous system injury
Loss of bicarbonate  Hyperalimentation Pulmonary (secretions, atelectasis, mucus plug, pneumonia, pleural
 Acetazolamide (Carbonic effusion)
GI losses (diarrhea, fistulas) anhydrase inhibitor) Pain from abdominal or thoracic injuries or incisions
 Renal tubular acidosis Limited diaphragmatic excursion from intra-abdominal pathology
Ureterosigmoidoscopy  Diarrhea (abdominal distention, abdominal compartment syndrome, ascites)
 Ureteroenteric fistula
Renal tubular acidosis  Pancreticoduodenal
fistula 4. Respiratory Alkalosis
Carbonic anhydrase inhibitor  In the surgical patient, most cases are acute and
secondary to alveolar hyperventilation
 Causes include pain, anxiety, neurologic disorders
 Lactic acidosis is a common cause of severe metabolic (central nervous system injury and assisted ventilation),
acidosis in surgical patients drugs (salicylates), fever, gram-negative bacteremia,
 In circulatory shock, lactate is produced in the presence thyrotoxicosis, and hypoxemia
of hypoxia from inadequate tissue perfusion  Acute hypocapnia can cause an uptake of potassium and
 Treatment || Restore perfusion with volume phosphate into cells and increased binding of calcium to
resuscitation rather than to attempt to correct with albumin, leading to symptomatic hypokalemia,
exogenous bicarbonate hypophosphatemia, and hypocalcemia with
 With adequate perfusion, lactate is rapidly metabolized subsequent arrhythmias, paresthesias, muscle cramps,
by the liver and the pH level returns to normal and seizures
 Administration of bicarbonate for the treatment of  Treatment || Directed at the underlying cause
metabolic acidosis is controversial  Direct treatment of the hyperventilation using
o Overzealous administration of bicarbonate can controlled ventilation may also be required
lead to metabolic alkalosisand can be
associated with arrhythmias
 QUICK REVIEW a
o An additional disadvantage is that sodium
bicarbonate actually can exacerbate
 Evaluation of a patient with metabolic acidosis includes
intracellular acidosis
determination of the anion gap (AG) to differentiate
HAGMA from NAGMA (TIP: Memorize the mnemonics!)
2. Metabolic Alkalosis
 Normal AG is <12 mmol/l
 Results from the loss of fixed acids orgain of
 Treatment of metabolic acidosis is to restore perfusion
bicarbonate(Refer to Table 15)
with volume resuscitation rather than exogenous
 Majority of patients will have hypokalemia, because
bicarbonate
extracellular potassium ions exchange with intracellular
 Metabolic alkalosis is associated with hypokalemia
hydrogen ions and allow the hydrogen ions to buffer
excess HCO3
 Treatment || Includes replacement of the volume
deficit with isotonic saline and then potassium
replacement once adequate urine output is achieved REVIEW QUESTIONS a

Table 15. Etiology of Metabolic Alkalosis 1. A patient develops a high output fistula following
Increased bicarbonate generation abdominal surgery. The fluid is sent for evaluation with
Chloride losing (urinary chloride > 20 mEq/l) the following results: Na+ 135, K+ 5, Cl- 70. Which of the
 Mineralocorticoid excess following is the most likely source of the fistula?
 Profound potassium depletion a. Stomach
Chloride sparing (urinary chloride < 20 mEq/l) b. Small bowel
 Loss from gastric secretions (emesis or nasogastric suction) c. Pancreas
 Diuretics d. Biliary tract
Excess administration of alkali
 Acetate in parenteral nutrition Answer: C
 Citrate in blood transfusions The composition of pancreatic secretions is marked by
 Antacids
high level of bicarbonate (Refer to Table 8), compared
 Bicarbonate
 Milk-alkali syndrome
to other GI secretions. In this example, the patient has a
Impaired bicarbonate excretion total of 140 mEq of cation (Na+ + K+) and only 70 mEq of
Decreased glomerular filtration anion (Cl-). The remaining 70 mEq (to balance the 140
Increased bicarbonate reabsorption (hypercarbia or potassium mEq of cation) must be bicarbonate.
depletion)
2. A postoperative patient with a potassium of 2.9 is given
1 mEq/kg replacement with KCl (potassium chloride).
3. Respiratory Acidosis Repeat tests after the replacement show the serum K to
 Associated with retention of CO2 secondary to be 3.0. The most likely diagnosis is:
decreased alveolar ventilation a. Hypomagnesemia
 Principal causes are listed in Table 16 b. Hypocalcemia
 Because compensation is primarily a renal mechanism, c. Metabolic acidosis
it is a delayed response d. Metabolic alkalosis
 In the chronic form, partial pressure of arterial CO2
remains elevated and the bicarbonate concentration Answer: A
rises slowly as renal compensation occurs In cases in which potassium deficiency is due to
 Treatment || Directed at the underlying cause magnesium depletion, potassium repletion is difficult
unless hypomagnesemia is first corrected.
TOPNOTCH MEDICAL BOARD PREP SURGERY SUPPLEMENT HANDOUT Page 11 of 83
For inquiries visit www.topnotchboardprep.com.ph or email us at topnotchmedicalboardprep@gmail.com
TOPNOTCH MEDICAL BOARD PREP SURGERY SUPPLEMENT HANDOUT - Jules Lopez,MD-MBA,Teddy Carpio,MD-MBA
For inquiries visit www.topnotchboardprep.com.ph or email us at topnotchmedicalboardprep@gmail.com
Alkalosis will change serum potassium (a decrease in 0.3 o Thromboxane A2 (TXA2) ,potent constrictor
mEq/l for every 0.1 increase in pH above normal). This is of smooth muscle,is produced locally at the
not enough to explain the lack of response to repletion in site of injury
the patient. Metabolic acidosis would not decrease o Endothelin ,also a potent vasoconstrictor, is
potassium. Calcium does not play a role in potassium synthesized by injured endothelium and
metabolism. serotonin
o Bradykinin and fibrinopeptidesare capable
3. Which of the following is a cause of acute of contracting vascular smooth muscle.
hypophosphatemia?  Extent of vasoconstriction varies with the degree of
a. Chronic ingestion of magnesium containing vessel injury (more pronounced in vessels with medial
laxatives smooth muscles)
b. Insulin coma
c. Refeeding syndrome 2. Platelet Plug Formation
d. Rhabdomyolosis  Platelets do not normally adhere to each other or to the
vessel wall but during vascular disruption, they form a
Answer: C hemostatic plugthat aids in cessation of bleeding
Acute hypophosphatemia is usually caused by an  Injury to the intimal layer in the vascular wall exposes
intracellular shift of phosphate in association with von Willebrand's factor (vWF), a subendothelial
respiratory alkalosis, insulin therapy, refeeding protein, where platelets adhere via glycoprotein I/IX/V
syndrome, and hungry bone syndrome. Clinical  After adhesion, platelets initiate a release reaction that
manifestations include cardiac dysfunction or muscle recruits other platelets to seal the disrupted vessel
weakness but are usually absent until levels fall  The aforementioned process, mediated by adenosine
significantly. Refer to page 8 for a discussion on diphosphate (ADP) and serotonin,is reversible and is
refeeding syndrome. known as primary hemostasis
Magnesium containing laxatives can cause  In the second wave of platelet aggregation,
hypermagnesemia in patients with renal failure but does anotherrelease reaction occurs that results in
not affect phosphorous. Patients with insulin coma compaction of the platelets viaglycoprotein IIb/IIIa into
(hypoglycemia) are not at risk for hypophosphatemia. a plug
Rhabdomyolosis is associated with hyperkalemia and  With fibrinogen as a cofactor, this process, mediated by
hyperphosphatemia ADP, Ca2+, serotonin, TXA2, is irreversible

3. Fibrin Formation / Coagulation


HEMOSTASIS, SURGICAL BLEEDING,  As a consequence of the release reaction, alterations
AND TRANSFUSION occur in the phospholipids of the platelet membrane
that initiates coagulation
A. Hemostasis  Coagulation cascade typically has been depicted as two
B. Evaluation of Hemostatic Risk intersecting pathways
C. Surgical Bleeding o Intrinsic pathway begins with factor XII and
D. Special Cases through a series of enzymatic reactions, which
E. Transfusion is intrinsic to the circulating plasma and no
surface is required to initiate the process
o Extrinsic pathwayrequires exposure of
A. HEMOSTASIS tissue factor on the surface of the injured
 Function is to limit blood loss from an injured vessel vessel wall to initiate the arm of the cascade
 Four major physiologic events participate in the beginning with factor VII
hemostatic process(Refer to Figure 9): o The two arms of the coagulation cascade
o Vascular constriction merge to a common pathway at factor X, and
o Platelet plug formation activation of factors II (prothrombin) and I
o Fibrin formation (fibrinogen)proceeds in sequence
o Fibrinolysis  Secondary hemostasis or fibrin clot formation
Figure 9. Biology of Hemostasis occurs after conversion of fibrinogen to fibrin

4. Fibrinolysis
 During the wound-healing process, the fibrin clot
undergoes fibrinolysis, which permits restoration of
blood flow
 This is initiated at the same time as the clotting
mechanism under the influence of circulating kinases,
tissue activators, and kallikrein, which are present in
the vascular endothelium
 Plasmindegrades the fibrin mesh at various places,
which leads to the production of circulating fragments
that are cleared by proteases or by the kidney and liver

B. EVALUATION OF HEMOSTATIC RISK


1. Preoperative Evaluation of Hemostasis
 Most important component of the bleeding risk
assessment is a directed bleeding history
 When history is unreliable or incomplete or when
abnormal bleeding is suggested, a formal evaluation of
hemostasis should be performed before surgery
o Hemoglobin levels below 7 or 8 g/dl appear to
be associated with significantly more
1. Vascular Constriction perioperative complications
 Initial response to vessel injury o Determination of the need for preoperative
 Dependent on local contraction of smooth muscle transfusion must consider factors other than
the absolute hemoglobin level, including the

TOPNOTCH MEDICAL BOARD PREP SURGERY SUPPLEMENT HANDOUT Page 12 of 83


For inquiries visit www.topnotchboardprep.com.ph or email us at topnotchmedicalboardprep@gmail.com
TOPNOTCH MEDICAL BOARD PREP SURGERY SUPPLEMENT HANDOUT - Jules Lopez,MD-MBA,Teddy Carpio,MD-MBA
For inquiries visit www.topnotchboardprep.com.ph or email us at topnotchmedicalboardprep@gmail.com
presence of cardiopulmonary disease, type of
surgery, and likelihood of surgical blood loss
 Laboratory tests of hemostatic parameters in patients
with low risk of bleeding are NOT required

2. Evaluation of Intraoperative or Postoperative Bleeding


 Excessive bleeding during or after a surgical procedure
may be the result of ineffective hemostasis, blood
transfusion, undetected hemostatic defect, disseminated
intravascular coagulation (DIC) or consumptive
coagulopathy, and/or fibrinolysis  Treatment || Depends on the extent and cause of
platelet reduction
C. SURGICAL BLEEDING o Platelets are given preoperatively to rapidly
1. Systemic Bleeding Disorders increase the count in surgical patients
 Thrombocytopenia, secondary to any platelet o A count of >50,000/L generally requires no
pathology, is the most common abnormality of specific therapy
hemostasis that results in bleeding in surgical patients o One unit of platelet concentrate is expected
 Systemic causes of surgical bleeding can either be to increase the circulating platelet count by
inherited or acquired (Refer to Table 17) ~10,000/L in the average 70-kg person
 Inherited platelet functional defects include o In patients whose thrombocytopenia is
abnormalities of platelet surface proteins, abnormalities refractory to standard platelet transfusion,
of platelet granules, and enzyme defects the use of human leukocyte antigen (HLA)
 Acquired abnormalities of platelets may be quantitative compatible plateletshas proved effective
or qualitative, although some patients have both types
o Quantitative defects may be a result of failure 2. Local Hemostasis
of production due to bone marrow disorders,  Significant surgical bleeding usually is caused by
shortened survival, or sequestration ineffective local hemostasis
o Qualitative defects include massive  Goal is to prevent further blood loss from a disrupted
transfusionand drugs that interfere with vessel that has been incised or transected
platelet function  Hemostasis may be accomplished by interrupting the
flow of blood to the involved area or by direct closure of
Table 17. Etiology of Surgical Bleeding the blood vessel wall defect
Congenital Factor Deficiencies o Mechanical procedure: When pressure is
Coagulation Factor Deficiencies applied (whether through direct digital
von Willibrand’s Disease pressure, hemostatic clamp, or tourniquet) to
Platelet Functional Defects an artery proximal to an area of bleeding,
Acquired Hemostatic Defects
profuse bleeding may be reduced so that more
Platelet Abnormalities: Quantitative
definitive action is permitted
Leukemia
Myeloproliferative disorders
o Thermal agents: Heat (via cautery or
Failure of Vitamin B12 or Folate deficiency harmonic scalpel) achieves hemostasis by
Production Chemotherapy or radiation therapy denaturation of protein that results in
Acute alcohol intoxication coagulation of large areas of tissue
Viral infections o Topical hemostatic agents: Include physical
Immune-mediated disorders (Idiopathic or mechanical, caustic, biologic, and
thrombocytopenia, Heparin-induced physiologic agents that works either by
thrombocytopenia, Autoimmune disorders or B- inducing protein coagulation and precipitation
Decreased cell maligancies, Secondary thrombocytopenia) or activating biologic responses to bleeding
Survival Disseminated intravascular coagulation
Disorders related to platelet thrombi D. SPECIAL CASES
(Thrombocytopenic purpura, Hemolytic uremic
syndrome)
1. Disseminated Intravascular Coagulation (DIC)
Portal hypertension  An acquired syndrome characterized by intravascular
Sarcoid activation of coagulation
Sequestration  Can originate from and cause damage to the
Lymphoma
Gaucher’s disease microvasculature, which if sufficiently severe, can
Platelet Abnormalities: Qualitative produce organ dysfunction
Massive transfusion  Excessive thrombin generation leads to microthrombus
Therapeutic administration of platelet inhibitors formation, followed by consumption and depletion of
Myeloproliferative disorders coagulation factors and platelets, which leads to the
Disease states Monoclonal gammopathies classic picture of diffuse bleeding
Liver disease
 Causes include the following:
o Central nervous system injuries with
embolization of brain matter
PATHOLOGY a o Fractures with embolization of bone marrow
o Malignancy
 Bernard-Soulier Syndromeis caused by a defect in the o Organ injury (severe pancreatitis, liver failure)
glycoprotein Ib/IX/V receptor for vWF, leading to o Certain vascular abnormalities (aneurysms)
defective platelet adhesion o Others: snakebites, illicit drugs, transfusion
o Decreased platelet count reactions, transplant rejection, and sepsis
o Treatment || Platelet transfusion  Diagnosis is made on the basis of an inciting cause with
 Glanzmann Thrombastheniais caused by a defect in associated thrombocytopenia, prolonged PT, low
the platelet glycoprotein IIb/IIIa complex , leading to fibrinogen level, and elevated levels of fibrin
defective platelet aggregation markers (fibrin degradation products, D-dimer, soluble
o Normal platelet count fibrin monomers)
o Treatment || Platelet transfusion  Treatment || Relieving the patient's causative primary
medical or surgical problem and maintaining adequate
perfusion

TOPNOTCH MEDICAL BOARD PREP SURGERY SUPPLEMENT HANDOUT Page 13 of 83


For inquiries visit www.topnotchboardprep.com.ph or email us at topnotchmedicalboardprep@gmail.com
TOPNOTCH MEDICAL BOARD PREP SURGERY SUPPLEMENT HANDOUT - Jules Lopez,MD-MBA,Teddy Carpio,MD-MBA
For inquiries visit www.topnotchboardprep.com.ph or email us at topnotchmedicalboardprep@gmail.com
 If there is active bleeding, hemostatic factors should be with high risk of thrombosis
replaced using fresh frozen plasma (FFP), which Emergency Rapid reversal of anticoagulation can be
generally is sufficient to correct the hypofibrinogenemia surgery accomplished with FFP

2.Anticoagulation and Bleeding


 Spontaneous bleeding can be a complication of  Other drugs that interfere with platelet function are
anticoagulant therapy with either heparin, warfarin,or aspirin, clopidogrel, dipyridamole, and glycoprotein
low molecular weight heparin IIb/IIIa inhibitors
o Risk of spontaneous bleeding with heparinis o Aspirin inhibit platelet function through
relatively high but reduced with continuous irreversible acetylation of platelet
infusion technique prostaglandin synthase
o Therapeutic anticoagulation is more reliably o Clopidogrelinhibit platelet function through
achieved withlow molecular weight selective irreversible inhibition of ADP-
heparinbecause laboratory testing is not induced platelet aggregation
routinely done, which makes them attractive  General recommendation is that a period of ~7 days is
options for outpatient anticoagulation required from the time the drug is stopped until an
o Warfarin is used for long-term outpatient elective procedure can be performed
anticoagulation in various clinical conditions  Timing of urgent and emergent surgeries is unclear
including deep vein thrombosis, valvular heart  Preoperative platelet transfusions may be beneficial
disease (with or without prosthetic valves),
atrial fibrillation, and recurrent myocardial 3.Coagulopathy of Liver Disease
infarction  Liver plays a key role in hemostasis because it
synthesizes manycoagulation factors
PHARMACOLOGY a  Most common coagulation abnormalities associated
with liver dysfunction are thrombocytopenia and
 Warfarin inhibits vitamin K epoxide reductase and impaired humoral coagulation function manifested
thereby interferes with production of functional vitamin as prolonged PT and increase in the International
K-dependent clotting and anticlotting factors Normalized Ratio (INR)
 Side effect||bleeding, warfarin-induced skin necrosis  Thrombocytopenia is related to hypersplenism, reduced
production of thrombopoietin, and immune-mediated
destruction of platelets
Mnemonic: “Ethel Booba takes Phen-
phen and Refuses Greasy Carb Shakes”  Before any therapy for thrombocytopenia is initiated,
Cytochrome P450  Ethanol the actual need for correction should be strongly
Inducers considered
 Barbiturates
increase clearance  Treatment || Platelet transfusions; however, the
 Phenytoin
and reduce the effect typically lasts only several hours
 Rifampicin
anticoagulant  Potential alternative strategy is administration of
 Griseofulvin
effect of warfarin interleukin-11, a cytokine that stimulates proliferation
 Carbamazepine
of hematopoietic stem cells and megakaryocyte
 St. John’s Wort / Smoking progenitors
Mnemonic: “Inhibitors Stop Cyber Kids  Less well accepted option is splenectomy or splenic
from Eating GRApefruit Q” embolization to reduce hypersplenism but reduced
 Isoniazid splenic blood flow can reduce portal vein flow with
Cytochrome P450  Sulfonamides
inhibitors subsequent development of portal vein thrombosis.
 Cimetidine
reduce clearance
 Ketoconazole 4. Coagulopathy of Trauma
and increase the
 Erythromycin  Recognized causes of traumatic coagulopathy include
anticoagulant
 Grapefruit juice acidosis, hypothermia, and dilution of coagulation factors
effect of warfarin
 Ritonavir  Significant proportion of trauma patients arrive at the
 Amiodarone ER coagulopathic, and this early coagulopathy is
 Quinidine associated with increased mortality
 Shockhas been postulated to induce coagulopathy
 Bleeding complications can be manifested through systemic activation of anticoagulant and
throughhematuria, soft tissue bleeding, intracerebral fibrinolytic pathways
bleeding, skin necrosis, and abdominal bleeding  Hypoperfusion causes activation of thrombomodulin(on
 Bleeding into the abdominal cavity is the most the surface of endothelial cells), which complexes with
common complication of warfarin therapy circulating thrombin thereby inducing not only an
 Intramural bowel hematoma is the most common anticoagulant statebut also enhancing fibrinolysis
cause of abdominal pain in patients receiving
anticoagulation therapy 5. Massive Transfusion
 Certain surgical procedures should not be performed  Well-known cause of thrombocytopenia due to
such as procedures involving the central nervous hypothermia, dilutional coagulopathy, platelet
system or the eye dysfunction, fibrinolysis, or hypofibrinogenemia
 Impaired ADP-stimulated aggregation occurs with
Table 18. Reversal of anticoagulation for patients undergoing surgery massive transfusion(>10 units of packed RBC) leading to
Reversal of Heparin Therapy surgical bleeding
Not indicated when aPTT is <1.3 times the control value
Emergency Discontinue drug and use ofprotamine sulfate for D. TRANSFUSION
surgery more rapid reversal of anticoagulation  General indications for transfusion is listed in Table 19
Reversal of Warfarin Therapy
Not indicated when the INR is <1.5 Table 19. Indications for Replacement of Blood and its Elements
Discontinue drug several days before the operation General Indications for Transfusion
with monitoring of prothrombin concentration Improvement in  Oxygen-carrying capacity is primarily a
(>50% is safe) Oxygen function of RBC
Elective Parenteral administration of vitamin K is indicated Carrying  Therefore, transfusion of RBC should
surgery in patients with biliary obstruction or malabsorption Capacity augment oxygen-carrying capacity
who may be vitamin K deficient  Critically ill patients frequently receive
Low molecular weight heparin should be Treatment of
transfusions at a hemoglobin level
administered while the INR is decreasing in patients Anemia
approaching 9 g/dl
TOPNOTCH MEDICAL BOARD PREP SURGERY SUPPLEMENT HANDOUT Page 14 of 83
For inquiries visit www.topnotchboardprep.com.ph or email us at topnotchmedicalboardprep@gmail.com
TOPNOTCH MEDICAL BOARD PREP SURGERY SUPPLEMENT HANDOUT - Jules Lopez,MD-MBA,Teddy Carpio,MD-MBA
For inquiries visit www.topnotchboardprep.com.ph or email us at topnotchmedicalboardprep@gmail.com
 Most common indication for blood  Occurs in ~1% of all transfusions
transfusion in surgical patients is  Reactions usually are mild
thereplenishment of the blood volume  Clinical manifestations ||rash, urticaria, and fever
 Measurements of hemoglobin levels or
within 60 to 90 minutes of the start of the transfusion
hematocrit are frequently used to assess
blood loss  misleading in acute loss,  In rare instances, anaphylactic shock develops
because levels can be normal in spite of  Caused by transfusion of antibodies from
Volume severely contracted blood volume hypersensitive donors or the transfusion of antigens to
Replacement  Estimated total blood volume is 7-8% of which the recipient is hypersensitive
TBW  Can occur after the administration of any blood product
 Blood loss of up to 20% of total blood volume:  Treatment ||Administration of antihistamines or in
Replaced with crystalloid solution
more serious cases, use of epinephrine or steroids may
 Blood loss >20% of total blood
volume:Addition of packed RBC, and in the
be indicated
case of massive transfusion, the addition of
FFP 3. Respiratory Complications
 Circulatory overload can occur with rapid infusion of
blood, plasma expanders, and crystalloids, particularly
 QUICK REVIEW a in older patients with underlying heart disease
 Clinical manifestations ||dyspnea, rales,and cough
 Thrombocytopenia is the most common abnormality of  Treatment || Initiate diuresis, slow the rate of blood
hemostasis administration, and minimize delivery of fluids while
 Significant surgical bleeding usually is caused by blood products are being transfused
ineffective local hemostasis  Another significant respiratory complication is
 Most important management of DIC is treatment of the Transfusion-related Acute Lung Injury (TRALI)
underlying cause o Defined as noncardiogenic pulmonary edema
 Bleeding into the abdominal cavity is the most related to transfusion
common complication of warfarin therapy o Can occur with the administration of any
 Intramural bowel hematoma is the most common plasma-containing blood product
cause of abdominal pain in patients receiving o Clinical manifestations ||similar to those of
anticoagulation therapy circulatory overload and often accompanied
 A period of ~7 days is required from the time aspirin by fever, rigors, and bilateral pulmonary
and/or clopidogrel is stopped until an elective procedure infiltrates on chest radiograph
can be performed o Most commonly occurs within 1 to 2 hours
after the onset of transfusion, but virtually
 Most common coagulation abnormalities associated with
always before 6 hours
liver dysfunction are thrombocytopenia and impaired
humoral coagulation function o Etiology is not well established, but TRALI is
thought to be related to anti-HLA or anti-
 Most common indication for blood transfusion in
human neutrophil antigen antibodies in
surgical patients is volume replacement
transfused blood that primes neutrophils in
the pulmonary circulation.
o Treatment || Discontinuation of any
 Complications of transfusionis primarily related to
transfusion, notification of the transfusion
blood-induced proinflammatory responses
service, and provision of pulmonary support
 Complications (discussed below) occur in (from supplemental oxygen to mechanical
approximately 10% of all transfusions, but <0.5% are ventilation)
serious
4. Hemolytic Reactions
1. Febrile Non-hemolytic Reactions
 Can be classified as either acute ordelayed(Refer to
 Defined as an increase in temperature [>1°C (1.8°F)] Table 20)
associated with a transfusion
 Approximately 1% of all transfusions Table 20. Classification of Hemolytic Reactions
 Preformed cytokines in donated blood and recipient Classification of Hemolytic Reactions
antibodies reacting with donated antibodies are  Occur with the administration of ABO-
postulated causes incompatible blood
 Can be reduced by the use of leukocyte-reduced blood  Fatal in up to 6% of cases
products with  Contributing factors include technical or clerical
 Pretreatment with paracetamol reduces the severity of errors in the laboratory andadministration of
wrong blood type
the reaction
 Characterized by intravascular hemolysis and
 Rare but potentially lethal febrile reaction is secondary consequent hemoglobinemia and hemoglobinuria
to bacterial contamination of infused blood  Clinical manifestations ||pain at the site of
o Gram-negative organisms, especially transfusion, facial flushing, and back and chest
Acute
Yersinia enterocolitica and Pseudomonas pain, associated with fever, respiratory distress,
Hemolytic
species are the most common cause Reaction
hypotension, and tachycardia
o Most cases are associated with the  In anesthetized patients, diffuse bleeding and
administration of platelets hypotension are the hallmarks
o Pathogenesis is related to lability of factor V,  Positive Coombs' test is diagnostic
 Treatment || stop transfusion, get a sample of the
which appears necessary for this interaction
recipient's blood and send along with the suspect
o Results in sepsis and death in 25% of patients unit to the blood bank for comparison with the
o Clinical manifestations ||fever and chills, pretransfusion samples
tachycardia, and hypotension, GI symptoms  Urine output should be monitored and adequate
(abdominal cramps, vomiting, and diarrhea), hydration maintained to prevent precipitation of
and hemorrhagic manifestations such as hemoglobin within the tubules
hemoglobinemia, hemoglobinuria, and DIC  Reactions occur 2 to 10 days after transfusion
o If suspected, transfusion should be  Occur when an individual has a low antibody titer
discontinued and the blood cultured at the time of transfusion
Delayed
o Treatment ||Administration of oxygen,  Characterized by extravascular hemolysis, mild
Hemolytic
anemia, indirect hyperbilirubinemia, decreased
adrenergic blocking agents, and antibiotics Reaction
haptoglobin levels, low-grade hemoglobinemia
and hemoglobinuria
2. Allergic Reaction  Clinical manifestations ||fever and jaundice
TOPNOTCH MEDICAL BOARD PREP SURGERY SUPPLEMENT HANDOUT Page 15 of 83
For inquiries visit www.topnotchboardprep.com.ph or email us at topnotchmedicalboardprep@gmail.com
TOPNOTCH MEDICAL BOARD PREP SURGERY SUPPLEMENT HANDOUT - Jules Lopez,MD-MBA,Teddy Carpio,MD-MBA
For inquiries visit www.topnotchboardprep.com.ph or email us at topnotchmedicalboardprep@gmail.com
 Coombs' test usually yields a positive result 15% of the population. Therefore, the administration of
 Treatment || Do not usually require specific Rh positive blood is acceptable if Rh negative blood is
intervention not available. However, Rh positive blood should not be
transfused to Rh negative females who are of
childbearing age.
5. Transmission of Disease
 Among the diseases that have been transmitted by
transfusion are malaria, Chagas' disease, brucellosis,
and, very rarely, syphilis
 Transmission of hepatitis C virus and HIV-1has been
SURGICAL INFECTIONS AND SHOCK
dramatically minimized by the introduction of better
antibody and nucleic acid screening for these pathogens
A. Definitions
 Hepatitis B virus transmission may still occur in about 1
B. Surgical Wounds Classification
in 100,000 transfusions in nonimmune recipients
C. Prevention and Treatment of Surgical Infections
D. Infections of Significance in Surgical Patients
 QUICK REVIEW a E. Shock

 Gram-negative organismsare the most common cause


of bacterial contamination of infused blood, especially
A. DEFINITIONS
with platelet administration
1. Infection (Refer to Figure 10 )
 Identifiable source of microbial insult
 Transfusion-related Acute Lung Injury (TRALI) most
2. Systemic Inflammatory Response Syndrome (SIRS)
commonly occurs within 1 to 2 hours after the onset of
 Two or more of the following criteria met:
transfusion, but virtually always before 6 hours
o Temperature  38C or  36C
o Heart rate  90 beats per minute
 Acute hemolytic reaction is characterized by o Respiratory rate  20 breaths per minute or
intravascular hemolysis while delayed reaction is PaCO2 32 mmHg or mechanical ventilation
characterized by extravascular hemolysis o White blood cell count  12,000/uL or 
4,000/uL or  10% band forms
3. Sepsis
 SIRS + Identifiable source of infection
REVIEW QUESTIONS a 4. Severe Sepsis
 Sepsis + Organ dysfunction
1. What percentage of platelets can be sequestered in the 5. Septic Shock
spleen?  Sepsis + Cardiovascular collapse (needs vasopressors)
a. 15%
b. 30% Figure 10. Relationship between infection and SIRS. Sepsis is the
presence both of infection and SIRS, shown here as the intersection of
c. 45%
these two areas. Other conditions may cause SIRS as well (trauma,
d. 60% aspiration, etc.). Severe sepsis (and septic shock) are both subsets of
sepsis.
Answer: B
Platelets are anucleate fragments of megakaryoctes. The
normal circulating number of platelets ranges between
150,000 and 400,000/L. Up to 30% of circulating
platelets may be sequestered in the spleen. If not
consumed in a clotting reaction, platelets are normally
removed by the spleen and have an average life span of 7
to 10 days.

2. A patient on chronic warfarin therapy presents with


acute appendicitis. INR is 1.4. Which of the following is
the most appropriate management?
a. Proceed immediately with surgery without
stopping the warfarin
b. Stop the warfarin, give FFP, and proceed with B. SURGICAL WOUNDS CLASSIFICATION 
surgery *Based on the magnitude of bacterial load at the time of surgery
c. Stop the warfarin and proceed with surgery in **IR = Infection rate
8-12 hours 1. Clean (Class I)
d. Stop the warfarin and proceed with surgery in  Include those in which no infection is present
24-36 hours  Only skin microflora potentially contaminate the wound
 No hollow viscus (that contains microbes) is entered
Answer: A  Class ID wounds are similar except that a prosthetic
When the INR <1.5 in a patient taking warfarin, reversal device (e.g. mesh or valve) is inserted
of anticoagulation therapy may not be necessary. (Refer  Example: Hernia repair, Breast biopsy(IR: 1-5.4%)
to Table 18). However, meticulous surgical technique is
mandatory, and the patient must be observed closely 2. Clean/Contaminated (Class II)
throughout the postoperative period.  Include those in which a hollow viscus such as
respiratory, GI, or GU tracts with inherent bacterial flora
3. What percent of the population is Rh negative? is opened under controlled circumstances without
a. 5% significant spillage of contents
b. 15%  Example: Cholecystectomy, Elective GI surgery(not colon)
c. 25% (IR: 2.1-9.5%), Colorectal surgery(IR: 9.4-25%)
d. 35%
3. Contaminated (Class III)
Answer: B  Include open accidental wounds encountered early
Rh negative recipients should receive transfusion only of after injury, those with extensive introduction of
Rh negative blood. However, this groups represents only bacteria into a normally sterile area of the body due
TOPNOTCH MEDICAL BOARD PREP SURGERY SUPPLEMENT HANDOUT Page 16 of 83
For inquiries visit www.topnotchboardprep.com.ph or email us at topnotchmedicalboardprep@gmail.com
TOPNOTCH MEDICAL BOARD PREP SURGERY SUPPLEMENT HANDOUT - Jules Lopez,MD-MBA,Teddy Carpio,MD-MBA
For inquiries visit www.topnotchboardprep.com.ph or email us at topnotchmedicalboardprep@gmail.com
to major breaks in sterile technique (e.g. open cardiac Ceftriaxone
massage), gross spillage of viscus contents such as Breast, Hernia Cefazolin Vancomycin
from the intestine, or incision through inflamed albeit
nonpurulent tissue
 Example: Penetrating abdominal trauma, large tissue  Empiric therapycomprises the use of an antimicrobial
injury, enterotomy (IR: 3.4-13.2%) agent(s)when the risk of a surgical infection is high,
based on the underlying disease process (e.g. ruptured
appendicitis), or when significant contamination
during surgery has occurred (e.g. inadequate bowel
4. Dirty (Class IV) preparation or considerable spillage of colon contents)
 Include traumatic wounds in which a significant o Prophylaxis merges into empiric therapy in
delay in treatment has occurred and in which situations in which the risk of infection
necrotic tissue is present, those created in the increases markedly because of intraoperative
presence of overt infection as evidenced by the findings
presence of purulent material, and those created to o Limited to a short course of drug (3-5 days),
access a perforated viscus accompanied by a high and should be curtailed based on
degree of contamination microbiologic data (i.e. culture and sensitivity
 Example: Perforated diverticulitis, necrotizing soft tissue pattern) coupled with improvements in the
infections (IR: 3.1-12.8%) clinical course of the patient
o Manner in which therapy is used differs
C. PREVENTION AND TREATMENT OF SURGICAL INFECTIONS depending on whether the infection is
 Resident microflora of the skin and other barrier monomicrobial or polymicrobial
surfaces represent a potential source of microbes that
Table 22. General principles in empiric therapy
can invade the body during trauma, thermal injury, or
Empiric Therapy
elective or emergent surgical intervention
 Frequently are nosocomial infections
 Maneuvers to diminish the presence of exogenous occurring in postoperative patients, such
(surgeon and operating room environment) and asUTIs, pneumonia, or bacteremia
endogenous (patient) microbes consist of the use of  Therapy should be initiated in patients with
mechanical, chemical, and antimicrobial modalities, or a evidence of SIRS, coupled with evidence of local
combination of these methods infection (e.g., an infiltrate on chest X-ray plus a
 These modalities are NOT capable of sterilizing the positive Gram's stain in BAL sample)
hands of the surgeon or the skin or epithelial surfaces of  Within 24 to 72 hours, culture and sensitivity
Monomicrobial reports will allow directed antibiotic regimen
the patient BUT the inoculum can be reduced
 Empiric regimen for common infections are as
considerably follows:
 Thus, entry through the skin, into the soft tissue, and o UTI: 3-5 days
into a body cavity or hollow viscus invariably is o Pneumonia: 7-10 days
STILLassociated with the introduction of some o Bacteremia: 7-14 days
degree of microbial contamination o Osteomyelitis, endocarditis, or
 Therefore, antimicrobial agents should be given in prosthetic infections: 6-12
patients who undergo procedures that may be associated weeks
with the ingress of significant numbers of microbes(e.g.,  Primary therapeutic modality is source
control(discussed below) but antimicrobial
colonic resection) or in whom the consequences of any agents play an important role as well
type of infection due to said process would be dire (e.g.,  Culture results are of lesser importance in
prosthetic vascular graft infection) managing these infections, as it has been
demonstrated that only a limited group of
Polymicrobial
1. Appropriate Use of Antimicrobial Agents microbes predominate in the established
 Prophylaxis is the administration of an antimicrobial infection, selected from a large number present
agent(s)before and during the operative procedure at the time of initial contamination
to reduce the number of microbes that enter the tissue  As such, antibiotic regimen should NOT be
modified solely on the basis of culture
or body cavity
information
o Only a single dose of antibiotic is required,
and only for certain types of surgical
procedures (Refer to Table 21) 2. Source Control
o Patients who undergo complex, prolonged  Primary precept of surgical infectious disease therapy
procedures in which the duration of the consists of drainage of all purulent material,
operation exceeds the serum drug half-life debridement of all infected, devitalized tissue, and debris,
should receive an additional dose(s) and/or removal of foreign bodies at the site of infection,
o Administration of postoperative doses plus remediation of the underlying cause of infection
DOES NOT provide additional benefit, and o Discrete, walled-off purulent fluid collection
should be discouraged, as it is costly and is (abscess) requires drainage via percutaneous
associated with increased rates of microbial drain insertion or an incision and drainage
drug resistance o Ongoing source of contamination (e.g. bowel
perforation) or presence of an aggressive,
Table 21. Prophylactic therapy
rapidly-spreading infection (e.g. necrotizing
Site Antibiotic Alternative
Cardiovascular Cefazolin or Cefuroxime Vancomycin soft tissue infection) requires aggressive
Cefazolin, Cefotetan operative intervention, both to remove
Gastroduodenal Cefoxitin Fluoroquinolone contaminated material and infected tissue (e.g.
Ampicillin-sulbactam radical debridement or amputation) and to
Biliary tract with Ampicillin-sulbactam Fluoroquinolone + remove the initial cause of infection (e.g.
active infection Ticarcillin-clavulanate Clindamycin or bowel resection)
(cholecystitis) Piperacillin-tazobactam Metronidazole
Cefazolin+Metronidazole Gentamicin or D. INFECTIONS OF SIGNIFICANCE IN SURGICAL PATIENTS
Colorectal ,
Ertapenem Fluoroquinolone1. Surgical Site Infection (SSI)
Obstructed small
Ticarcillin-clavulanate plus Clindamycin
bowel
Piperacillin-tazobactam or Metronidazole  Infections of the tissues, organs, or spaces exposed
Aminoglycoside + by surgeons during surgery
Head and neck Cefazolin  Development of SSI is related to three factors
Clindamycin
Neurosurgery Cefazolin Vancomycin (Refer to Table 23):
Orthopedics Cefazolin Vancomycin o Patient factors
TOPNOTCH MEDICAL BOARD PREP SURGERY SUPPLEMENT HANDOUT Page 17 of 83
For inquiries visit www.topnotchboardprep.com.ph or email us at topnotchmedicalboardprep@gmail.com
TOPNOTCH MEDICAL BOARD PREP SURGERY SUPPLEMENT HANDOUT - Jules Lopez,MD-MBA,Teddy Carpio,MD-MBA
For inquiries visit www.topnotchboardprep.com.ph or email us at topnotchmedicalboardprep@gmail.com
o Local factors examination that reveals diffuse tenderness and
o Microbial factors guardingwithout localized findings, absence of
 Treatment || Prophylactic antibiotics reduce the pneumoperitoneum, presence of >100 WBCs/ml,
and microbes with a single morphology on Gram’s
incidence of SSI during certain types of procedures
stain on fluid obtained via paracentesis
o Single dose of an antimicrobial agent  Treatment || Antibiotic therapy for 14 to 21
should be administered immediately before days and removal of indwelling devices (e.g.,
commencing surgery for class ID, II, III, and peritoneal dialysis catheter or peritoneovenous
IV types of wounds shunt)
 Surgical management of the wound is also a critical  Occurs due to contamination of the peritoneal
determinant of the propensity to develop an SSI cavity due to perforation or severe
o Class I and II woundsmay be closed primarily inflammation and infection of an intra-
o Class III and IV wounds areallowed to heal by abdominal organ
Secondary  Examples: Appendicitis, perforation of any
secondary intention where superficial aspects
Microbial portion of the GI tract, or diverticulitis
of these wounds should be packed open only Peritonitis  Treatment ||Effective therapy requires source
control to resect or repair the diseased organ,
Table 23. Risk factors for development of surgical site infections debridement of necrotic, infected tissue and
Patient Factors debris, and administration of antimicrobial
Older age agents directed against aerobes and anaerobe
Immunosuppression  Develops by leakage from a GI anastomosis or
Obesity intra-abdominal abscess in patients in whom
Diabetes Mellitus standard therapy fails
Chronic inflammatory process  Common in immunosuppressed patients
Malnutrition  Microbes such as E. faecalis and faecium, S.
Peripheral vascular disease epidermidis, C. albicans, and P. aeruginosa can be
Anemia identified
Radiation  Abscess is diagnosed via abdominal CT
Chronic skin disease  Treatment || CT-guided percutaneous drainage
Carrier state (e.g. chronic staphylococcus carriage) for intra-abdominal abscess
Recent operation  Surgical intervention is reserved for patients
Local Factors with multiple abscesses, those with abscesses in
Poor skin penetration proximity to vital structures such that
Tertiary
Contamination of instruments percutaneous drainage would be hazardous, and
(persistent)
Inadequate antibiotic prophylaxis those in whom an ongoing source of
Peritonitis or
contamination (e.g., enteric leak) is identified
Prolonged procedure Postoperative
Local tissue necrosis Peritonitis  Necessity of antimicrobial agent therapy and
Hypoxia, hypothermia precise guidelines that dictate duration of
catheter drainage have NOT been established
Microbial Factors
 Short course (3 to 7 days) of antibiotics that
Prolonged hospitalization (leading to nosocomial organisms)
covers for aerobic and anaerobic bacteria can be
Toxin secretion
given
Resistance to clearance (e.g. capsule formation)
 Unfortunately, even with effective antimicrobial
agent therapy, this disease process is associated
 Surgical site infections are classified into with mortality rates of more than 50%
incisionaland organ/space infections  Drainage catheter is left in situ until the abscess
 Incisional infections are further subclassified into cavity collapse, its output is less than 10-20 ml/d
superficial (limited to skin and subcutaneous tissue) with no evidence of an ongoing source of
and deep incisional categories contamination and the patient's clinical
condition has improved
o Treatment ||Effective therapy for incisional
SSIs consists of incision and drainage
without the addition of antibiotics
3. Postoperative Nosocomial Infections
o Antibiotic therapy is reserved for patients in
 Include SSIs, UTIs, pneumonia, and bacteremia
whom evidence of significant cellulitis is
present, or who manifest concurrent SIRS  Most infections are related to prolonged use of
o Open wound often is allowed to heal by indwelling tubes and catheters for the purpose of
secondary intention, with dressings being urinary drainage, ventilation, and venous and arterial
changed twice a day access, respectively (Refer to Table 25).
o Use of topical antibiotics and antiseptics to
Table 25. Postoperative nosocomial infections
further wound healing remains unproven
Postoperative nosocomial infections
o Vacuum-assisted closure is increasingly used  Should be considered based on urinalysis with
in management of problem wounds and can be WBCs or bacteria, a positive test for leukocyte
applied to complex wounds in difficult esterase, or a combination of these elements
locations  Diagnosis is established after more than 104
 Treatment of organ/space infections is discussed in CFU/ml of microbes are identified by culture
Intra-Abdominal Infections section Postoperative techniques in symptomatic patients, or more
Urinary Tract than 105 CFU/ml in asymptomatic individuals
2.Intra-Abdominal Infections/Peritonitis Infection (UTI)  Treatment ||Single antibiotic therapy for 3 to
5 days
 Microbial contamination of the peritoneal cavity
 Indwelling urinary catheters should be
 Classified according to etiology (Refer to Table 24) removed as quickly as possible, typically
within 1 to 2 days, as long as the patients are
mobile
Table 24. Intra-abdominal infections  Associated with prolonged mechanical
Intra-abdominal Infections ventilation and is frequently due to pathogens
 Occurs when microbes invade the normally common in the nosocomial environment
sterile peritoneal cavity via hematogenous  Diagnosis should be made using the presence
dissemination from a distant source of infection of a purulent sputum, elevated leukocyte
or direct inoculation count, fever, and new chest x-ray abnormality
Primary Pneumonia
 More common among patients with ascites, and  Presence of two of the clinical findings, plus
Microbial
in those individuals who are undergoing chest x-ray findings, significantly increases the
Peritonitis
peritoneal dialysis likelihood of ventilator-associated
 Often monomicrobial and rarely require pneumonia
surgical intervention  Treatment ||Antibiotic therapy for 7 to 10
 Diagnosis is established based on physical days
TOPNOTCH MEDICAL BOARD PREP SURGERY SUPPLEMENT HANDOUT Page 18 of 83
For inquiries visit www.topnotchboardprep.com.ph or email us at topnotchmedicalboardprep@gmail.com
TOPNOTCH MEDICAL BOARD PREP SURGERY SUPPLEMENT HANDOUT - Jules Lopez,MD-MBA,Teddy Carpio,MD-MBA
For inquiries visit www.topnotchboardprep.com.ph or email us at topnotchmedicalboardprep@gmail.com
 Surgical patients should be weaned from  Treatment ||After airway is secured and
mechanical ventilation as soon as feasible, ventilation is adequate, fluid resuscitation and
based on oxygenation and inspiratory effort restoration of intravascular volume often will
improve perfusion
 Administration of vasoconstrictors will
E. SHOCK improve peripheral vascular tone, decrease
 Failure to meet the metabolic needs of the cell and vascular capacitance, and increase venous
return
the consequences that ensue
 If the patient's blood pressure has not
 Consists of inadequate tissue perfusion marked by responded, dopamine may be used
decreased delivery of required metabolic substrates  Results from failure of the heart as a pump,
and inadequate removal of cellular waste products as in arrhythmias or acute myocardial
(Refer to Figure 11) infarction (most common)
 Initial cellular injury that occurs is reversible; however,  Hemodynamic criteria include sustained
injury will become irreversible if tissue perfusion is hypotension (i.e. SBP <90 mmHg for at least 30
prolonged or severe enough such that, at the cellular minutes), reduced cardiac index (<2.2
L/min/m2), and elevated pulmonary artery
level, compensation is no longer possible
wedge pressure (>15 mmHg)
 Treatment ||Ensure adequate airway is
Figure 11. Pathways leading to decreased tissue perfusion and shock Cardiogenic present and ventilation is sufficient
 Treatment of cardiac dysfunction includes
maintenance of adequate oxygenation to
ensure adequate myocardial O2 delivery and
judicious fluid administration to avoid fluid
overload and development of cardiogenic
pulmonary edema
 Significant dysrhythmias and heart block must
be treated with antiarrhythmic drugs, pacing,
or cardioversion

 Form of cardiogenic shock that results from


mechanical impediment to circulation
leading to depressed cardiac output rather
than primary cardiac failure
 Causes include cardiac tamponade, pulmonary
Obstructive
embolism, tension pneumothorax, IVC
obstruction (DVT, gravid uterus), increased
intrathoracic pressure (neoplasm)
 Treatment || Dependent on the etiology of the
obstructive shock
 Soft tissue and bony injury lead to the
activation of inflammatory cells and the release
of circulating factors that modulate the
 Clinical manifestations of several physiologic responses immune response
are most often what lead practitioners to the diagnosis  These effects of tissue injury are combined
with the effects of hemorrhage, creating a more
of shock as well as guide the management of patients
complex and amplified deviation from
 Shock is classified into six types (Refer to Table 26) homeostasis.
Traumatic
Table 26. Types of Shock  Treatment || Correction of the individual
Postoperative nosocomial infections elements to diminish the cascade of
 Most common type proinflammatory activation, and includes
 Results from loss of circulating blood prompt control of hemorrhage, adequate
volumedue to loss of whole blood volume resuscitation to correct O2 debt,
(hemorrhagic shock), plasma, interstitial fluid debridement of nonviable tissue, stabilization
(bowel obstruction) of bony injuries, and appropriate treatment of
 Clinical and physiologic response is classified soft tissue injuries
according to the magnitude of volume loss
(Refer to Table 27)
Hypovolemic  Treatment || Instituted with diagnostic Table 27. Signs and symptoms of advancing stages of hypovolemic shock
evaluation to identify a bleeding source 
 Appropriate priorities are secure the airway, Class I Class II Class III Class IV
control source of blood loss, and IV volume Blood loss
resuscitation Up to 750 750-1,500 1,500-2,000 >2,000
(ml)
 Patients who fail to respond to initial Blood loss
resuscitative efforts should be assumed to have (%blood Up to 15% 15-30% 30-40% >40%
ongoing active hemorrhage and require volume)
prompt operative intervention Pulse rate <100 >100 >120 >140
 Results from decreased resistance within Blood
capacitance vessels Normal Normal Decreased Decreased
pressure
 Evaluationbegins with an assessment of the Pulse Normal or
Decreased Decreased Decreased
adequacy of airway and ventilation pressure increased
 Treatment ||Fluid resuscitation and Respiratory
14-20 20-30 30-40 >35
restoration of circulatory volume rate
Vasogenic  Empiric antibiotics must be chosen carefully Urine
(Septic) (gram-negative rods, gram-positive cocci, and output >30 20-30 5-15 Negligible
anaerobes) (ml/h)
 However, IV antibiotics without source control Confused
will be insufficient to adequately treat patients CNS/mental Slightly Mildly Anxious and
and
with infected fluid collections, infected foreign status anxious anxious confused
lethargic
bodies, and devitalized tissue
 Vasopressors may be necessary as well
 Form of vasogenic shock in which spinal cord
injury or spinal anesthesia causes
Neurogenic
vasodilation due to acute loss of
sympathetic vascular tone
TOPNOTCH MEDICAL BOARD PREP SURGERY SUPPLEMENT HANDOUT Page 19 of 83
For inquiries visit www.topnotchboardprep.com.ph or email us at topnotchmedicalboardprep@gmail.com
TOPNOTCH MEDICAL BOARD PREP SURGERY SUPPLEMENT HANDOUT - Jules Lopez,MD-MBA,Teddy Carpio,MD-MBA
For inquiries visit www.topnotchboardprep.com.ph or email us at topnotchmedicalboardprep@gmail.com
beds but produce a dramatic reduction in splanchnic
blood volume, which holds 20% of the blood volume.

3. Which of the following best describes the hemodynamic


response to neurogenic shock?
a. Increased cardiac index, unchanged venous
capacitance
b. Increased cardiac index, decreased venous
INITIAL FLUID RESUSCITATION FOR HEMORRHAGIC SHOCK capacitance
GOAL: To re-establish tissue perfusion and oxygenation c. Variable change in cardiac index (can increase
or decrease), increased venous capacitance
 Crystalloid is the first fluid of choice for resuscitation.
d. Variable change in cardiac index (can increase
Immediately administer 2 L of isotonic sodium
or decrease), decreased venous capacitance
chloride solution or lactated Ringer’s solution in
response to shock from blood loss. Fluid
Answer: A
administration should continue until the patient's
Choice B and D are most commonly associated with
hemodynamics become stabilized. Because
septic shock. Choice C, on the other hand, is most likely
crystalloids quickly leak from the vascular space,
seen in cardiogenic shock.
each liter of fluid expands the blood volume by 20-
30%; therefore, 3 L of fluid need to be administered
4. An unconscious patient with a systolic BP of 80 and a HR
to raise the intravascular volume by 1 L.
of 80 most likely has?
a. Cardiogenic shock
 Endpoints in resuscitation can be divided into
b. Hemorrhagic shock
systemic or global parameters, tissue-specific
c. Neurogenic shock
parameters, and cellular parameters (Refer to Table
d. Obstructive shock
28)

 Global endpoints include vital signs, cardiac output, Answer: C


pulmonary artery wedge pressure, O2 delivery and Sympathetic input to the heart, which normally increases
consumption, lactate, and base deficit heart rate and cardiac contractility, and input to the
adrenal medulla, which increases catecholamine release,
may also be disrupted (with spinal cord injury),
Table 28. Endpoints in resuscitation preventing the typical reflex tachycardia that occurs with
Systemic/Global hypovolemia.
Lactate The classic description of neurogenic shock consist of
Base deficit decreased blood pressure associated with
Cardiac output bradycardia (absence of reflex tachycardia due to
Oxygen delivery and consumption disrupted sympathetic discharge), warm extremities
Tissue Specific
(loss of peripheral vasoconstriction), motor and sensory
Gastric tonometry
deficits indicative of a spinal cord injury, and
Tissue pH, Oxygen, Carbon dioxide levels
Near infrared spectroscopy
radiographic evidence of a vertebral column fracture.
Cellular
Membrane potential
Adenosine triphosphate TRAUMA

A. General Principle
REVIEW QUESTIONS a B. Primary Survey
C. Resuscitation
1. Which of the following is the most effective dosing of D. Secondary Survey
antibiotics in a patient undergoing elective colon E. Diagnostic Evaluation
resection? F. Definitive Care
a. A single dose given within 30 min prior to skin
incision
b. A single doe given at the time of skin incision A. GENERAL PRINCIPLE
c. A single preoperative dose + 24 hours of  Trauma or injury is a cellular disruption caused by
postoperative antibiotics an exchange with environmental energy that is
d. A single preoperative dose + 48 hours of beyond the body’s resilience
postoperative antibiotics  Most common cause of death for all individuals
between the ages of 1 and 44 years
Answer: A  Third most common cause of death regardless of age
Prophylaxis is the administration of an antimicrobial
 Most common cause of years of productive life lost
agent(s) before and during the operative procedure to
 Initial management of seriously injured patients
reduce the number of microbes that enter the tissue or
according to the Advanced Trauma Life Support (ATLS)
body cavity. Only a single dose of antibiotic is required,
consists of the following:
and only for certain types of surgical procedures. There
o Primary survey
is no evidence that administration of postoperative
o Concurrent resuscitation
doses provides additional benefit.
o Secondary survey
o Diagnostic evaluation
2. What percentage of the blood volume is normally in the
o Definitive care
splanchnic circulation?
a. 10%
B. PRIMARY SURVEY
b. 20%
 Goal is to identify and treat conditions that constitute an
c. 30%
immediate threat to life (Refer to Table 29)
d. 40%
 Assessment of the “ABCDE” (Airway with cervical
spine protection, Breathing, Circulation, Disability,
Answer: B
Most alterations in cardiac output in the normal heart and Exposure)
are related to changes in preload. Increases in
Table 29. Life-threatening injuries identified during the primary survey
sympathetic tone have a minor effect on skeletal muscle Airway
TOPNOTCH MEDICAL BOARD PREP SURGERY SUPPLEMENT HANDOUT Page 20 of 83
For inquiries visit www.topnotchboardprep.com.ph or email us at topnotchmedicalboardprep@gmail.com
TOPNOTCH MEDICAL BOARD PREP SURGERY SUPPLEMENT HANDOUT - Jules Lopez,MD-MBA,Teddy Carpio,MD-MBA
For inquiries visit www.topnotchboardprep.com.ph or email us at topnotchmedicalboardprep@gmail.com
Airway obstruction performed immediately before a chest
Airway injury radiograph is obtained
Breathing  Occurs with full thickness loss of the chest
Tension pneumothorax wall, permitting free communication
Open pneumothorax between the pleural space and the
Flail chest with underlying pulmonary contusion atmosphere
Circulation  Compromises ventilation due to
equilibration of atmospheric and pleural
Massive hemothorax or hemoperitoneum
pressures, which prevents lung inflation
Open
Hemorrhagic shock Mechanically unstable pelvis fracture and alveolar ventilation, and results in
Pneumothorax
hypoxia and hypercarbia
Extremity losses (Sucking chest
 Complete occlusion of the chest wall defect
Cardiogenic shock: Cardiac tamponade wound)
WITHOUT a tube thoracostomy may
Neurogenic shock: Cervical spine injury convert an open pneumothorax to a
Disability tension pneumothorax
Intracranial hemorrhage/mass lesion  Treatment ||Definitive treatment is closure
of the chest wall defect and closed tube
1. Airway management with cervical spine protection thoracostomy remote from the wound
 Ensuring a patent airway is the first priority in the (Refer to Figure 11)
primary survey  Occurs when 3 or more contiguous ribs
are fractured in at least 2 locations
 Efforts to restore cardiovascular integrity will be futile (Refer to Figure 12)
unless the oxygen content of the blood is adequate  Paradoxical movement of this free floating
 All patients with blunt trauma require cervical spine segment of chest wall may be evident in
immobilization (hard collar or placing sandbags on Flail chest with patients with spontaneous ventilation, due
both sides of the head with the patient’s forehead taped underlying to the negative intrapleural pressure of
across bags to the backboard) until injury is excluded pulmonary inspiration
 Patients who are conscious, do not show tachypnea, and contusion  Associated pulmonary contusion is
typically the source of postinjury
have a normal voice do not require early attention to the
pulmonary dysfunction (Decreased
airway EXCEPT the following: compliance and increased shunt fraction)
o Patients with penetrating injuries to the neck  Treatment ||May require presumptive
and an expanding hematoma intubation and mechanical ventilation
o Evidence of chemical or thermal injury to the
mouth, nares, or hypopharynx
o Extensive subcutaneous air in the neck Figure 12. Tension Pneumothorax (inset) with Needle Thoracostomy
o Complex maxillofacial trauma
o Airway bleeding
 Elective intubation should be performed on the cases
above before evidence of airway compromise
 Altered mental status is the most common
indication for intubation
 Options for endotracheal intubation include
nasotracheal, orotracheal, or surgical routes
o Nasotracheal: Only done in patients, who are
breathing spontaneously, requiring emergent
airway support in whom chemical paralysis
cannot be used
o Orotracheal: most common technique used
to establish a definitive airway
o Surgical (cricothyroidotomy): Done in
2nd ICS
patients in whom attempts at intubation have
failed or who are precluded from intubation
due to extensive facial injuries
o Surgical (emergent tracheostomy): Indicated Figure 13. Closed Tube Thoracostomy (CTT).A. Performed in the MAL at
in patients with laryngotracheal separation or the 4th-5th ICS to avoid iatrogenic injury to the liver or spleen. B. Heavy
laryngeal fractures, in whom scissors are used to cut through the intercostal muscle into the pleural
cricothyroidotomy may cause further damage space done on top of the rib to avoid injury to the intercostal bundle
or result in complete loss of airway located just beneath the rib. C. Incision is digitally explored to confirm
intrathoracic location and identify pleural adhesions. D. A 36F chest tube
2. Breathing and Ventilation is directed superiorly and posteriorly with the aid of a large clamp.
 Once a secure airway is obtained, adequate oxygenation
and ventilation must be assured
 All injured patients should receive supplemental oxygen
and be monitored by pulse oximetry
 The following conditions constitute an immediate threat
to life due to inadequate ventilation (Refer to Table 30)

Table 30. Life-threatening injury identified due to inadequate ventilation


Inadequate Ventilation
 Diagnosis is implied by respiratory distress
and hypotension in combination with any of
the following physical signs in patients with
chest trauma: tracheal deviation away from
the affected side, lack of or decreased breath
sounds on the affected side, and
Tension
subcutaneous emphysema on the affected
Pneumothorax
side(Refer to Figure 10inset)
 Treatment ||Needle thoracostomy
decompressionin the 2nd ICS in the MCL
may be indicated in the acute setting(Refer
to Figure 10)
 Closed tube thoracostomy should be
TOPNOTCH MEDICAL BOARD PREP SURGERY SUPPLEMENT HANDOUT Page 21 of 83
For inquiries visit www.topnotchboardprep.com.ph or email us at topnotchmedicalboardprep@gmail.com
TOPNOTCH MEDICAL BOARD PREP SURGERY SUPPLEMENT HANDOUT - Jules Lopez,MD-MBA,Teddy Carpio,MD-MBA
For inquiries visit www.topnotchboardprep.com.ph or email us at topnotchmedicalboardprep@gmail.com
 Patients with a SBP <70 mmHg warrant
emergency department thoracotomy(EDT)
with opening of the pericardium to address
the injury (Refer to Table 31)
 EDT is best accomplished using aleft
anterolateral thoracotomy, with the
incision started to the right of the sternum
(Refer to Figure 15)
Figure 14. Mechanism of a Flail Chest.Paradoxical movement of the flail
chest during inspiration and expiration.
4. Disability and Exposure
 Glasgow Coma Scale (GCS) score should be determined
for all injured patients
 Scores of 13 to 15 indicate mild head injury, 9 to 12
moderate injury, and <9 severe injury
 Abnormal mental status should prompt an immediate
re-evaluation of the ABCs and consideration of central
nervous system injury

Figure 15. Cardiac Tamponade with ultrasound findings (*) on the left.

3. Circulation with hemorrhage control


 Initial approximation of the patient’s cardiovascular
status can be obtained by palpating peripheral pulses
o Carotid pulse: 60 mmHg systolic BP
o Femoral pulse: 70 mmHg Figure 16. Pericardiocentesis. Access to the pericardium is obtained
o Radial pulse: 80 mmHg to be palpable through a subxiphoid approach, with the needle angled 45 degrees up
from the chest wall and toward the left shoulder.
 Any hypotensive episode (SBP <90 mmHg) is assumed
to be caused by hemorrhage until proven otherwise
 IV access for fluid resuscitation is obtained with 2
peripheral catheters, 16-gauge or larger in adults
 In patients under 6 years old, an intraosseus needle can
be placed in the proximal tibia (preferred) or distal
femur of an unfractured extremity
 External control of hemorrhage should be achieved
promptly while circulating volume is restored
 The following conditions constitute an immediate threat
to life due to inadequate circulation (Refer to Table 31)

Table 31. Life-threatening injury identified due to inadequate circulation


Inadequate Circulation
 Defined as >1,500 ml of blood or, in the
pediatrics, 1/3 of the patient’s blood
volume in the pleural space
 After a blunt trauma, hemothorax is usually
due to multiple rib fractures with severed
Massive
intercostal arteries, but occasionally
Hemothorax
bleeding isfrom lacerated lung parenchyma
 After a penetrating trauma, a systemic or
pulmonary hilar vessel injury should be Table 32. Emergency Department Thoracotomy (EDT) Indications and
presumed Contraindications. CPR = Cardiopulmonary resuscitation
 Treatment || Operative intervention Indications
 Occurs most commonly after penetrating Patients sustaining witnessed penetrating
thoracic injuries, although occasionally blunt Salvegeable trauma with <15 min of prehospital CPR
rupture of the heart, particularly the atrial postinjury cardiac Patients sustaining witnessed blunt trauma
appendage, is seen arrest with <5 min of
 <100 ml of pericardial blood may cause prehospital CPR
pericardial tamponade Persistent severe Cardiac tamponade
(Refer to Figure 13 right) postinjury Hemorrhage (intrathoracic, intra-
 Beck’s triad (dilated neck veins, muffled hypotension abdominal, extremity, cervical)
heart tones, and a decline in arterial pressure) (SBP60 mmHg) Air embolism
is NOT often observed Contraindications
Diagnosis is best achieved by ultrasound of
Penetrating trauma: CPR >15 min and no signs of life (papillary
the pericardium
Cardiac response, respiratory effort, motor activity)
(Refer to Figure 13 left)
Tamponade Blunt trauma: CPR >5 min and no signs of life or asystole
 Early in the course of tamponade, blood
pressure and cardiac output will transiently
Figure 17. Emergency department thoracotomy (EDT) is performed
improve with fluid administration
through the 5th ICS using the anterolateral approach. Pericardium is
 Treatment ||Pericardiocentesis is successful
opened anterior to the phrenic nerve, and the heart is rotated out for
in decompressing tamponade in
repair
approximately 80% of cases (Refer to
Figure 14)
 Removing as little as 15 to 20 ml of blood will
often temporarily stabilize the patient’s
hemodynamic status, prevent subendocardial
ischemia, and associated lethal arrhythmias,
and allow transport to the OR for sternotomy
TOPNOTCH MEDICAL BOARD PREP SURGERY SUPPLEMENT HANDOUT Page 22 of 83
For inquiries visit www.topnotchboardprep.com.ph or email us at topnotchmedicalboardprep@gmail.com
TOPNOTCH MEDICAL BOARD PREP SURGERY SUPPLEMENT HANDOUT - Jules Lopez,MD-MBA,Teddy Carpio,MD-MBA
For inquiries visit www.topnotchboardprep.com.ph or email us at topnotchmedicalboardprep@gmail.com
o Nonresponders: These patients have
persistent hypotension despite aggressive
resuscitation
 Patients with ongoing hemodynamic instability,
whether nonresponders or transient responders, require
systematic evaluation and prompt intervention

D. SECONDARY SURVEY
ANATOMY a  Once the immediate threats to life have been addressed,
a thorough history is obtained and the patient is
 Closed Tube Thoracostomy (CTT)is done on the examined in a systematic fashion
superior border of the lower ribon the 4th-5th ICS MAL  Patient (or surrogate) should be queried to obtain an
o Directed superiorly for air drainage “AMPLE” (Allergies, Medications, Past illnesses or
o Directed inferiorly for fluid drainage Pregnancy, Last meal, and Events related to the
o Tube passes through the following: Skin  injury)
Superficial fascia  Serratus anterior   Physical examination should be head to toewith special
External intercostals  Internal intercostals  attention to the patient's back, axilla, and perineum,
Innermost intercostals  Endothoracic fascia  because injuries here are easily overlooked
Parietal pleura  All potentially seriously injured patients should
undergo digital rectal examination to evaluate for
sphincter tone, presence of blood, rectal perforation, or a
 QUICK REVIEW a high-riding prostate, which is particularly critical in
patients with suspected spinal cord injury, pelvic
 Primary survey consists of the assessment of the fracture, or transpelvic gunshot wounds
“ABCDE” (Airway with cervical spine protection,  Vaginal examination with a speculum also should be
Breathing, Circulation, Disability, and Exposure) performed in women with pelvic fractures to exclude an
 Ensuring a patent airway is the first priority in the open fracture
primary survey
 Altered mental status is the most common indication E. DIAGNOSTIC EVALUATION
for intubation  Selective radiography and laboratory tests are done
 Massive hemothorax is defined as >1,500 ml of blood early in the evaluation after the primary survey
or, in the pediatrics, 1/3 of the patient’s blood volume  For patients with severe blunt trauma, lateral cervical
in the pleural space spine, chest, and pelvic radiographs should be
 Tension pneumothorax is the most common cause of obtained, often termed the big three
cardiogenic shock in trauma patients  For patients with truncal gunshot wounds,
anteroposterior and lateral radiographs of the chest
and abdomen are warranted
C. RESUSCITATION  In critically injured patients, blood samples for a routine
 Quantity of acute blood loss correlates with trauma panel (type and cross-match, complete blood
physiologicabnormalities (Refer to Table 26) count, blood chemistries, coagulation studies,
o Tachycardia is often the earliest sign of lactate level, and arterial blood gas analysis) should
ongoing blood loss but watch out for relative be sent to the laboratory
tachycardia (HR<90 in patients with a resting  For less severely injured patients only a complete blood
pulse rate in the 50s) count and urinalysis may be required
o Bradycardia, an ominous sign, occurs with
severe blood loss, often heralding impending F. DEFINITIVE CARE
cardiovascular collapse  All injured patients undergoing an operation should
o Hypotension is NOT a reliable early sign of receive preoperative antibiotics
hypovolemia, because blood volume must  Extended postoperative antibiotic therapy is
decrease by >30% before hypotension occurs administered only for open fractures or significant intra-
 Goal is to re-establish tissue perfusion abdominal contamination
o Urine output is a quantitative, reliable  Tetanus prophylaxis is administered to all patients
indicator of organ perfusion  Trauma patients particularly (a) those with multiple
o Adequate urine output is 0.5 ml/kg/hr in fractures of the pelvis and lower extremities, (b) those
an adult, 1 ml/kg/hr in a child, and 2 with coma or spinal cord injury, and (c) those requiring
ml/kg/hr in an infant <1 year of age ligation of large veins in the abdomen and lower
o Fluid resuscitation begins with a 2L (adult) or extremitiesare at risk for venous thromboembolism
20 ml/kg (child) IV bolus of isotonic and its associated complications
crystalloid, typically Ringer’s lactate o Low molecular weight heparin is initiated as
o For persistent hypotension, this is repeated soon as bleeding has been controlled and
once in adult and twice in a child before RBCs there is no intracranial pathology
are administered o In high-risk patients, removable inferior
 Based on the initial response to fluid resuscitation, vena caval filters should be considered if
hypovolemic injured patients can be separated into there are contraindications to administration
three broad categories: responders, transient responders, of low molecular weight heparin
and nonresponders o Pulsatile compression stockings or
o Responders:Individuals who are stable or sequential compression devices are used
have a good response to the initial fluid routinely unless there is a fracture
therapy as evidenced by normalization of vital  Another prophylactic measure is thermal protectionby
signs, mental status, and urine output are maintaining a comfortable ambient temperature,
unlikely to have significant ongoing covering stabilized patients with warm blankets, and
hemorrhage, and further diagnostic evaluation administering warmed IV fluids and blood products.
for occult injuries can proceed in an orderly o Hemorrhagic shock impairs perfusion and
fashion (Secondary survey) metabolic activity throughout the body, with
o Transient Responders: Those who respond resultant decrease in heat production and
initially to volume loading by an increase in body temperature
blood pressure only to then hemodynamically
deteriorate once more
TOPNOTCH MEDICAL BOARD PREP SURGERY SUPPLEMENT HANDOUT Page 23 of 83
For inquiries visit www.topnotchboardprep.com.ph or email us at topnotchmedicalboardprep@gmail.com
TOPNOTCH MEDICAL BOARD PREP SURGERY SUPPLEMENT HANDOUT - Jules Lopez,MD-MBA,Teddy Carpio,MD-MBA
For inquiries visit www.topnotchboardprep.com.ph or email us at topnotchmedicalboardprep@gmail.com
o Hypothermia causes coagulopathy and Figure 19.2. Laryngeal Trauma Management Protocol
myocardial irritability
 PRBC transfusion should occur once the patient's
hemoglobin level is <7 g/dl, in the acute phase of
resuscitation the endpoint is 10 g/dl
 FFP is transfused to keep theINR <1.5 and PTT <45 sec
 Target of 100,000/l is the target platelet count with
massive transfusion

1. Neck
 Divided into three distinct zones that is important in the
management of neck injuries (Refer to Figure 18 )

Figure 18. For the purpose of evaluating penetrating injuries, the neck is
divided into three zones. Zone I is up to the level of the cricoid and is also
known as the thoracic outlet.Zone II is located between the cricoid
cartilage and the angle of the mandible. Zone III is above the angle of the
mandible. 

2. Abdomen
 Diagnostic approach differs for penetrating trauma
(i.e. gun shot/stab wound) and blunt abdominal trauma
 Management algorithm for penetrating abdominal
injury patients is primarily based on the anatomic
location of injury (Refer to Figure 18)
 As a rule, minimal evaluation is required before
 Imaging options include CT scan or five plain laparotomy for abdominal gunshot or shotgun wounds
radiograph views of the cervical spine: lateral view because over 90% of patients have significant internal
with visualization of C7-T1, anteroposterior view, injuries EXCEPT those isolated in the liver by CT scan;
transoral odontoid views, and bilateral oblique views in hemodynamically stable patients where
nonoperative observation may be considered
 Identification of penetrating injuries to the neck with
exsanguination, expanding hematomas, and airway  Abdominal stab wounds are less likely to injure intra-
obstruction is a priority during the primary survey abdominal organs and thus, diagnostic evaluation can
be afforded
 Management algorithm for penetrating neck injury
patients is based on the presenting symptoms and
Figure 20. Algorithm for the evaluation of penetrating abdominal
anatomic location of injury (Refer to Figure 19) injuries. AASW = anterior abdominal stab wound (from costal margin to
 All blunt trauma patients should be assumed to have inguinal ligament and bilateral MAL); CT = computed tomography; DPL =
cervical spine injuries until proven otherwise diagnostic peritoneal lavage; GSW = gunshot wound; LWE = local wound
exploration; RUQ = right upper quadrant; SW = stab wound.
Figure 19.1. Algorithm for the selective management of penetrating neck
injuries. CT = computed tomography; CTA = computed tomographic
angiography; GSW = gunshot wound; IR Embo = interventional radiology
embolization

 Anterior abdominal stab wounds (AASW) should be


explored under local anesthesia in the ED to determine
if the fascia has been violated
o Injuries that do not penetrate the peritoneal
cavity do not require further evaluation, and
the patient is discharged from the ED
o Patients with fascial penetration must be
further evaluated for intra-abdominal injury,
because there is up to a 50% chance of
requiring laparotomy
o Debate remains over whether the optimal
diagnostic approach is serial examination,
diagnostic peritoneal lavage
(Refer to Figure 20), or CT scanning

TOPNOTCH MEDICAL BOARD PREP SURGERY SUPPLEMENT HANDOUT Page 24 of 83


For inquiries visit www.topnotchboardprep.com.ph or email us at topnotchmedicalboardprep@gmail.com
TOPNOTCH MEDICAL BOARD PREP SURGERY SUPPLEMENT HANDOUT - Jules Lopez,MD-MBA,Teddy Carpio,MD-MBA
For inquiries visit www.topnotchboardprep.com.ph or email us at topnotchmedicalboardprep@gmail.com
o Values representing positive findings for Figure 22. FAST is used to identify free intraperitoneal fluid in
diagnostic peritoneal lavage are summarized (1) subxiphoid/pericardium, (2) Morison's pouch/hepatorenal recess,
in Table 33 (3) left upper quadrant/perisplenic, and (4) pelvis. Although this method
is sensitive for detecting intraperitoneal fluid of >250 ml, it does not
reliably determine the source of bleeding nor grade solid organ injuries.
Figure 21. Diagnostic peritoneal lavage (DPL) is performed through an
infraumbilical incision unless the patient has a pelvic fracture or is
pregnant. Linea alba is sharply incised, and the catheter is directed into
the pelvis. Abdominal contents (diagnostic peritoneal aspiration) is
considered positive if >10 ml of blood is aspirated. If <10 ml is obtained, a
liter of NSS is instilled. Effluent is withdrawn via siphoning and sent to
the laboratory for analysis

 Patients with fluid on FAST examination, considered a


"positive FAST," who do not have immediate indications
for laparotomy and are hemodynamically stable
undergo CT scanning to quantify their injuries
 Management algorithm for blunt abdominal injury
patients is shown in Figure 23

Figure 23. Algorithm for the initial evaluation of a patient with suspected
blunt abdominal trauma. CT = computed tomography; DPA = diagnostic
peritoneal aspiration; FAST = focused abdominal sonography for
trauma/focused assessment with sonography for trauma; Hct=hematocrit

 QUICK REVIEW a
Table 33. Criteria for positive finding on diagnostic peritoneal lavage.
Between 1,000-10,000/ml, do laparoscopy/thoracoscopy 
 Tachycardia is the earliest sign of ongoing blood loss
Anterior Anterior Abdominal Thoracoabdominal
Abdominal Stab Wound Stab Wound  Adequate urine output is 0.5 ml/kg/hr in an adult, 1
Red blood cell ml/kg/hr in a child, and 2 ml/kg/hr in an infant <1 year
>100,000/ml >10,000/ml of age
(RBC) count
White blood cell  Secondary survey consists of “AMPLE” (Allergies,
>500/ml
(WBC) count Medications, Past illnesses or Pregnancy, Last meal,
Amylase and Events related to the injury)
>19 IU/l
level
Alkaline
>2 IU/l
phosphatase level
Bilirubin
level
>0.01 mg/dl REVIEW QUESTIONS a

1. Which of the following trauma patients with airway


 Blunt abdominal trauma initially is evaluated by FAST compromise and failed endotracheal intubation should
(Refer to Figure 22) exam in major trauma centers undergo emergency tracheostomy (rather than a
 FAST is not 100% sensitive so diagnostic peritoneal cricothyroidotomy)?
aspiration is still advocated in hemodynamically a. 84 y/o male with blunt trauma to the neck
unstable patients without a defined source of blood loss b. 65 y/o female with a stab wound to the
to rule out abdominal hemorrhage submandibular region
c. 16 y/o male with a gun shot wound to the neck
d. 6 y/o female with a crush injury to the face

Answer: D
In patients under the age of 8, cricothyroidotomy is
contraindicated due to the risk of subglottic stenosis, and
tracheostomy should be performed.

2. A patient presents with stable vital signs and respiratory


distress after a stab wound to the chest. Chest tubes are
placed and an air leak is noted. The patient is electively
intubated. The patient arrests after positive pressure
ventilation is started. What is the most likely diagnosis?

TOPNOTCH MEDICAL BOARD PREP SURGERY SUPPLEMENT HANDOUT Page 25 of 83


For inquiries visit www.topnotchboardprep.com.ph or email us at topnotchmedicalboardprep@gmail.com
TOPNOTCH MEDICAL BOARD PREP SURGERY SUPPLEMENT HANDOUT - Jules Lopez,MD-MBA,Teddy Carpio,MD-MBA
For inquiries visit www.topnotchboardprep.com.ph or email us at topnotchmedicalboardprep@gmail.com
a. Unrecognized hemorrhage in the abdomen  Long-term neurologic and visual symptoms are also
b. Tension pneumothorax common and thus, neurologic and ophthalmologic
c. Pericardial tamponade consultation should be done
d. Air embolism
3. Chemical
Answer: D  Less common but usually severe
Air emboli can occur after blunt or penetrating trauma,  Offending agents can be systematically absorbed; may
when air from an injured bronchus enters an adjacent cause specific metabolic derangements
injured pulmonary vein and returns air to the left heart.  Careful removal of toxic substance from patient and
Air accumulation in the left ventricle impedes diastolic irrigation of the affected area with water (~30 mins)
filling, and during systole air is pumped into the EXCEPT in cases of concrete powder or powdered forms
coronary arteries, disrupting coronary perfusion. of lye, which should be swept from the patient instead
Patient should be placed in Trendelenburg’s position to to avoid activation of AlOH with water
trap the air in the apex of the left ventricle. Emergency
thoracotomy is followed by cross clamping (left picture) B. BURN DEPTH 
of the pulmonary hilum on the side of the injury to  Burn wounds are commonly stratified according to
prevent further introduction of air. Air is aspirated from depth as superficial, partial thickness, full thickness, and
the apex of the left ventricle and the aortic root with an fourth degree burns, which affect underlying soft tissue
18-g needle and 50-ml syringe (right picture). Vigorous  They are also described according to zone of tissue
massage is used to force air bubble through the coronary injury (Refer to Table 34)
arteries. If unsuccessful, a tuberculin syringe may be
used to aspirate air from the right coronary artery. Once 1. Superficial (First degree burn)
circulation is restored, patient should be kept in  Painful but DO NOT blister
Trendelenburg’s with the pulmonary hilum clamped 2. Partial thickness (Second degree burn)
until pulmonary venous injury is controlled operatively.  Extremely painful with weeping and blisters
 Classified as either superficial or deep depending on the
depth of dermal involvement
o Superficial: Heals with expectant management
o Deep: Requires excision and skin grafting

3. Full thickness (Third degree burn)


 Painless, hard, and non-blanching

4. Fourth degree burn


 Affects underlying soft tissue

Table 34. Jackson’s three zones of tissue injury following burn


Jackson’s three zones of tissue injury following burn
 Most severely burned area (typically the
3. Which of the following is the expected blood loss in a Zone of center of the wound)
patient with 6 rib fractures? Coagulation  Affected tissue is coagulated and sometimes
a. 240 ml necrotic, and will need excision and grafting
b. 480 ml  Between the first and third zones with local
c. 750 ml response of vasoconstriction and ischemia
d. 1500 ml  It has marginal perfusion and questionable
viability
Zone of Stasis
 Resuscitation and wound care may help
Answer: C
prevent conversion to a deeper burn
For each rib fracture, there is ~100-200 ml of blood  Burn wounds evolve over 48-72 hours after
loss; for tibial fractures, 300-500 ml; for femur fractures, injury
800-1000 ml; and for pelvic fractures, >1000 ml.  Outermost area, usually heals with minimal or
Although no single injury may appear to cause a patient’s Zone of
no scarring
hemodynamic instability, the sum of the injuries may Hyperemia
 There is increased blood flow in this area
result in life-threatening blood loss

BURNS ANATOMY a
LAYERS OF THE SKIN
A. Classification of Burns  Epidermis is the outermost layer of the integument
B. Burn Depth composed of stratified squamous epithelial layer that is
C. Initial Evaluation of Burns devoid of blood vessels, consisting of 4-5 layers:
D. Management of Burns o Stratum Corneum is a superficial stratum later
E. Inhalational Injury consisting of flat, anucleated and keratinized
cells filled with keratin filaments embedded in
a dense matrix of proteins
A. CLASSIFICATION OF BURNS o Stratum Lucidum is only found in regions of
1. Thermal thick stratum corneum of palms and soles;
 Flame: Most common cause for hospital admission; not found in thin skin
highest mortality (due to association with inhalational o Stratum Granulosum is polygonal cells with
injury and/or Carbon Monoxide (CO) poisoning) basophilic keratohyalin granules; 1 layer in
 Contact thin skin while multiple layers in thin skin
 Scald o Stratum Spinosum is a multilaminar layer of
cuboidal-like cells that are bound together by
2. Electrical means of numerous desmosomal junctions
(tonofibrils) and they produce keratin
 Potential for cardiac arrhythmias; do baseline ECG i
o Stratum Basale/germinativum is a
 Compartment syndromes with concurrent
mitotically active, single layer of columnar or
rhabdomyolysis is more common in high-voltage
cuboidal cells attached to the dermis via
injuries; check for neurologic or vascular compromise
TOPNOTCH MEDICAL BOARD PREP SURGERY SUPPLEMENT HANDOUT Page 26 of 83
For inquiries visit www.topnotchboardprep.com.ph or email us at topnotchmedicalboardprep@gmail.com
TOPNOTCH MEDICAL BOARD PREP SURGERY SUPPLEMENT HANDOUT - Jules Lopez,MD-MBA,Teddy Carpio,MD-MBA
For inquiries visit www.topnotchboardprep.com.ph or email us at topnotchmedicalboardprep@gmail.com
hemidesmosome Figure 24. Rule of nines to estimate burn size 
o Mnemonics: “Californians Like Girls in String
Bikinis”
 Dermis is the connective tissue layer below the
epidermis and its basement membrane, consisting of 2
layers:
o Papillary layer appears loose that fills the
hollows at the deep surface of the epidermis
with frequent capillaries
o Reticular layer appears denser and contains
fewer cells
 Hypodermis is a layer of loose vascular connective
tissue infiltrated by adipocytes

4. Diagnosis of Carbon Monoxide and Cyanide poisoning


 Unexpected neurologic symptoms should raise the level
of suspicion for CO poisoning
o Affinity of CO for hemoglobin is 200-250x
more than that of O2, which decreases the
levels of normal oxygenated hemoglobin and
can quickly lead to anoxia and death
o Treatment ||Administration of 100% oxygen
is the gold standard, and reduces the half-life
of CO from 250 mins in room air to 40-60 mins
 Cyanide poisoning is seen in smoke inhalation injury
o May have lactic acidosis or ST elevation
o Cyanide inhibits cytochrome oxidase, which in
turn inhibit cellular oxygenation
C. INITIAL EVALUATION OF BURNS o Treatment ||Consists of sodium thiosulfate,
1. Airway management hydroxocobalamin, and 100% oxygen
 With direct thermal injury to the upper airway and/or
smoke inhalation (perioral burns, signed nasal hairs), D. MANAGEMENT OF BURNS
rapid and severe airway edema is a potentially lethal 1. Referral to a burn center (Refer to Table 35)
threat
Table 35. Guidelines for referral to a burn center
 Anticipating the need for intubation and establishing an
Guidelines for referral to a burn center
early airway is critical 1. Partial thickness burns greater than 10% TBSA
 Signs of impending respiratory compromise: hoarse 2. Burns involving the face, hands, feet, genitalia, perineum, or
voice, wheezing, or stridor major joints
3. Third degree burns in any age group
2. Evaluation of other injuries 4. Electric burns (including lightning injury)
 Burn patients should be first considered 5. Chemical burns
traumapatients (especially when details of the injury 6. Inhalational injury
are unclear), as such, a primary survey should be 7. Patients with complicated preexisting medical disorders
conducted 8. Patients with burns and concomitant trauma in which the burn
is the greatest risk
 An early and comprehensive secondary survey must
9. Burned children in hospitals without qualified personnel for
also be performed in all burn patients the care of children
 Urgent radiology studies (i.e. CXR) should be performed 10. Burn injury in patients who will require special social,
in the ER, but non urgent skeletal evaluation (i.e. emotional, or rehabilitative intervention
extremity X-rays) can be done later to avoid
hypothermia and delays in burn resuscitation 2. Resuscitation
 Rationale: Burn (and/or inhalational injury) drives
3. Estimation of burn size inflammatory response that leads to capillary leak
 Most burn resuscitation formulas estimate fluid  As the plasma leaks into the extravascular space,
requirements using the burn size as %Total Body crystalloid administration maintains the
Surface Area (TBSA) intravascular volume
 “Rule of nines” is a crude but quick and effective  Therefore, if a patient receives large fluid bolus in a
method of estimating burn size (Refer to Figure 24 ) prehospital setting or ER, that fluid has likely
 Thorough cleaning of soot and debris is mandatory to leaked into the interstitium and the patient will still
avoid confusing areas of soiling with burns require ongoing burn resuscitation
 Superficial (first degree) burns SHOULD NOT be  Several formulas are available to compute for the
included when calculating the %TBSA total fluid requirement but among the most widely
used one is the Parkland formula(Refer to Table
36 )
Table 36. Parkland formula 
Parkland formula
Total fluid requirement* = 4 mg/kg per %TBSA burn
½ volume during first 8 hours ½ during next 16 hours post-
post-injury injury
*Use of lactated ringer’s solution

TOPNOTCH MEDICAL BOARD PREP SURGERY SUPPLEMENT HANDOUT Page 27 of 83


For inquiries visit www.topnotchboardprep.com.ph or email us at topnotchmedicalboardprep@gmail.com
TOPNOTCH MEDICAL BOARD PREP SURGERY SUPPLEMENT HANDOUT - Jules Lopez,MD-MBA,Teddy Carpio,MD-MBA
For inquiries visit www.topnotchboardprep.com.ph or email us at topnotchmedicalboardprep@gmail.com
 Continuation of fluid volumes should depend on o Warning signs include paresthesia, pain,
the time since injury, UO, and MAP decreased capillary refill, and progression to
 As the leak closes, patient will require less volume loss of distal pulses
to maintain the UO and BP
o Target MAP: 60 mmHg to ensure optimal 5. Nutrition of burn patients
end-organ perfusion  Burn injury causes a hypermetabolic response raising
o Target UO:30 cc/h in adults and 1 to 1.5 baseline metabolic rates by as much as 200%, leading to
cc/kg/hr in pediatric patients catabolism of muscle proteins and decreased lean
 Maintenance IV fluid with glucose body mass that delay functional recovery
supplementation in addition to the calculated  Early enteral feeding for patients help prevent loss of
resuscitation fluid with LR is given in children lean body mass, slow the hypermetabolic response, and
under 20 kg result in a more efficient protein metabolism
o They do not have sufficient glycogen  If enteral feeds are started within the first few hours
stores to maintain an adequate glucose after admission, gastric ileus can often be avoided
level in response to the inflammation.
 Blood transfusions be used only when there is an 6. Surgery
apparent physiologic need  Escharotomies are rarely needed within the first 8
hours following injury and SHOULD NOT be performed
unless indicated because of the aesthetic sequelae
3. Treatment of burn wound  Burn excision and wound coverage should ideally
 Patients with acute burn injuries should NEVER receive start within the first several days, and in larger burns,
prophylactic oral/IV antibiotics serial excisions can be performed as the patient’s
 This intervention has been clearly demonstrated to condition allows
promote development of fungal infections and resistant  Excision is performed with repeated tangential slices
organisms until only non burned tissue remains
 Silver sulfadiazine: most widely used  It is appropriate to leave healthy dermis, which will
o Wide range of anti-microbial activity, appear white with punctate areas of bleeding
primarily as topical prophylaxis against burn
wound infections rather than treatment of 7. Wound coverage/ Grafts
existing infection  Split thickness sheet autografts make the most durable
o Not significantly absorbed systemically wound coverings
o Side effects ||Neutropenia as a result of  In larger burns, meshing of autografted skin provides a
neutrophil margination due to the larger area of wound coverage, allowing drainage of
inflammatory response to burn injury blood and serous fluid to prevent accumulation under
o Destroy skin grafts and is contraindicated on the skin graft with subsequent graft loss
burns in proximity to newly grafted areas
 Others: Mafenide acetate, Silver nitrate, Bacitracin, 8. Rehabilitation
Neomycin, and Polymyxin B  Should be initiated on admission
 Pain management  Immediate and ongoing physical and occupational
o Important to administer an anxiolytic such as therapy is mandatory to prevent loss of physical
benzodiazepine with the initial narcotics function

4. Complications of burn E. INHALATIONAL INJURY


 Hypothermia is one of the common pre-hospital  Commonly seen in tandem with burn injuries
complications that contributes to resuscitation failure  Drastically increase mortality in burn patients
o Patients should be kept wrapped with clean  Causes injury in 2 ways:
blankets o Direct heat injury to the upper airways
 Ventilator-associated pneumonia, like all critically ill  Leads to maximal edema in the first
patients, is a significant problem in burn patients 24 to 48 hours after injury
o Simple measures such as elevating the head of  Will require short course of
the bed and maintaining excellent oral hygiene endotracheal intubation for airway
and pulmonary toilet are recommended to protection
help decrease the risk of postinjury o Inhalation of combustion products into the
pneumonia lower airways
4. Complications of burn(continuation)  Irritants (combustion products)
 Massive resuscitation of burn patients may lead to an cause direct mucosal injury leading
abdominal compartment syndrome to mucosal sloughing, edema,
o Characterized by increased airway pressures reactive bronchoconstriction, and
with hypoventilation, and decreased urine eventually obstruction of the lower
output and hemodynamic compromise airways
o Treatment ||Decompressive laparotomy is  Physiologic effects include decrease lung compliance,
the standard of care for refractory abdominal increase airway resistance work of breathing, increase
compartment syndrome but carries an overall metabolic demands, and an increase in fluid
especially lethal prognosis in burn patients requirements during resuscitation of patients with burn
o Adjunctive measures such as minimizing fluid, injuries
performing truncal escharotomies, decreasing  Treatment ||Supportive care including aggressive
tidal volumes, and chemical paralysis should pulmonary toilet, routine use of nebulized agents (e.g.
be initiated before resorting to decompressive Salbutamol) and ventilation for ARDS
laparotomy
 Burn patients may be at higher risk for catheter- REVIEW QUESTIONS a
related bloodstream infections
 Full thickness burns with a rigid eschar can form a 1. Which of the following patients should be immediately
tourniquet effect as the edema progresses, leading to referred to a burn center?
compromised venous outflow and eventually arterial a. 20 y/o with a 12% partial thickness burn
inflow, leading to compartment syndrome b. 30 y/o with a major liver injury and a 15%
o Common in circumferential extremity burns partial thickness burn
c. 2% TBSA partial thickness burn to the anterior
leg, crossing the knee
TOPNOTCH MEDICAL BOARD PREP SURGERY SUPPLEMENT HANDOUT Page 28 of 83
For inquiries visit www.topnotchboardprep.com.ph or email us at topnotchmedicalboardprep@gmail.com
TOPNOTCH MEDICAL BOARD PREP SURGERY SUPPLEMENT HANDOUT - Jules Lopez,MD-MBA,Teddy Carpio,MD-MBA
For inquiries visit www.topnotchboardprep.com.ph or email us at topnotchmedicalboardprep@gmail.com
d. 10 y/o with a 7% partial thickness burn Figure 25. Different clinical approaches to the closure and healing of
acute wounds 
Answer: A
All patients with a partial thickness burn >10% TBSA
should be transferred to a burn center. A patient with a
burn and other major trauma can be treated in the
trauma center first. Burns that involve the entire joint
should be transferred to a burn center, but a small burn
to the anterior surface of the knee would not necessarily
mandate transfer. Children should be transferred if there
are no personnel able to care for them, but for a child
with a 7% TBSA burn, this would not be mandatory
(Refer to Table 35)

2. Which of the following is indicated in a 46 y/o patient


with a 22% TBSA partial thickness burn?
a. Prophylactic 1st generation cephalosporin
b. Prophylactic clindamycin
c. Tetanus booster
d. Tetanus toxoid

Answer: C
Patients with acute burn injuries should never receive
prophylactic antibiotics. This intervention promote
development of fungal infections and resistant
organisms and was abandoned in the mid-1980s. A B. NORMAL PHASES OF WOUND HEALING 
tetanus booster should be administered in the ER.  Normal wound healing follows a predictable pattern
that can be divided into three overlapping phases:
3. Formic acid burns are associated with? 1. Hemostasis and inflammation
a. Hemoglobinuria 2. Proliferation
b. Rhabdomyolosis 3. Maturation and remodeling
c. Hypocalcemia
d. Hypokalemia 1. Hemostasis and Inflammation
 Hemostasis precedes and initiates inflammation with
Answer: A the ensuing release of chemotactic factors from wound
The offending agents in chemical burns can be site
systematically absorbed and may causes specific  Cellular infiltration after injury follows a characteristic,
metabolic derangements. Formic acid has been known predetermined sequence
to cause hemolysis and hemoglobinuria. o PMNs are the first infiltrating cells to enter
the wound site, peaking at 24 to 48 hours,
4. The major improvement in burn survival in the 20th stimulated by increased vascular permeability,
century can be attributed to the introduction of which of local prostaglandin release, and the presence
the following therapies? of chemotactic substances
a. Antibiotics o These cells DO NOT play a role in collagen
b. Central venous fluid resuscitation deposition and collagen synthesis
c. Nutritional support
d. Early excision of the burn wound  Macrophages (Refer to Figure 26)
o Recognized to be essential in successful
Answer: D wound healing
o Achieve significant numbers by 48 to 96
hours post injury and remain present until
WOUND HEALING wound healing is complete
o Participate in wound debridement via
A. Classification of Wound Healing phagocytosis
B. Normal Phases of Wound Healing o Contribute to microbial stasis via oxygen
C. Classification of Wounds radical and nitric oxide synthesis
o Activation and recruitment of other cells via
mediators as well as directly by cell-cell
interaction and intercellular adhesion
A. CLASSIFICATION OF WOUND HEALING
molecules
 Surgical wounds can heal in several ways
(Refer to Figure 25 )
 T lymphocytes
o Primary intention: an incised wound that is
o Less numerous than macrophages
clean and closed by sutures
o Peak at about 1 week post injury and truly
o Secondary intention: Because of bacterial
bridge the transition from the inflammatory to
contamination or tissue loss, a wound will be
the proliferative stage of wound healing
left open to heal by granulation tissue
o Role is not fully defined
formation and contraction
o Theory is that they play an active role in
o Tertiary intention or delayed primary
modulation of the wound environment
closure: represents a combination of the first
o Exert a downregulating effect on fibroblast
two, consisting of the placement of sutures,
collagen synthesis by cell-associated
allowing the wound to stay open for a few
interferon-gamma, TNF alpha, and IL1
days, and the subsequent closure of the
sutures

TOPNOTCH MEDICAL BOARD PREP SURGERY SUPPLEMENT HANDOUT Page 29 of 83


For inquiries visit www.topnotchboardprep.com.ph or email us at topnotchmedicalboardprep@gmail.com
TOPNOTCH MEDICAL BOARD PREP SURGERY SUPPLEMENT HANDOUT - Jules Lopez,MD-MBA,Teddy Carpio,MD-MBA
For inquiries visit www.topnotchboardprep.com.ph or email us at topnotchmedicalboardprep@gmail.com
 Characterized primarily by proliferation and
Figure 26. Phases of wound healing migration of epithelial cells adjacent to the wound
 Process begin within day 1 of injury and is seen as
thickening of epidermis at the wound edge
 Re-epithelialization is complete in less than 48 hours
in the case of approximated incised wounds, but may
take longer in case of larger wounds, in which there is a
significant epidermal/dermal defect
 Mediated by a combination of a loss of contact
inhibition, exposure to constituents of the extracellular
matrix, particularly fibronectin, and cytokines produced
by immune mononuclear cells

5. Wound Contraction
 All wounds undergo some degree of contraction
 Starts almost immediately after injury despite the
absence of myofibroblasts
 For wounds that do not have surgically approximated
edges, the area of the wound will be decreased by this
action (healing by secondary intention), the shortening
of the scar itself results in contracture
 Myofibroblast has been postulated as being the major
cell responsible for contraction, and it differs from the
2. Proliferation
normal fibroblast in that it possesses a cytoskeletal
 Roughly spans day 4 through 12
structure
 Phase where tissue continuity is re-established
 Fibroblasts and endothelial cells are the last cell
C. CLASSIFICATION OF WOUNDS
populations to infiltrate the healing wound
1. Acute
 Strongest chemotactic factor for fibroblasts is PDGF
 Heal in a predictable manner and time frame
 Upon entering the wound environment, recruited
 Process occurs with few complications and the end
fibroblasts first need to proliferate, and then become
result is a well-healed wound
activated, to carry out their primary function of matrix
synthesis remodeling  Normal process of wound healing is characterized by a
 Fibroblasts from wounds synthesize more collagen, constant and continual increase that reaches a plateau
proliferate less, and actively carry out matrix at some point post injury
contraction  Wounds with delayed healing are characterized by
o Type I collagen is the major component of decreased wound breaking strength in comparison to
extracellular matrix in skin wounds that heal at a normal rate, however, they
o Type III, which is also normally present in eventually achieve the same integrity and strength as
skin, becomes more prominent and important wounds that heal normally
during the repair process  Delayed healing is caused by conditions such as
 Endothelial cells also proliferate extensively during this nutritional deficiencies, infections, or severe trauma
phase of healing, participating in angiogenesis, under which reverts to normal with correction of the
the influence of cytokines and growth factors such as underlying pathophysiology (Refer to Figure 27)
TNF-alpha, TGF-beta, and VEGF  Impaired healing is characterized by a failure to
 Macrophages represent a major source of VEGF achieve mechanical strength equivalent to normally
healed wounds
3. Maturation and Remodeling  Patients with compromised immune system (diabetics,
 Begins during the fibroplastic phase chronic steroid usage, tissues damaged by
 Characterized by a reorganization of previously radiotherapy) are prone to impaired healing
synthesized collagen (Refer to Table 37)
 Collagen is broken down by matrix metalloproteases,
and the net wound collagen content is the result of a Figure 27. The acquisition of wound mechanical strength over time in
normal, delayed, and impaired healing
balance between collagenolysis and collagen synthesis
 There is a net shift toward collagen synthesis and
eventually the re-establishment of extracellular matrix
composed of a relatively acellular collagen-rich scar
 Wound strength and mechanical integrity in the fresh
wound are determined by both the quantity and
quality of the newly deposited collagen
 The deposition of matrix at the wound site follows a
characteristic pattern: fibronectin and collagen type III
constitute the early matrix scaffolding,
Glycosaminoglycans and proteoglycans represent the
next significant matrix components, and collagen type I
is the final matrix
 By several weeks post injury, the amount of collagen in
the wound reaches a plateau, but the tensile strength
continues to increase for several more months
 Scar remodeling continues for 6 to 12 months post
injury, gradually resulting in a mature, avascular, and
acellular scar
 Mechanical strength of the scar never achieves that
of the uninjured tissue

4. Epithelialization
 While tissue integrity and strength are being re-
established, the external barrier must also be restored
TOPNOTCH MEDICAL BOARD PREP SURGERY SUPPLEMENT HANDOUT Page 30 of 83
For inquiries visit www.topnotchboardprep.com.ph or email us at topnotchmedicalboardprep@gmail.com
TOPNOTCH MEDICAL BOARD PREP SURGERY SUPPLEMENT HANDOUT - Jules Lopez,MD-MBA,Teddy Carpio,MD-MBA
For inquiries visit www.topnotchboardprep.com.ph or email us at topnotchmedicalboardprep@gmail.com
formation and fibril cross-linking result in decreased
2. Chronic collagen solubility, increased strength and increased
 Defined as wounds that have failed to proceed through resistance to enzymatic degradation of the collagen
the orderly process that produces satisfactory anatomic matrix. Scar remodeling continues for many months (6-
and functional integrity or that have proceeded through 12) post-injury, gradually resulting in a mature,
the repair process without producing an adequate avascular and acellular scar. The mechanical strength of
anatomic and functional result the scar never achieves that of the uninjured tissue.
 Wounds that have NOT healed in 3 months
4. Which layer of the intestine has the greatest tensile
Table 37. Factors affecting wound healing strength (ability to hold sutures)?
Factors affecting wound healing
Systemic a. serosa
Age b. muscularis
Nutrition c. submucosa
Trauma d. mucosa
Metabolic diseases
Immunosuppression
Answer: C
Connective tissue disorders
The submucosa is the layer that imparts the
Smoking
Local greatest tensile strength and gretest suture-holding
Mechanical injury capacity, a characteristic that should be kept in
Infection mind during surgical repair of GI tract. Additionally,
Edema serosal healing is essential for quickly achieving a
Ischemic/necrotic tissue watertight seal from the luminal side of the bowel.
Topical agents The importance of the serosa is underscored by the
Ionizing radiation significantly higher rates of anastomotic failure
Low oxygen tension observed clinically in segments of bowel that are
Foreign bodies extraperitoneal and lack serosa (ex. Esophagus and
rectum)

 QUICK REVIEW a 5. A 20 year old male presents to the ER with large


contaminated laceration received during a touch
 Normal wound healing follows a predictable pattern that football game. It has been irrigated with normal saline
can be divided into three overlapping and subsequently debrided. Which suture should be
phases:Hemostasis and inflammation, Proliferation, used to close the subcutaneous layer?
and Maturation and remodeling
 PMNs are the first infiltrating cells to enter the wound a. biologic absorbable monofilament (plain gut)
site, peaking at 24 to 48 hours b. synthetic absorbable monofilament
 Myofibroblast has been postulated as being the major c. absorbable braided
cell responsible for contraction d. none of the above

Answer: C
REVIEW QUESTIONS a In general, the smallest suture required to hold the
various layers of the wound in approximation
1. The peak number of fibroblasts in a healing wound should be selected in order to minimize suture-
occurs? related inflammation. Nonabsorbable or slowly
a. 2 days post injury absorbing monofilament sutures are most suitable
b. 6 days post injury for approximating deep fascial layers, particularly
c. 15 days post injury in the abdominal wall. Subcutaneous tissues should
d. 60 days post injury be closed with braided absorbable sutures, with
care to avoid placement of sutures in fat. Although
Answer: B traditional teaching in wound closure emphasized
See Figure 26 multiple-layer closures, additional layers of suture
closure are associated with increased risk of wound
2. The first cells to migrate into a wound are: infection, especially when placed in fat. Drains may
a. Macrophages be placed in areas at risk of forming fluid
b. T Lymphocytes collections.
c. PMNs
d. Fibroblasts

Answer: C ORGAN SYSTEM PATHOLOGIES


PMNs are the 1st infiltrating cells to enter the wound site,
peaking at 24-48 hours. Increased vascular permeability, SKIN AND SOFT TISSUES
local prostaglandin release and the presence of
chemotactic substances, such as complement factors, IL- A. Anatomy and Physiology of the Skin
1, TNF-alpha, TGF beta, platelet factor 4, or bacterial B. Injuries to the Skin and subcutaneous
products, all stimulate neutrophil migration. C. Infections of the skin and the subcutaneous
D. Inflammatory diseases of the skin and
3. The tensile strength of a completely healed wound subcutaneous
approaches the strength of uninjured tissue? E. Benign tumors of the skin and subcutaneous
a. 2 weeks after injury F. Malignant tumors of the skin
b. 3 months after injury G. Syndromic skin malignancies
c. 12 months after injury
d. NEVER

Answer: D
By several weeks postinjury, the amount of collagen in
the wound reaches a plateau, but the tensile strength
continues to increase for several more months. Fibril
TOPNOTCH MEDICAL BOARD PREP SURGERY SUPPLEMENT HANDOUT Page 31 of 83
For inquiries visit www.topnotchboardprep.com.ph or email us at topnotchmedicalboardprep@gmail.com
TOPNOTCH MEDICAL BOARD PREP SURGERY SUPPLEMENT HANDOUT - Jules Lopez,MD-MBA,Teddy Carpio,MD-MBA
For inquiries visit www.topnotchboardprep.com.ph or email us at topnotchmedicalboardprep@gmail.com

B. INJURIES TO THE SKIN AND SUBCUTANEOUS


A. ANATOMY AND PHYSIOLOGY OF THE SKIN
a. Exposure to Caustic substances
 QUICK REVIEW a Table 38. Difference between acidic and alkali injury 
Layers of the skin – p. 26 Acidic Alkali
Phases of wound healing – p. 29 Coagulative necrosis – can Liquefactive necrosis –
damage nerves, blood vessels causes fat saponification that
and tendons but is less facilitates tissue penetration
 Epidermis damaging compared to alkali and increases tissue damage
o Keratinocyte transit time (basal layer to injury  producing a longer more
shedding) is approximately 40 to 56 days. sustained injury compared to
acidic burns
o Melanocytes
 Derived from precursor cells of the neural Tx: copious irrigation with Tx: continuous irrigation with
crest/neuroectodermal in origin either saline or water for 30 water for 2 hours or until
 Produce melanin from tyrosine and cysteine minutes symptomatic relief is
 Despite differences in skin tone, the density achieved
of melanocytes is constant among
individuals. It is the rate of melanin - Intravenous fluid (IVF) extravasation: leakage of
production, transfer to keratinocytes, and injectable fluids into the interstitial space
melanosome degradation that determine o Is considered a chemical burn
the degree of skin pigmentation o Produces chemical toxicity, osmotic toxicity and
 Cutaneous melanocytes play a critical role in pressure effects in a closed environment.
neutralizing the sun's harmful rays. o Culprits:
 UV-induced damage affects the function of  Cationic substances: K, Ca and bicarbonate
tumor suppressor genes, directly causes cell  Osmotically active agents: TPN, hypertonic
death, and facilitates neoplastic dextrose solution
transformation.  Antibiotics
 Cytotoxic drugs / chemotherapeutic drugs –
  QUICK REVIEW a most common cause of extravasation in adults
What factors increase melanin production? o Most common site of extravasation in adults:
1. UV radiation dorsum of the hand
2. Estrogen o Most common cause of extravasation in infants
3. Adrenocorticotropic hormone causing necrosis: high concentration dextrose, Ca,
4. Melanocyte-stimulating hormone bicarbonate and TPN

b. Thermal injuries – hypothermic vs hyperthermic


o Keratinocytes injuries
 Primarily found in the spindle layer
 Contains intermediate filaments (keratin)  QUICK REVIEW a
provides flexible scaffolding  resist external Jackson’s 3 zones of tissue injury for hyperthermic
stress injuries – p. 26
 Point mutations cause blistering diseases, such
as epidermolysis bullosa, associated with
spontaneous release of dermal-epidermal Hypothermic injuries
attachments. o Severe hypothermia primarily exerts its damaging
o Langerhans’ cells (not Langhan’s cells!)  effect by causing direct cellular injury to bv walls
 skin's macrophages; from the bone marrow and microvascular thrombosis.
 expresses class II major histocompatibility o skin's tensile strength decreases by 20% in a cold
antigens  antigen-presenting capabilities. environment [12°C, (53.6°F)].
 Functions: o Trench foot: reactive hyperthermia with blistering
1. rejection of foreign bodies as a result of prolonged exposure to ice-cold water
2. immunosurveillance against viral infections after rapidly bringing it back to normal
3. immunosurveillance against neoplasms of temperature
the skin
 Dermis c. Pressure injury
o Collagen (main functional component of the dermis) - 1 hour of 60 mmHg pressure  can lead to
comprises 70% of its dry weight histologically identifiable venous thrombosis, muscle
o Skin is primarily comprised of type I collagen degeneration, and tissue necrosis
o Fetal dermis is primarily comprised of type III - Pressures:
collagen (reticulin fibers)  provides tensile strength  Normal arteriole: 32 mmHg
(property of the skin that resists stretching) to both  Normal capillary: 20 mmHg
dermis and epidermis  Normal venule: 12 mmHg
 Cutaneous Adnexal Structures  Sitting: 300 mmHg
1. Eccrine glands: sweat-producing glands located over  Sacral pressure at hospital mattress bed: 150
the entire body but are concentrated on the palms, mmHg
soles, axillae, and forehead - Muscle tissue is more sensitive to ischemia than
2. Apocrine glands: Pheromone producing glands skin. Implication: necrosis usually extends to a deeper
primarily found in the axillae and anogenital region. area than that apparent on superficial inspection
It is these structures that predispose both regions to - Treatment: relief of pressure, wound care, systemic
suppurative hidradenitis enhancement (nutritional optimization) and surgical
3. hair follicles: contains a reservoir of pluripotential management (debridement of all necrotic tissue
stem cells critical in epidermal reproductivity followed by irrigation; if shallow ulcer  close by
secondary intention; if deeper ulcer  require surgical
debridement and coverage)

d. Radiation exposure
TOPNOTCH MEDICAL BOARD PREP SURGERY SUPPLEMENT HANDOUT Page 32 of 83
For inquiries visit www.topnotchboardprep.com.ph or email us at topnotchmedicalboardprep@gmail.com
TOPNOTCH MEDICAL BOARD PREP SURGERY SUPPLEMENT HANDOUT - Jules Lopez,MD-MBA,Teddy Carpio,MD-MBA
For inquiries visit www.topnotchboardprep.com.ph or email us at topnotchmedicalboardprep@gmail.com
- Solar or UV radiation: most common form of radiation (pathognomonic: (+) sulfur granules within
exposure purulent specimen).
- Melanin: most important protective factor from UV - Usual site: face or head (60%)
related damage - Risk factors: tooth extraction, odontogenic infection, or
- UV spectrum: facial trauma.
 UVA (400 to 315 nm): majority of solar radiation - Tx: Penicillin and sulfonamides; surgery for deep seated
that reaches the Earth infections.
 UVB (315 to 290 nm): less than 5% of all solar UV
radiation; responsible for acute sunburn and e. Viral infections – HPV
chronic skin damage leading to malignant - Warts are epidermal growths resulting from human
degeneration (known risk factor in the papillomavirus (HPV) infection.
development of melanoma.)
 UVC (290 to 200 nm): absorbed by the ozone layer Table 40: Comparison of HPV infections
Common Plantar Flat Venereal warts
wart warts warts (condylomata
C. INFECTIONS OF THE SKIN AND THE SUBCUTANEOUS (verruca (verruca (verruca acuminata)
a. Cellulitis, Folliculitis, furuncles & carbuncles vulgaris) plantaris) plana)
Table 39: Comparison of skin infections  -fingers -soles and - the face, - the vulva, anus, and
Cellulitis Folliculitis Furuncles Carbuncles and toes palms legs, and scrotum (relatively
- Superficial, -infection of -begins as folliculitis - deep seated -described -resemble hands moist areas)
spreading the hair but progresses as a infections as rough a common - slightly - STD
infection of the follicle fluctuant nodule that result in and callus raised - HPV 6 & 11
skin and subQ -usual cause: (boil/furuncle) multiple bulbous and flat. -buschke
-usual cause: Staphylococc -tx: warm water draining Lowenstein tumor:
Grp. A strep & us, followed hastens liquefaction sinuses Extensive growths,
S. aureus by G(-) & spontaneous -tx: incision facilitated by
-tx for organisms rupture; incision and drainage concomitant HIV
uncomplicated -tx: adequate and drainage if infection
cellulitis with hygiene necessary
no morbidities:
outpatient oral
antibiotics - histopathology: hyperkeratosis (hypertrophy of the
horny layer), acanthosis (hypertrophy of the
b. Necrotizing soft tissue infections spinous layer), and papillomatosis
- Basis of classification: - Tx: formalin, podophyllum, and phenol-nitric acid;
 the tissue plane affected and extent of invasion Curettage with electrodesiccation also can be used for
 necrotizing fasciitis: rapid, extensive scattered lesions
infection of the fascia deep to the adipose - HPV types 5, 8, and 10: (+) association with
tissue squamous cell carcinoma:
 necrotizing myositis: primarily involves the  lesions that grow rapidly, atypically, or ulcerate
muscles but typically spreads to adjacent soft should be biopsied
tissues
D. INFLAMMATORY DISEASES OF THE SKIN AND
 the anatomic site
SUBCUTANEOUS
 Most common sites: the external genitalia,
perineum, or abdominal wall (Fournier
a. Pyoderma gangrenosum
gangrene)
- Main characteristic: rapidly enlarging, destructive,
 the causative pathogen
cutaneous necrotic lesion with undermined border and
 polymicrobial more common than single
surrounding erythema
organism infections
- (+) associated with a systemic disease 50% of the time
 most common causative organisms: group A
(inflammatory bowel disease, rheumatoid arthritis,
streptococci, enterococci, coagulase-negative
hematologic malignancy, and monoclonal
staphylococci, S. aureus, S. epidermidis, and
immunoglobulin A gammapathy)
Clostridium species
- Tx: Recognition of the underlying disease, systemic
 others (Gram negatives): Escherichia coli,
steroids or cyclosporine & chemotherapy with
Enterobacter, Pseudomonas species, Proteus
aggressive wound care and skin graft coverage
species, Serratia species, and bacteroides
- risk factors: diabetes mellitus, malnutrition, obesity,
b. SSS vs TEN
chronic alcoholism, peripheral vascular disease, CLL, Table 41: comparison between SSSS & TEN 
steroid use, renal failure, cirrhosis, and autoimmune SSSS TEN
deficiency syndrome Difference:
- tx: prompt recognition, broad-spectrum IV antibiotics, -caused by an exotoxin (TSS -caused by an immune response to
aggressive surgical debridement (should be extensive- toxin-1)produced during staph certain drugs (sulfonamides,
including all skin, subcutaneous tissue, and muscle, until infection of the nasopharynx or phenytoin, barbiturates,
there is no further evidence of infected tissue followed middle ear  cytokine release tetracycline)
by as needed debridement), and aggressive fluid throughout the body causing -more than 30% TBSA involved
replacement (needed to offset acute renal failure from diffuse injury and systemic (if less than 10% TBSA  SJS)
ongoing sepsis) symptoms

-histopath: cleavage plane in the - histopath: structural defect at


c. Hidradenitis suppuritiva granular layer of epidermis dermoepidermal jxn; similar to a
- is a defect of the terminal follicular epithelium  2nd degree burn
leading to apocrine gland blockage  gives rise to
Similarity:
abscess formation in the axillary, inguinal, and
- appearance: skin erythema, bullae formation, wide area of tissue
perianal regions loss
- Tx: warm compresses, antibiotics, and open drainage if -diagnosis: skin biopsy
acute; wide excision with closure using skin graft or treatment: fluid and electrolyte replacement, as well as wound care
local flap placement if chronic similar to burn therapy
- appearance: skin erythema, bullae formation, wide area of tissue
d.
Actinomycosis loss
- is a granulomatous suppurative bacterial disease & -diagnosis: skin biopsy
deep cutaneous infections that present as nodules and treatment: fluid and electrolyte replacement, as well as wound care
similar to burn therapy
spread to form draining tracts caused by Actinomyces
TOPNOTCH MEDICAL BOARD PREP SURGERY SUPPLEMENT HANDOUT Page 33 of 83
For inquiries visit www.topnotchboardprep.com.ph or email us at topnotchmedicalboardprep@gmail.com
TOPNOTCH MEDICAL BOARD PREP SURGERY SUPPLEMENT HANDOUT - Jules Lopez,MD-MBA,Teddy Carpio,MD-MBA
For inquiries visit www.topnotchboardprep.com.ph or email us at topnotchmedicalboardprep@gmail.com
Table 44: Comparison between hemangioma, vascular malformation,
E. BENIGN TUMORS OF THE SKIN AND SUBCUTANEOUS port wine stain and glomus tumors.
Hemangioma Vascular Capillary Glomus
a. Cutaneous cysts: Epidermal, dermoid or malformat malformat tumors
trichelemmal ion ion (port
Table 42: Comparison between epidermal, dermoid & trichilemmal cyst: wine
stain)
Epidermal cyst Dermoid cyst Trichilemmal (pilar) -most common - vascular -flat, dull- -benign
cyst cutaneous lesion of malformati red lesion -located at
Difference: infancy ons are a often the
- most common - congenital lesions that -2nd most common -benign lesion that result of located on extremities
- single, firm result when epithelium - when ruptured: present soon after structural the - arise from
nodule is trapped during fetal produce an intense birth (not at birth!) abnormalit trigeminal dermal
midline closure characteristic odor -histopath: mitotically ies formed (CN V) neuromyoa
-location: - most common active endothelial cells during distributio rterial
anywhere in the location: eyebrow -location: scalp (of surrounding several, fetal n on the apparatus
body females) confluent blood-filled developm face, trunk, (glomus
spaces ent or bodies).
-histopath: -histopath: -enlarge at 1st year of -histopath: extremities - usually
mature demonstrates -histopath: no life  90% eventually enlarged ; presents
epidermis squamous epithelium, granular layer; involute vascular associated with severe
complete with eccrine glands, and -tx: if it interferes with spaces with pain, point
granular layer pilosebaceous units. In airway, vision, and lined by sturge- tenderness
addition, these feeding or results to nonprolifer weber and cold
particular cysts may systemic problems ating symdrome sensitivity
develop bone, tooth, or (thrombocytopenia or endotheliu s -tx: tumor
nerve tissue on high-output cardiac m. (leptomeni excision
occasion failure)  resection - grow in ngeal
and proportion angiomato
prednisone/interferon to the body sis,
Similiarity: alpha 2 (for rapidly and never epilepsy,
-contain keratin (not sebum) enlarging lesions) involute and
-appear the same clinically (subcutaneous, thin-walled nodule glaucoma)
containing a white, creamy material)
-treatment: excision; incision and drainage if infected; make sure to
remove the cyst wall to prevent recurrence

e. Soft tissue tumors


b. Keratosis – seborrheic vs solar Table 45. Comparison of lipoma, dermatofibroma &
achrochordon
Table 43. Comparison between seborrheic keratosis and Lipoma Dermatofibroma Achrochordo
actinic keratosis: n (skin tags)
Seborrheic (or solar) Actinic keratosis - most common - solitary, soft-tissue - fleshy,
keratosis subcutaneous nodules measuring 1 to pedunculated
- considered as a -considered as a neoplasm 2 cm masses
premalignant lesion of SCC premalignant lesion of SCC - soft and fleshy -usual location: legs and -usual
- appearance: light brown or (although at least 25% on palpation flanks location:
yellow with a velvety, greasy spontaneously regress) --usual location: -histopath: preauricular
texture back unencapsulated areas, axillae,
-arise in sun exposed areas -histopath: connective tissue whorls trunk, and
(face, forearms, back of hands) lobulated tumor containing fibroblasts eyelids
-common in old age groups composed of - do biopsy for atypical -tx: “tying-off”
-sudden eruptions are normal fat cells presentation or resection
associated with internal -tx: excision -tx: excision
malignancies
-treatment: topical 5- f. Neural tumors
fluorouracil, surgical excision, - Benign
electrodesiccation, and - Arise from the nerve sheath
dermabrasion
Table 46. Comparison of neurofibroma, neurilemoma &
c. Nevi – acquired vs congenital: both are histologically granular cell tumor
similar. Neurofibroma Neurilemoma Granular cell
tumor
Acquired melanocytic nevi -sporadic, solitary - solitary tumors - solitary lesions
-can be syndromic arising from cells of the skin or,
- Classification is based on different stages of maturation
(von Recklingh - of the peripheral more commonly,
 Junctional: epidermis ausen's disease: nerve sheath the tongue
 Compound: extend partially into dermis café au lait spots,
 Dermal: dermis Lisch nodules,
and an autosomal
Congenital nevi dominant
- Rare (less than 1% of neonates) inheritance)
- Giant congenital lesions (giant hairy nevi): appear in a - with direct
nerve attachment
swim trunk distribution, chest, or back
- may develop into malignant melanoma in 1 to 5% of - histopath: , - histopath: tumor -histopath:
cases proliferation of contains Schwann granular cells
- tx: total excision of nevus perineurial and cells with nuclei derived from
endoneurial packed in Schwann cells
d. Vascular tumors of the skin and subcutaneous fibroblasts with palisading row that often
Schwann cells - tx: resection infiltrate the
embedded in surrounding
collagen striated muscle.
-tx:resection

TOPNOTCH MEDICAL BOARD PREP SURGERY SUPPLEMENT HANDOUT Page 34 of 83


For inquiries visit www.topnotchboardprep.com.ph or email us at topnotchmedicalboardprep@gmail.com
TOPNOTCH MEDICAL BOARD PREP SURGERY SUPPLEMENT HANDOUT - Jules Lopez,MD-MBA,Teddy Carpio,MD-MBA
For inquiries visit www.topnotchboardprep.com.ph or email us at topnotchmedicalboardprep@gmail.com
F. Malignant tumors of the skin  Excision with 1 cm margin + histologic
confirmation of tumor free borders
Basal cell carcinoma   If located at cheek, nose, or lip: Mohs' surgery
- most common type of skin cancer.  Regional LN excision is indicated for clinically
- Arises from the pluripotential basal epithelial cells of palpable nodes
epidermis and NOT DERMIS!  If SCC arises from chronic wounds,
- Slow growing and metastasis is rare but are capable lymphadenectomy before development of palpable
of extensive local tissue destruction nodes (prophylactic LN dissection) is indicated
- Subtypes: because it is more aggressive and lymph node
 Nodulocystic/noduloulcerative metastases are observed more frequently
 70% of BCC tumors (most frequent form) - Metastatic disease is a poor prognostic sign (13%
 Waxy and frequently cream survival after 10 years).
colored/translucent;over time, can present as
a rolled, pearly borders surrounding a central  MUST KNOW a
ulcer (rodent ulcer)
 Morpheaphorm Keratoacanthoma, which is characterized by rapid growth,
 flat, plaque-like lesion rolled edges and a crater filled with keratin, can be confused
 most aggressive clinically (due to presence of with SCC or BCC. It spontaneously involutes over time.
type IV collagenase that facilitates local Biopsy lesion to rule out carcinoma.
spread)  early excision
 basosquamous type
 combination of both BCC + SCC Moh’s surgery for BCC and SCC (nice to know!)
 aggressive treated right away! - This precise, specialized surgical technique uses
- Tx: minimal tissue resection and immediate microscopic
 Less than 2mm nodular lesions: curettage, analysis to confirm appropriate resection yet limit
electrodesiccation, or laser vaporization. removal of valuable anatomy.
 If located at cheek, nose, or lip: Mohs' surgery - Done for aesthetic purposes
 Large tumors, those that invade surrounding - uses serial excision in small increments coupled with
structures, & aggressive histologic types immediate microscopic analysis to ensure tumor
(morpheaform, infiltrative, and removal  all specimen margins are evaluated.
basosquamous): surgical excision with 0.5-cm Recurrence and metastases rates are comparable to
to 1-cm margins. those of wide local excision.
- Syndromic skin malignancies associated with BCC:
 basal cell nevus (Gorlin's) syndrome:
 autosomal dominant disorder characterized Malignant Melanoma 
by the growth of hundreds of BCCs during
- Arise from melanocytes
young adulthood.
- Premalignant lesion: dysplastic nevi (vs freckles -
 Palmar and plantar pits: common physical
benign melanocytic neoplasms found on the skin)
finding
- Most common location: skin (>90%); other sites: anus,
 Tx: excision of aggressive and symptomatic
eyes
lesions
- 4%: discovered as metastases without any identifiable
 nevus sebaceus of Jadassohn:
primary site.
 lesion containing several cutaneous tissue
- Suspicious features: pigmented lesion with an
elements that develops during childhood. irregular border, darkening coloration, ulceration,
raised surface and recent changes in nevus
Squamous cell carcinoma  appearance
- Arise from epidermal keratinocytes - Risk factors:
- Less common than BCC  increased sun exposure of fair skinned people to
- Highly invasive and tends to metastasize solar radiation
- Tend to occur in persons with blond hair, light, thin, dry  Familial dysplastic nevus syndrome
and irritated skin.  autosomal dominant disorder
- In situ lesions: Bowen’s disease; if in the penis  - Subtypes:
erythroplasia of Queyrat
- Risk factors: Table 47. comparison of malignant melanoma subtypes:
 Skin lesions: actinic keratosis, atrophic
dermatitis Superficial Nodular Lentigo Acral lentiginous
spreading maligna
 Occupational exposure: arsenics, nitrates and
- most common -15 to 30% -4 to -2 to 8% of
hydrocarbons type (70% of of 15% of melanomas (least
 Syndromic malignancies associated with SCC: melanomas) melanoma melano common)
 epidermolysis bullosus - location: s mas -occurs at palms,
 lupus erythematosus anywhere on the - darker - occur soles, and subungual
 Epidermodysplasia verruciformis skin except the coloration on neck, regions
 rare autosomal recessive disease hands and feet and often face, - Hutchinson's sign:
associated with infection with HPV - flat and raised and presence of
measure 1 to 2 -lack hands of pigmentation in the
 Xeroderma pigmentosum
cm in diameter radial elderly proximal or lateral
 autosomal recessive disease associated at diagnosis growth -best nail folds; diagnostic
with a defect in cellular repair of DNA - Before vertical - progno of subungual
damage. extension, a aggressive sis melanoma
- tumor thickness correlates well with malignant prolonged but same -dark skinned +
behavior. radial growth prognosis acral lentiginous
 more than 4 mm: Tumor recurrence is more phase is with melanoma:
prevalent characteristic of superficial increased risk of
these lesions spreading malignancy
 if 10 mm or more: these lesions usually have
associated metastasis
- Prognostic indicators:
- Burn scars (Marjolin's ulcer), areas of chronic
osteomyelitis, and areas of previous injury  tend to  Location: lesions of the extremities have a better
metastasize early. prognosis than patients with melanomas of the
- Tx: head, neck, or trunk

TOPNOTCH MEDICAL BOARD PREP SURGERY SUPPLEMENT HANDOUT Page 35 of 83


For inquiries visit www.topnotchboardprep.com.ph or email us at topnotchmedicalboardprep@gmail.com
TOPNOTCH MEDICAL BOARD PREP SURGERY SUPPLEMENT HANDOUT - Jules Lopez,MD-MBA,Teddy Carpio,MD-MBA
For inquiries visit www.topnotchboardprep.com.ph or email us at topnotchmedicalboardprep@gmail.com
 (+) ulceration (due to increased angiogenesis):
worse prognosis REVIEW QUESTIONS
 Gender: females have higher survival rates than
men 1. Match the item in the left hand column with the
 Tumor types: appropriate item in the right hand column
 Best: lentigo maligna
 Worse: acral lentiginous a. modulate cold sensation a. Ruffini’s
- Staging from AJCC: breslow and clark level endings
 Breslow thickness: the vertical thickness of the b. modulate sensitivity to warmth b. Krause’ end-
primary tumor (from the granular layer of the bulb
epidermis or base of ulcer to the greatest depth of c. modulate sensation of pressure c. Meissner’s
the tumor); most important prognostic variable corpuscles
predicting survival in those with cutaneous d. modulate tactile sensation d. Pacinian
melanoma; considered to be more precise in corpuscles
predicting biologic behavior e. modulate thermoregulation e. autonomic
 I: 0.75 mm or less nerve endings
 II: 0.76 to 1.5 mm
 III: 1.51 to 4.0 mm Answer: A-b; B-a; C-d; D-c; E-e
 IV: 4.0 mm or more A variety of highly specialized structures are responsible
 Clark level: anatomic depth of invasion for modulating the skin’s various sensory functions. The
 I: superficial to basement membrane (in situ) numbers of these structures vary with the region of the
 II: papillary dermis body. Pacinian corpuscles are found in the
 III: papillary/reticular dermal junction subcutaneous tissue, in the nerves of the palm of the
 IV: reticular dermis hand and the sole of the foot, and in other areas. Each of
 V: subcutaneous fat these corpuscles is attached to and encloses the
- LN status & Metastasis: termination of a single nerve fiber. They are involved in
 Evidence of tumor in regional LNs is a poor the sensation of pressure. Ruffini’s endings are a variety
prognostic sign (automatic stage III) of nerve endings in the subcutaneous tissue of the
 Identification of distant metastasis is the worst fingers and modulate sensitivity to warmth. Krause’s
prognostic sign ( automatic stage IV) end-bulbs are formed by the expansion of the
- Dx: excisional biopsy connective tissue sheath of medullated fibers and are
- Tx: involved in the sensation of cold. Meissner’s corpuscles
 Melanoma in situ/lentigo maligna melanoma in occur in the papillae of the corium of the hands, the feet,
face: 0.5 cm margins the skin of the lips and other areas concerned with
 1mm or less: excision with 1 cm margin tactile sensation. Autonomic fibers that synapse to
 1 to 4 mm: excision with 2 cm margin sweat glands and receptors in the vasculature govern
 More than 4 mm or (+) satellosis: 3-5 cm margin thermoregulation.
 High dose interferon has a role in high risk
melanoma 2. Select the treatment options(s) in the 1st set of choices
 LN dissection: (UPPER CASE) that is/are most appropriate for the
 Sentinel LN biopsy: 1mm or thicker with melanoma case summaries outlined in the 2nd set of
clinically negative nodes or 0.75 mm thick + choices (lower case)
clark level IV or ulcerated
 Radical regional lymphadenectomy: A. Level III superficial spreading melanoma (0.4 mm
clinically (+) nodes with no evidence of distant thick with clinically negative regional lymph nodes
disease on metastatic work up. B. Level IV nodular melanoma (2mm thick) with
satellosis and clinically negative regional lymph
nodes
 QUICK REVIEW a C. Level IV superficial spreading melanoma (1.5 mm
Remember:
thick) with palpable regional lymph nodes
 Moh’s surgery is not appropriate for any type of D. Level IV acral lentiginous melanoma (2 mm thick)
melanoma with clinically negative regional lymph nodes
 If melanoma is 4mm or greater + clinically negative E. Level II lentigo maligna melanoma (0.3 mm)
nodes  perform metastatic work up first

f. Moh’s micrographic surgery


Merkel cell carcinoma g. Wide local excision with 0.5 cm margins
- Primary Neuroendocrine Carcinoma of the Skin h. Wise local excision with 1.0 cm margins
- associated with a synchronous or metasynchronous SCC i. Wide local excision with 2.0 cm margins
25% of the time. j. Wide local excision with 4.0 cm margins
- Tx: wide local resection with 3-cm margins + k. Sentinel lymph node biopsy
Prophylactic regional LN dissection + adjuvant radiation l. Regional lymph node biopsy
therapy are recommended. m. Radical regional lymphadenectomy
- Prognosis: worse than malignant melanoma
Answer: A – c; B – e,f; C – d,h; D – d,f; E – b
Kaposi’s sarcoma Virtually all melanomas are best treated by wide
- rubbery bluish nodules that occur primarily on the excision. The excision margin that minimizes the risk of
extremities (also skin and viscera) local recurrence depends on the thickness of the tumor.
- usually multifocal rather than metastatic. Melanoma in situ and thin lentigo maligna melanomas
- Histopath: capillaries lined by atypical endothelial cells. of the face are treated adequately by margins of 0.5 cm.
- seen in people of Eastern Europe or sub-Saharan Africa, For melanomas less than 1.0 mm thick, 1 cm excision
AIDS or immunosuppression with chemotx margins are appropriate. For intermediate thickness
- locally aggressive but undergo periods of remission melanoma (1-4 mm), a 2 cm margin is sufficient.
Margins of 3-5 cm are generally employed for
melanomas 4 mm in thickness and for those with
Extramammary Paget’s disease associated satellosis. Moh’s chemosurgery is not
- cutaneous lesion that appears as a pruritic red patch appropriate for the treatment of any melanomas. The
that does not resolve indications for elective lymph node dissection remain
- histologically similar to the mammary type. controversial. Sentinel lymph node biopsy is indicated
TOPNOTCH MEDICAL BOARD PREP SURGERY SUPPLEMENT HANDOUT Page 36 of 83
For inquiries visit www.topnotchboardprep.com.ph or email us at topnotchmedicalboardprep@gmail.com
TOPNOTCH MEDICAL BOARD PREP SURGERY SUPPLEMENT HANDOUT - Jules Lopez,MD-MBA,Teddy Carpio,MD-MBA
For inquiries visit www.topnotchboardprep.com.ph or email us at topnotchmedicalboardprep@gmail.com
for aptients with melanoma 1 mm or thicker with
clinically negative nodes. The indication is extended to B. FUNCTIONAL ANATOMY OF THE BREAST
patients with 0.75 mm thick melanomas if they are - It extends from the level of the 2nd or 3rd rib to the
Clark’s level IV or ulcerated. Patients with clinically inframammary fold at the 6th or 7th rib.
positive lymph nodes with no evidence of distant - It extends transversely from the lateral border of the
disease on metastatic workup (CT of chest, abdomen sternum to the anterior axillary line.
and pelvis; MRI of brain; PET) should undergo radical - The deep or posterior surface of the breast rests on the
regional lymphadenectomy. Patients with primary fascia of the pectoralis major, serratus anterior, external
tumors 4 mm or greater with clinically negative nodes oblique abdominal muscles, & the upper extent of the
should undergo metastatic workup before undergoing rectus sheath.
sentinel node biopsy and wide local excision. - retromammary bursa: located at posterior aspect of
the breast between the investing fascia of the breast and
3. With regards to keloids and hypertrophic scars, which of the fascia of the pectoralis major muscles.
the following statements is/are true? - The axillary tail of Spence extends laterally across the
anterior axillary fold.
A. There are no histologic differences between the two - upper outer quadrant: greatest volume; most
B. The differences between hypertrophic scar and common site of breast cancer 
keloid are clinical, not pathologic - Blood supply:
C. Hypertrophic scars outgrow their original borders
D. Hypertrophic scars and keloids have been treated Table 48. Blood supply of the breast
successfully with intralesional injection of steroids Arterial blood supply Venous blood supply
E. Keloids are seen in dark-skinned individuals, - perforating branches of the -perforating branches of the
internal mammary artery internal thoracic vein
whereas hypertrophic scars are seen in fair-skinned
- lateral branches of the -perforating branches of the
individuals posterior intercostal arteries posterior intercostal veins
-branches from axillary artery -tributaries of the axillary vein.
Answer: A,B,D (highest thoracic, lateral - Batson's vertebral venous
Histologically, keloids and hypertrophic scars appear the thoracic, and pectoral branches plexus : possible route for
same. Hypertrophic scars are thick, red, raised scars of the thoracoacromial artery breast cancer metastases to the
that do not outgrow their original borders, whereas vertebrae, skull, pelvic bones,
keloids do. Keloids are dense accumulations of fibrous and central nervous system.
tissue that form at the surface of the skin. The defect
appears to result from a failure in collagen breakdown - Innervation:
rather than an increase in its production. Keloids and  Sensory innervation to breast & anterolateral chest
hypertrophic scars have been successfully treated with wall: Lateral cutaneous branches of the 3rd – 6th
intralesional steroid injectin, radiation, pressure and the intercostal nerves (slips out in between serratus
use of silicone gel sheets. anterior muscles)
 Intercostobrachial nerve: lateral cutaneous
branch of the second intercostal nerve; injury
BREAST to this nerve results to loss of sensation over
the medial aspect of the upper arm.
A. Embryology of the breast  Cutaneous branches from cervical plexus (anterior
B. functional anatomy of the breast branches of the supraclavicular nerve): supply a
C. Physiology of breast limited area of skin over the upper portion of the
D. infectious and inflammatory disorders of the breast breast.
E. common benign disorders and diseases of the - Lymphatics:
breast  6 axillary lymph node groups:
F. breast cancer
Table 49. Location and drainage pattern of breast 
G. special clinical situations
Name location drainage
Lateral medial or upper extremity; receives
(axillary posterior to the 75% drainage of the
vein vein breast; most common site
A. EMBRYOLOGY OF THE BREAST group) – of axillary LN metastasis
- 5th or 6th week of fetal development  mammary ridges level I
(thickened ectoderm): precursors of breast Anterior or lower border of lateral aspect of the breast
 Extends from the base of the forelimb (future pectoral the pectoralis
(external minor muscle
axilla) to the hind limb (future inguinal region)
mammary contiguous with
- Witch’s milk: (+) breast secretions in an infant group) – the lateral
secondary to maternal hormones that crosses the level I thoracic vessels
placenta Posterior posterior wall of lower posterior neck, the
- Anomalies in embryology: of the axilla at the posterior trunk, and the
 Polymastia: accessory breast; can be seen in subscapula lateral border of posterior shoulder
Turner’s syndrome (ovarian agenesis and r (scapular the scapula
dysgenesis)and Fleischer’s syndrome group) – contiguous with
(displacement of the nipples and bilateral renal level I the subscapular
vessels
hypoplasia); can enlarge during pregnancy &
Central embedded in receive lymph drainage both
lactation group – the fat of the from the axillary vein,
 Polyethelia: accessory nipples; maybe associated level II axilla lying external mammary, and
with CVS and urinary tract anomalies immediately scapular groups of lymph
 Amastia: congenital absence of breast due to posterior to the nodes, and directly from the
arrest in mammary ridge development during the pectoralis minor breast
6th week muscle
 Poland’s syndrome: hypoplasia or complete Apical posterior and from all of the other groups
absence of the breast, costal cartilage and rib (subclavicu superior to the of axillary lymph nodes
lar group) upper border of
defects, hypoplasia of the subcutaneous – level III the pectoralis
tissues of the chest wall, and brachysyndactyly minor muscle
 Symmastia: webbing between the breasts across Interpector interposed receive lymph drainage
the midline (no cleavage) al group between the directly from the breast. The
 Inverted nipple: occurs in 4% of infants (Rotter’s pectoralis major lymph fluid that passes

TOPNOTCH MEDICAL BOARD PREP SURGERY SUPPLEMENT HANDOUT Page 37 of 83


For inquiries visit www.topnotchboardprep.com.ph or email us at topnotchmedicalboardprep@gmail.com
TOPNOTCH MEDICAL BOARD PREP SURGERY SUPPLEMENT HANDOUT - Jules Lopez,MD-MBA,Teddy Carpio,MD-MBA
For inquiries visit www.topnotchboardprep.com.ph or email us at topnotchmedicalboardprep@gmail.com
nodes)- and pectoralis through the interpectoral
level II minor muscles group of lymph nodes d. Zuska's disease (recurrent periductal mastitis)
passes directly into the - recurrent retroareolar infections and abscesses.
central and subclavicular - Risk factor: smoking
groups.
- Tx: antibiotics + I&D as necessary.
Figure 28. Axillary lymph node groups 
e. Hidradenitis suppurativa
Reference point: - can also occur in the nipple-areola complex
pectoralis minor - originates within the Montgomery glands or axillary
(not major!) sebaceous glands.
Level I: lateral to - Risk factor: chronic acne
the pectoralis - may mimic Paget's disease of the nipple or invasive
minor muscle (PM) breast cancer.
Level II: deep to
- Tx: Antibiotic + I&D
PM
Level III: medial to
the PM. f. Mondor’s disease
- a benign self limited condition which is a variant of
Arrows indicate the thrombophlebitis that involves the superficial veins of
direction of lymph the anterior chest wall and breast.
flow. - Involved veins: lateral thoracic vein, the
Also seen: axillary thoracoepigastric vein, and, less commonly, the
vein & its major
superficial epigastric vein.
tributaries,
supraclavicular LN - SSx: acute pain in the lateral aspect of the breast or the
anterior chest wall with palpation of a tender, firm cord
C. PHYSIOLOGY OF BREAST along the distribution of the major superficial veins.
- Breast development and function - Tx: anti-inflammatory medications + warm compresses
 Hormonal stimuli: along the symptomatic vein + Restriction of motion of
 Estrogen: ductal development the ipsilateral extremity and shoulder + brassiere
 Progesterone: differentiation of epithelium & support of the breast are important (4 to 6 weeks) or
lobular development excision of vein (if not improving)
 Prolactin: 1o hormonal stimulus for
lactogenesis in late pregnancy & the E. COMMON BENIGN DISORDERS AND DISEASES OF
postpartum period. THE BREAST
- Gynecomastia: enlarged breast in males measuring at a. Fibroadenoma 
least 2 cm in diameter - seen predominantly in younger women aged 15 to 25
- Gynecomastia generally does not predispose the male years
breast to cancer unless syndromic - can be self limiting
- Physiologic gynecomastia occurs due to excess in - if greater than 3cm  consider giant fibroadenoma
circulating estrogens (in relation to circulating - if multiple (more than 5 lesions in 1 breast)
testosterone): considered as abnormal
 neonatal period: action of placental estrogens on - tx: cryoablation, surgical removal or observation
neonatal breast tissues
 adolescence: excess of estradiol relative to b. Cyclical mastalgia and nodularity
testosterone; can be unilateral - associated with premenstrual enlargement of the breast
 senescence: circulating testosterone level falls; - physiologic.
usually bilateral - If Painful nodularity persists for >1 week of the
menstrual cycle  consider a disorder.
D. INFECTIOUS AND INFLAMMATORY DISORDERS OF - bilateral bloody nipple discharge can be seen in
THE BREAST epithelial hyperplasia of pregnancy

a. Breast abscess c. Breast cysts 


- Staphylococcus aureus (more localized & deep) and - occurs when the stroma involutes too quickly, and
Streptococcus (diffuse superficial involvement) species: alveoli remain  forming microcysts & macrocysts
causative organisms - characteristics of benign lesions: sharp, smooth
- SSx: point tenderness, erythema, and hyperthermia. margins, a homogenous interior and posterior
- Risk factor: lactation (because a lactating breast is an enhancement (vs malignancy which will show irregular
excellent culture medium) and jagged margins, heterogenous interior and posterior
- Tx: preoperative UTZ + incision & drainage (if already shawoding)
with suppuration) + local wound care (warm - management: needle biopsy ( 1st line investigation for
compresses &IV antibiotics - penicillins or palpable breast masses)
cephalosporins).  if (+) fluid on aspiration  aspirate to dryness, no
 Remember: Biopsy of the abscess cavity wall is need to do cytologic examination
recommended at the time of I&D  rule out breast  If after aspiration, (+) residual mass  do UTZ
cancer with necrotic tumor. guided needle biopsy
- Chronic breast abscesses: consider acid-fast bacilli,  If blood stained fluid  aspirate 2 mL for cytologic
anaerobic and aerobic bacteria, and fungi. examination, utz imaging and biopsy solid areas
 If fungal. Consider blastomycosis or sporotrichosis - If complex cyst  rule out malignancy.
(rare)
d. Calcium deposits 
b. Epidemic puerperal mastitis - benign
- MRSA: causative organism - cause: cellular secretions , debris or by trauma and
- Transmission via suckling neonate inflammation.
- Tx: stop breastfeeding , antibiotics & I&D - (+) cancer if <0.5 mm in size, fine, linear calcifications,
may branch (microcalcifications).
c. Nonepidemic (sporadic) puerperal mastitis
- involvement of the interlobular CT of the breast e. Sclerosing adenosis
- tx: Emptying of the breast using breast suction pumps + - Common in childbearing and perimenopausal years
antibiotics - no malignant potential.

TOPNOTCH MEDICAL BOARD PREP SURGERY SUPPLEMENT HANDOUT Page 38 of 83


For inquiries visit www.topnotchboardprep.com.ph or email us at topnotchmedicalboardprep@gmail.com
TOPNOTCH MEDICAL BOARD PREP SURGERY SUPPLEMENT HANDOUT - Jules Lopez,MD-MBA,Teddy Carpio,MD-MBA
For inquiries visit www.topnotchboardprep.com.ph or email us at topnotchmedicalboardprep@gmail.com
- characterized by distorted breast lobules + multiple 4fold after > 4 years of use + no reduction
microcysts + benign calcifications in CAD or CVD

f. Radial scars (1 cm or less) or Complex central  screening mammogram


sclerosis (more than 1 cm)  routine screening mammography starting 50
- characterized by central sclerosis, epithelial years old age reduces mortality from breast
proliferation, apocrine metaplasia, and papilloma cancer by 33%
formation  baseline mammography at age 35
- can mimic cancer hence an excisional biopsy is done to  annual mammographic screening
to exclude diagnosis of cancer beginning at age 40.
 If (+) family history for breast cancer
g. Ductal hyperplasia  Baseline mammogram 10 years before
- Severity: the youngest age of diagnosis of breast ca
 Mild: 3-4 cell layers above the basement among 1st degree relatives. (this rule is
membrane. modified if age of diagnosis is less than
 Moderate:5 or more cell layers above the basement 35)
membrane.
 Florid ductal epithelial hyperplasia: occupies at  Chemoprevention
least 70% of a minor duct lumen.  Tamoxifen: selective estrogen receptor
- associated with an increased cancer risk modulator
 recommended only for women who have
h. Intraductal papillomas  a Gail relative risk of 1.70 or ↑er.
- Seen in premenopausal women.  SE: deep vein thrombosis, pulmonary
- common symptom: serous or bloody nipple emboli, endometrial cancer
discharge  reduce the incidence of LCIS and ductal
- Gross appearance: pinkish tan, friable, carcinoma in situ (DCIS)
- rarely undergo malignant transformation & no  Raloxifene: estrogen receptor modulator
increased risk of breast cancer, unless multiple  Equivalent to tamoxifen
 associated with a more favorable
i. Atypical proliferative disease adverse event profile
- has some of the features of Ca in situ but lack a major  no effect on LCIS or DCIS
defining feature of Ca in situ or have the features in less
than fully developed form  prophylactic mastectomy
- Atypical ductal hyperplasia & lobar hyperplasia  greatly reduces risk for breast cancer
 Increases risk of breast cancer 4x; if with (+)  only for high risk populations
family hx, 10x  + 3 years  if with 40% risk of having breast
Ca
F. BREAST CANCER   + 5 years  if with 85% risk of having breast
- risk factors cancer
 increased exposure to estrogen: early menarche,
nulliparity, late menopause, older age at first live - BRCA mutation
birth (after the age of 30 yo), HRT, obesity, (major  Constitutes 5-10% of breast cancers
source of estrogen in postmenopausal women is  Autosomal dominant inheritance
the conversion of androstenedione to estrone by  tumor-suppressor genes
adipose tissue)  prevalent in Ashkenazi Jews
 radiation exposure: patients with multiple
fluoroscopies, mantle radiation for treatment of Table 50. Comparison of BRCA 1 & 2
hodgkin’s lymphoma BRCA 1 BRCA 2
 increased alcohol intake (leads to increased - location: ch arm 17q -location: ch arm 13q
estradiol levels) -predisposing genetic factor: - lifetime risk for carrier 
45% of breast Ca & 85% of Ca: 85% for breast ca & 20%
 high fat diet (increased serum estrogen levels) ovarian Ca for ovarian ca ; if male
 prolonged use of OCPs (particularly estrogen- -lifetime risk for carrier  carrier: 6%
plus-progesterone) and HRT Ca: 90% for breast ca & 40% -usually develops invasive
 (+) family history of breast cancer: the greater for ovarian ca ductal carcinomas: well
the number of relatives affected, the closer the - usually develops invasive differentiated, hormone
genetic relationship, the younger the age at ductal carcinomas: poorly receptors (+)
diagnosis, and the presence of bilateral versus differentiated and hormone -early age of onset, bilateral
unilateral disease all increased the likelihood of receptor (-) breast cancer, & other
-early age of onset, bilateral associated ca: ovarian, colon,
development of breast cancer in an individual. breast cancer and other prostate, pancreatic,
associated ca: ovarian, colon gallbladder, bile duct &
 MUST KNOW and prostate stomach cancers, melanoma.
Remember:
 Risk mgt strategies for BRCA carriers:
Smoking is not considered a risk factor for breast  Prophylactic mastectomy and reconstruction
cancer!!!!!!!!!!! Please don’t make the mistake of answering this  Prophylactic oophorectomy (because of ↑ risk
as part of the risk factors in breast cancer. of ovarian ca) at the completion of
childbreaing or manopause + HRT
 Intensive surveillance for breast and ovarian
- risk management cancer
 Postmenopausal hormone replacement tx  Chemoprevention
 Widely prescribed because it is effective in  BRCA mutation carriers who do not undergo
controlling symptoms of estrogen deficiency prophylactic mastectomy must do clinical breast
(vasomotor symptoms such as hot flashes, examination every 6 months and mammography
night sweats and their associated sleep every 12 months beginning at age 25 years
deprivation, osteoporosis, and cognitive (because the risk of breast cancer in BRCA
changes) mutation carriers increases after age 30 years)
 Results of Women’s health initiative study
(2002): breast Ca risk is increased to 3- - HER-2 gene
TOPNOTCH MEDICAL BOARD PREP SURGERY SUPPLEMENT HANDOUT Page 39 of 83
For inquiries visit www.topnotchboardprep.com.ph or email us at topnotchmedicalboardprep@gmail.com
TOPNOTCH MEDICAL BOARD PREP SURGERY SUPPLEMENT HANDOUT - Jules Lopez,MD-MBA,Teddy Carpio,MD-MBA
For inquiries visit www.topnotchboardprep.com.ph or email us at topnotchmedicalboardprep@gmail.com
 Encodes transmembrane tyrosine kinase, a protein LCIS DCIS
with potent growth stimulating activity Interval to 15-20 y 5-10 y
 In breast cancer, this gene is amplified (indicating diagnosis
more rapid growth & aggressive behavior) Histo type Ductal; cytoplasmic ductal
mucoid globules are
 Treatment if there is a mutation: Herceptin
characteristic
Specifics Only in ♀ breast; more Other name:
- Breast cancer signs and symptoms:
common in whites; not intraductal
 mass (most common) considered an carcinoma (true
 if size is 1 cm  mass has been present for 5 anatomic precursor anatomic
years of breast ca; only a precursor); 5%
 breast enlargement or asymmetry risk marker of male cancers
 nipple changes - retraction, or discharge ( due to (increases risk of
shortening of Cooper's suspensory ligament) breast ca 9x)
 skin dimpling
 ulceration / erythema of the skin  Tx:
 axillary mass or mets  LCIS: observation, chemoprevention
 firm or hard with continued growth of the with tamoxifen, and bilateral total
metastatic cancer. mastectomy or may opt to do close follow
 involved sequentially from the low (level I) to up + periodic PE + bilateral mammograms
the central (level II) to the apical (level III) for a more conservative approach
lymph node groups.  DCIS:
 axillary lymph node status: most important o > 4 cm or disease in >1 quadrant:
prognostic correlate of disease-free and mastectomy
overall survival o Low-grade DCIS of the solid,
 peau d'orange (Localized edema): blocked cribriform, or papillary subtype that
drainage of lymph fluid is <0.5 cm: lumpectomy (If margins
 musculoskeletal discomfort. are free of disease)
 Distant metastases: most common cause of death o Adjuvant tamoxifen therapy has a
in breast cancer patients role for DCIS pt.
 Due to neovascularization (hematogenous
spread)  cancer cells shed directly to  MUST KNOW a
axillary and intercostals veins or vertebral Remember:
column via batson’s plexus of veins.  Mastectomy vs lumpectomy + adjuvant RT: same
 Metastatic foci occurs after the 1o ca exceeds mortality rate (<2%) but lumpectomy + adjuvant RT has
0.5 cm in diameter a higher local recurrence rate (up to 9%, compared to 2%
 Common sites of involvement (in order of for mastectomy)
frequency): bone, lung, pleura, soft tissues, and  Role of RT: markedly decreases the risk of in-breast
liver recurrence and significantly reduces the risk that any
recurrence will be invasive disease
 MUST KNOW a  High recurrence rate for DCIS comedo type
Remember:
 Breast pain is usually associated with benign disease.
- Invasive Breast Cancer
 Paget's disease of the nipple (unrelated to Paget’s
- In situ breast cancer disease of the bone)
 Multicentricity: occurrence of a second  chronic, eryhthematous, eczemamatoid rash
breast cancer outside the breast quadrant of or ulcer
the primary cancer (or at least 4 cm away)  associated with DCIS & invasive cancer.
 Multifocality: the occurrence of a second  Pathognomonic sign: large, pale,
cancer within the same breast quadrant as the vacuolated cells (Paget cells) in the rete
primary cancer (or within 4 cm of it) pegs of the epithelium.
 Difficult to differentiate from atypical  Rule out superficial spreading melanoma
hyperplasia or cancers with early invasion  (+) s-100 antigen in immunostaining (vs
 Subtypes: paget’s disease which is (+) in
carcinoembryonic antigen
Table 51. Comparison of LCIS vs DCIS
LCIS DCIS
immunostaining)
Age 44-47 54-58  Tx: lumpectomy, mastectomy, or MRM
Incidence 2-5 5-10 (depending on the extent of involvement and
Clinical sx None Mass, pain & the presence of invasive cancer)
nipple discharge  Invasive ductal carcinoma
Mammographic None / Microcalcification  Occurs in perimenopausal or postmenopausal
sx mammographically s (usually in areas ♀ (5th-6th decade)
featureless; may have of necrosis)  Most common carcinoma presenting as a
calcifications in breast mass
adjacent tissues
 poorly defined margin, central stellate
occasionally
Premenopausal 2/3 (more common) 1/3
configuration with chalky white or yellow
streaks extending into surrounding breast
Incidence of 5% 2-46% tissues
synchronous  macroscopic/microscopic axillary LN
invasive ca metastases in 60% of cases
Multicentricity 60-90 % 40-80% (more
common if  Medullary carcinoma
comedo type)  Associated with BRCA1phenotype & DCIS
Bilaterality 50-70% (more 10-20%  Gross appearance: well circumscribed, soft &
common)
hemorrhagic (when accompanied with a rapid
Axillary 1% 1-2%
metastasis increase in size)
Subsequent carcinoma  PE: bulky and mass is positioned deep within
Laterality Bilateral Ipsilateral the breast.
TOPNOTCH MEDICAL BOARD PREP SURGERY SUPPLEMENT HANDOUT Page 40 of 83
For inquiries visit www.topnotchboardprep.com.ph or email us at topnotchmedicalboardprep@gmail.com
TOPNOTCH MEDICAL BOARD PREP SURGERY SUPPLEMENT HANDOUT - Jules Lopez,MD-MBA,Teddy Carpio,MD-MBA
For inquiries visit www.topnotchboardprep.com.ph or email us at topnotchmedicalboardprep@gmail.com
 Can mimic a benign condition on diagnostic  Ca: may appear as irregular masses or as
imaging (looks like a fibroadenoma on multiple intraluminal filling defects
UTZ)
 Occurs bilaterally in 20% of cases.  Ultrasonography
 Microscopically: dense lymphoreticular  Ideal for younger patients (because of
infiltrate tendency to have denser breasts – can affect
results if mammography is used)
 Mucinous (colloid) carcinoma  Useful for resolving equivocal mammographic
 Occurs in the elderly findings, defining cystic masses, and
 Characteristic lesion: extracellular pools of demonstrating the echogenic qualities of
mucin specific solid abnormalities.
 Gross appearance: glistening & gelatinous with  breast cysts: well circumscribed, with smooth
a firm consistency margins and an echo-free center
 features of benign breast masses: smooth
 Papillary carcinoma contours, round or oval shapes, weak internal
 Usually occurs in the 7th decade of life echoes, and well-defined margins.
 More common in nonwhite ♀.  Features of breast ca: irregular walls but may
 defined by papillae with fibrovascular have smooth margins with acoustic
stalks and multilayered epithelium. enhancement.
 does not reliably detect lesions that are 1cm.
 Tubular carcinoma
 Usually occurs during perimenopausal or early - Breast cancer staging
menopausal periods.  Clinically based
 Microscopically: haphazard array of small,  tumor size correlates with the presence of axillary
randomly arranged tubular elements is seen. lymph node metastases
 Well-differentiated type of infiltrating ductal  The single most important predictor of 10- and
cancer 20-year survival rates in breast cancer is the
 Favorable diagnosis number of axillary lymph nodes involved with
metastatic disease.
 Invasive lobular carcinoma
 Histopath: small cells with rounded nuclei, Table 52. TNM breast cancer staging
inconspicuous nucleoli, and scant cytoplasm; T N M
(+) intracytoplasmic mucin, which may T1: <2cm N1: suspicious M1: (+) lung,
T2: 2-5 cm mobile liver or bone
displace the nucleus (signet-ring cell
T3: >5cm axillary nodes involvement
carcinoma). T4: (+) chest N2: matted or
 frequently multifocal, multicentric, and wall & direct fixed axillary
bilateral. skin nodes
 Hard to detect mammographically involvement N3: ipsilateral
internal
- Diagnosis of breast cancer mammary
 Mammography: nodes
 Views:
 Craniocaudal (CC) view: medial aspect of  (+) supraclavicular nodes: stage III disease (not
the breast; permits greater breast stage IV as formerly classified)
compression
 mediolateral oblique (MLO) view: images - Treatment for breast cancer
the greatest volume of breast tissue &  Treatment is dependent on the stage at diagnosis
upper outer quadrant and the axillary tail  Early invasive breast cancer (stage I, IIa, IIb)
of Spence  Lumpectomy +/- RT (breast conservation sx)
 Features suggestive of breast cancer: is an acceptable tx option since survival rates
are comparable to total mastectomy.
 solid mass +/- stellate features
 However, recurrence ↑er in the
 asymmetric thickening of breast tissues
lumpectomy with no RT stage I and II
 clustered microcalcifications
breast cancer.
 presence of fine, stippled calcium in &
 CI to breast conservation sx:
around a suspicious lesion is suggestive of
 prior RT to the breast or chest wall
breast cancer; occurs in 50% of
 involved surgical margins or unknown
nonpalpable cancers.
 Mimickers of breast ca mammographically: margin status after re-excision
radial scars, fibromatosis, granular cell  multicentric disease
tumor and fat necrosis (surgical excision is  scleroderma or lupus erythematosus.
indicated for these lesions, owing to their  If clinically negative nodes but with T1-T2
resemblance to ca) primary ca  perform sentinel LN
 % reduction in mortality for women after dissection
screening mammography.  If (+): perform axillary lymph node
 Recommendations: dissection should be performed.
 normal-risk women at 20 yo  breast  Adjuvant chemotherapy is indicated for node-
examination every 3 years positive cancers, >1 cm, and node-negative
 at age 40 yo  annual breast examination cancers of >0.5 cm when adverse prognostic
/ mammography features (blood vessel or lymph vessel
 false (-)/(+) rate: 10% invasion, high nuclear grade, high histologic
grade, HER-2/neu overexpression, and
negative hormone receptor status).
 Ductography  Tamoxifen therapy: women with hormone
 Indication: is nipple discharge, (particularly receptor (+) cancer that are >1 cm.
when bloody)  HER-2/neu expression is determined for node
 Intraductal papillomas are seen as small filling (-) breast ca
defects surrounded by contrast media  Trastuzumab: medication for HER-2/neu
(–)

TOPNOTCH MEDICAL BOARD PREP SURGERY SUPPLEMENT HANDOUT Page 41 of 83


For inquiries visit www.topnotchboardprep.com.ph or email us at topnotchmedicalboardprep@gmail.com
TOPNOTCH MEDICAL BOARD PREP SURGERY SUPPLEMENT HANDOUT - Jules Lopez,MD-MBA,Teddy Carpio,MD-MBA
For inquiries visit www.topnotchboardprep.com.ph or email us at topnotchmedicalboardprep@gmail.com
 doxorubicin, cyclophosphamide, & o injury to the thoracodorsal
paclitaxel: medication for HER-2/neu (+) vascular pedicle  ischemic loss of
& node (+) breast cancer. the entire latissimus dorsi flap
utilized for reconstruction
 Advanced local regional breast cancer (stage o injury to the medial pectoral
IIIa or IIIb) pedicle  progressive atrophy of
 Surgery (MRM) + adjuvant RT + CT the pectoralis muscle
(neoadjuvant) o injury to the 2nd intercostals
 Role of CT: maximize distant disease-free brachiocutaneous nerve 
survival hypesthesia of the upper inner
 Role of RT: maximize local-regional aspect of the ipsilateral arm
disease-free survival.  Halsted radical mastectomy: removes all
 If stage IIIA ca: neoadjuvant breast tissue and skin, the nipple-areola
(preoperative) CT  reduce the size of complex, the pectoralis major and pectoralis
the primary ca & permit breast- minor muscles & the level I, II, and III LN.
conserving surgery.
- Non surgical breast cancer tx
Figure 29. Treatment pathyway for stage IIIa & IIIb cancer  RT
 Adjuvant RT after mastectomy  decrease
local recurrence rates but will not prolong
survival 
o Indicated for those with high risk for
local recurrence: large tumors, skin
involvement, > 4 axillary LN involved
 Chemotx
 Adjuvant chemotheraphy
 Indicated if node (-) tumor >1cm that are
ER (-)
 Neoadjuvant chemotherapy
 Neoadjuvant endocrine therapy
 Tamoxifen: indicated if node (-) tumor
>1cm that are ER (+)
 Distant metastases (stage IV)  Herceptin: (+) her2/neu
 Not anymore curative but may prolong
survival - breast cancer in pregnancy
 occurs in 1 of every 3000 pregnant ♀
- Breast cancer prognosis  TX:
 5 year survival rate  MRM: 1st & 2nd trimesters of pregnancy
 Stage I: 94%  lumpectomy with axillary node dissection: 3rd
 stage IIA: 85% trimester
 stage IIB: 70%  adjuvant RT: after delivery.
 stage IIIA: 52%
 stage IIIB: 48% - male breast cancer
 stage IV: 18%  <1% of all breast cancers occur in men
 preceded by gynecomastia in 20% of men.
- surgical techniques in breast cancer tx  associated with radiation, estrogen tx, testicular
 sentinel LN dissection: used to assess the regional feminizing syndromes, and Klinefelter's syndrome
LN in women with early breast ca who are (XXY)
clinically node negative by PE & imaging studies  usual types of cancer: DCIS, infiltrating ductal ca
 breast conservation therapy (BCT)  Overall, men do worse because of the advanced
 if stage 0, I & II, BCT is preferable to total stage of their ca (stage III or IV) at the time of
mastectomy ( with equivalent survival rates) diagnosis and poorer prognosis. But stage for stage,
 mastectomy and axillary dissection the results of treatment are similar to those in
 simple mastectomy: removes all breast women.
tissue, the nipple-areola complex, skin & level I
LN G. special clinical situations
 Modified radical mastectomy: removes all a. nipple discharge
breast tissue, the nipple-areola complex, skin,  suggestive of cancerous lesion: spontaneous,
& level I and level II LN. unilateral, localized to a single duct, present in
 preserves pectoralis major, pectoralis women 40 years of age, bloody, clear, serous, or
minor, level III LN & medial (anterior associated with a mass.
thoracic) pectoral nerve  Suggestive of a benign condition: bilateral,
 complications  multiductal in origin, occurs in women 39 years of
o most frequent: Seromas beneath the age, or is milky or blue-green.
skin flaps or in the axilla  Consider prolactin-secreting pituitary
o injury to the long thoracic nerve adenomas ( ↑ serum prolactin levels, Optical
(affects serratus anterior)  nerve compression, visual field loss, &
winging of scapula infertility)
o lymphatic fibrosis  painless, slow
progressive swelling of the involved b. Cystosarcoma phylloides tumor
arm - Resembles a giant fibroadenoma
o injury to the axillary vein  - Can occur in benign and malignant forms
sudden painful early postoperative - gross appearance: classical leaf-like (phyllodes)
swelling of the involved arm (due to appearance; greater cellular activity than fibroadenoma
acute thrombosis as the collateral - metastasis is usually vascular and no axillary LN
channels do not have the chance to involvement is expected.
develop  acute and painful) - tx:
 if benign  total excision with 2-3 cm margin

TOPNOTCH MEDICAL BOARD PREP SURGERY SUPPLEMENT HANDOUT Page 42 of 83


For inquiries visit www.topnotchboardprep.com.ph or email us at topnotchmedicalboardprep@gmail.com
TOPNOTCH MEDICAL BOARD PREP SURGERY SUPPLEMENT HANDOUT - Jules Lopez,MD-MBA,Teddy Carpio,MD-MBA
For inquiries visit www.topnotchboardprep.com.ph or email us at topnotchmedicalboardprep@gmail.com
 if malignant  total mastectomy w/o axillary LN usually with a receptor-blocking agent (tamoxifen).
dissection; if small  wide excision with 2cm However, bilateral oophorectomy in premenopausal
margin is acceptable women is still considered a reasonable option. Surgical
 if large  mastectomy. hypophysectomy & adrenalectomy were at one point
 Follow up is important due to high local recurrence considered forms of hormonal manipulation, but are
rate now being replaced by “medical adrenalectomy” in the
form of anastrazole, which inhibit the production of
c. Inflammatory breast cancer  adrenal steroids and conversion of androgens to
- variant of infiltrating ductal ca estrogens in the adrenal gland and peripherally. The
- characterized by the skin changes of brawny aromatase inhibitors are beneficial only in
induration, erythema with a raised edge, and postmenopausal women.
edema or peau d'orange (hence the name
inflammatory) + breast mass 3. a 39 year old woman presents with an ill-defined 2 cm
- appearance is due to a dermal lymphatic mass in the outer quadrant of her breast. Mammography
invasion shows very dense tissue but no discrete lesion.
Ultrasound examination shows a solid lesion. An
ultrasound-guided fine needle aspiration (FNA) is
SURGICAL TECHNIQUES FOR BREAST SURGERY performed, and the aspirate is plated, fixed, and sent to
the laboratory for cytologic study. A highly cellular
1. Skin-Sparing Mastectomy: removes ALL breast tissue, monomorphic pattern is seen, with poorly cohesive
the nipple-areola complex and scars from any prior intact cells, nuclear “crowding” with a variation in
biopsy procedure nuclear size, radial dispersion and clumping of the
2. Total (Simple) Mastectomy: removes ALL breast tissue, chromatin, and prominent nucleoli. Which of the
the nipple-areola complex and skin following management choices is/are appropriate?
3. Extended Simple Mastectomy: removes ALL breast
tissue, the nipple-areola complex, skin and level I a. MRM
axillary lymph nodes b. Reassuring the patient that the process is
4. Modified Radical Mastectomy: removes ALL breast benign
tissue, nipple-areola complex, skin and the level I and II c. Lumpectomy, sentinel lymph node biopsy and
axillary lymph nodes irradiation
5. Halsted Radical Mastectomy: removes ALL breast d. Excision of a fibroadenoma with narrow
tissue and skin, the nipple-areola complex, the margins
pectoralis major and pectoralis minor muscles and the e. Lumpectomy and sentinel lymph node biopsy
level I, II and III axillary lymph nodes without irradiation

Answer: A,C
Aspiration biopsy with a 22 gauge needle is an effective
and safe way of assessing palpable breast lesions.
REVIEW QUESTIONS a Performing the aspiration under ultrasound guidance
ensures that the lesion has been sampled thoroughly
1. a 58 yo woman presents with chronic, erythematous, while under direct vision. Although a smaller volume of
oozing, eczematoid rash involving the left nipple and tissue is obtained than the core needle biopsy, FNA
areola. There are no breast masses palpable, and her frequently yields results that may be equal to core
mammogram is normal. Which of the following biopsy if read by an experienced cytopathologist. A
recommendations is appropriate? fibroadenoma would show broad sheets of cohesive
cells with nuclei that are unfirm in size and shape. The
a. Referral to a dermatologist chromatin pattern would be finely granular and large
b. Oral vitamin E and topical aloe and lanolin numbers of bare nuclei would be present. The cytologic
c. Biopsy findings described in this question is diagnostic of
d. Non allergenic brassiere carcinoma. Appropriate management, therefore,
e. Standard treatment that includes breast includes either a modified radical mastectomy or
conservation lumpectomy, axillary evaluation by either a sentinel
lymph node biopsy or an axillary nodal dissection, and
Answer: C whole-breast irradiation.
This is a case of Paget’s disease of the breast. It is a
case of primary ductal carcinoma that secondarily
invades the epithelium of the nipple and areola. Biopsy
of any chronic nipple rash is mandatory and will show HEAD and NECK: BENIGN CONDITIONS & TUMORS
the distinctive pagetoid cells. Because of the possible
invasion of the tumor on the underlying rich lymphatics A. Risk factors for tumors of head and neck
of the nipple areolar complex, mastectomy is usually B. Anatomy of Oral cavity
indicated. In selected cases, breast conservation C. Cancer of the Lip
therapies can also be employed. D. Cancer of the Tongue
E. Tumors of Alveolus/gingiva
2. If patient with metastatic breast ca is ER (+), which of F. Anatomy of pharynx
the following statements are appropriate? G. Tumors of Nasopharynx
H. Tumors of Oropharynx
a. Bilateral oophorectomy I. Tumors of Hypopharynx/cervical esophagus
b. Antiestrogen drugs (tamoxifen) J. Anatomy Larynx
c. Hypophysectomy K. Benign conditions of the Larynx
d. Adrenalectomy L. Laryngeal Carcinoma
e. Aromatase inhibitor M. Neck and associated conditions
N. Salivary gland tumors
Answer: A,B,E O. Thyroid and associated conditions
Patients with high ER & PR levels (based on
immunohistomchemical stains) have better prognosis
compared to those with zero or low levels. The most
common hormonal manipulation is estrogen withdrawal,
TOPNOTCH MEDICAL BOARD PREP SURGERY SUPPLEMENT HANDOUT Page 43 of 83
For inquiries visit www.topnotchboardprep.com.ph or email us at topnotchmedicalboardprep@gmail.com
TOPNOTCH MEDICAL BOARD PREP SURGERY SUPPLEMENT HANDOUT - Jules Lopez,MD-MBA,Teddy Carpio,MD-MBA
For inquiries visit www.topnotchboardprep.com.ph or email us at topnotchmedicalboardprep@gmail.com
A. RISK FACTORS FOR TUMORS OF HEAD AND NECK D. CANCER OF THE TONGUE
- muscular structure with overlying nonkeratinizing
- tobacco & alcohol: most common preventable risk squamous epithelium.
factors associated with head and neck CA. - Posterior border: circumvallate papillae
- betel nut chewing - Tongue cancer
- reverse smoking  Same risk factors with other H&N CA
- HPV 16 and 18.  Associated with plummer-vinson syndrome
- UV light exposure (for lip CA) (cervical dysphagia, IDA, atrophic oral mucosa,
- Patients with H&N CA are predisposed to the brittle spoon finger nails)
development of a 2nd tumor within the aerodigestive  Clinical findings: ulcerations or as exophytic
tract. masses
 presentation of a new-onset dysphagia,  The regional lymphatics of the oral cavity are to the
unexplained weight loss, or chronic submandibular space and the upper cervical
cough/hemoptysis must be assessed lymph nodes
thoroughly in patients with a history of prior  Involvement of lingual nerve  ipsilateral
treatment for a head and neck cancer paresthesias
 ex. If (+) primary malignancy of oral cavity  Involvement of hypoglossal nerve  deviation of
orpharynx  secondary malignancy at cervical tongue on protusion + fasciculations  atrophy
esophagus; (+) primary malignancy at larynx   most common location: lateral and ventral surfaces
secondary malignancy at lungs  if base of the tongue  advanced stage and poorer
- Synchronous neoplasm: a 2nd 1o tumor detected within prognosis
6 months of the diagnosis of the initial primary lesion  tx:
- Metachronous tumor: detection of a 2nd 1o lesion more  Surgical treatment of small (T1–T2) primary
than 6 months after the initial. tumors is wide local excision with either
- Initial evaluation of patients with primary CA of H&N: primary closure or healing by secondary
"panendoscopy." intention.
 If base of tonguePartial glossectomy with
B. ANATOMY OF ORAL CAVITY supraomohyoid dissection if N0 or MRND if
- Borders: N(+)
 Anterior: vermilion border of the lip
 Superior: hard-palate/soft-palate junction E. TUMORS OF ALVEOLUS/GINGIVAL
 Inferior: circumvallate papillae - Because of the tight attachment of the alveolar mucosa
 Lateral: anterior tonsillar pillars to the mandibular and maxillary periosteum, treatment
- The oral cavity includes lips, alveolar ridges, oral of lesions of the alveolar mucosa frequently requires
tongue, retromolar trigone, floor of mouth, buccal resection of the underlying bone.
mucosa, and hard palate. - Diagnosis for alveolar or gingival cancer
- Regional metastatic spread of lesions of the oral cavity  Panorex: demonstrate gross cortical invasion
is to the lymphatics of the submandibular and the  CT: imaging subtle cortical invasion
upper jugular region (levels I, II, and III)  MRI: demonstrates invasion of the medullary
- Majority of tumors in the oral cavity are squamous cavity
cell carcinoma (>90%) - Tx for alveolar or gingival cancer
 If minimal bone invasion: mandibular resection
C. CANCER OF THE LIP  If (+) medullary cavity invasion: segmental
- most commonly seen old people (50-70 years old) with mandibulectomy
fair complexion
- Risk factors: prolonged exposure to sunlight, fair F. ANATOMY OF PHARYNX
complexion, immunosuppression, and tobacco use. - three regions:
- Most common location: lower lip (88 to 98%), upper  nasopharynx
lip (2 to 7%) & oral commissure (1%).  extends from the posterior nasal septum and
- Predominantly squamous cell CA choana to the skull base
- Basal cell carcinoma presents more frequently on the  includes fossa of rossenmuller, Eustachian
upper lip than lower. tube orifices (torus tuberous) and adenoid pad
- Clinical findings:  bilateral regional metastatic spread in this
 ulcerated lesion on the vermilion or cutaneous area is common
surface.  Lymphadenopathy of the posterior triangle
 (+) paresthesia in the area of lesion: mental (level V) of the neck should provoke
nerve involvement. consideration for a nasopharyngeal primary
- unfavorable prognosticating factors: perineural  Oropharynx:
invasion, involvement of maxilla/mandible, upper lip or  Includes tonsillar region, base of tongue, soft
commissure involvement, regional lymphatic palate, and posterolateral pharyngeal walls
metastasis, and age younger than 40 years at onset.  Regional lymphatic drainage for
- primary echelon of nodes at risk is in the oropharyngeal lesions frequently occurs to the
submandibular and submental regions upper and lower cervical lymphatics
- Tx: (levels II, III, IV) +Retropharyngeal
 T1 & T2 (≤4cm): Surgery = RT metastatic spread
 T3 & T4: surgical excision with histologic  hypopharynx.
confirmation of tumor-free margins + postop RT  extends from the vallecula to the lower border
 Prophylactic supraomohyoid neck dissection of the cricoid posterior and lateral to the
should be considered for patients with tumors larynx.
greater than 4 cm, desmoplastic tumor & (+)  includes pyriform fossa, the postcricoid space,
perineural invasion and posterior pharyngeal wall.
 Realignment of the vermilion border during the  Regional lymphatic spread is frequently
reconstruction and preservation of the oral bilateral and to the mid- and lower cervical
commissure (when possible) are important lymph nodes (levels III, IV)
principles in attempting to attain an acceptable
cosmetic result. G. TUMORS OF THE NASOPHARYNX
- Prognosis is most favorable for all H&N CA - Tumors arising in the nasopharynx are usually of
squamous cell origin
TOPNOTCH MEDICAL BOARD PREP SURGERY SUPPLEMENT HANDOUT Page 44 of 83
For inquiries visit www.topnotchboardprep.com.ph or email us at topnotchmedicalboardprep@gmail.com
TOPNOTCH MEDICAL BOARD PREP SURGERY SUPPLEMENT HANDOUT - Jules Lopez,MD-MBA,Teddy Carpio,MD-MBA
For inquiries visit www.topnotchboardprep.com.ph or email us at topnotchmedicalboardprep@gmail.com
- Most common nasopharyngeal malignancy in the  Bilateral neck dissection is frequently indicated
pediatric age group: lymphoma given the elevated risk of nodal metastases found
- Risk factors for nasopharyngeal carcinoma: area of with these lesions
habitation & ethnicity (southern China, Africa,
Alaska, and in Greenland Eskimos.), EBV infection, & J. ANATOMY OF LARYNX:
tobacco use. - divided into 3 regions:
- Symptoms:  supraglottis: epiglottis (lined by stratified,
 nasal obstruction, posterior (level V) neck nonkeratinizing squamous epithelium), false vocal
mass, epistaxis, headache, serous otitis media cords (lined by pseudostratified, ciliated
with hearing loss, and otalgia. respiratory epithelium), medial surface of the
 Cranial nerve involvement is indicative of skull aryepiglottic folds, and the roof of the laryngeal
base extension and advanced disease. ventricles
- Lymphatic spread occurs to the posterior cervical,  has a rich lymphatic network, which
upper jugular, and retropharyngeal nodes. accounts for the high rate of bilateral spread
- Bilateral regional metastatic spread is common. of metastatic disease
- Diagnosis for nasopharyngeal CA:  glottis: the true vocal cords, anterior and posterior
 flexible or rigid fiber-optic endoscope commissure, and the floor of the laryngeal
 CT with contrast: determining bone destruction ventricle.
 MRI: assess for intracranial and soft-tissue  Subglottis: extends from below the true vocal
extension. cords to the cephalic border of the cricoid within
- Tx: chemoradiation the airway
 pseudostratified, ciliated respiratory
H. TUMORS OF THE OROPHARYNX epithelium
- Direct extension of tumors from the oropharynx into  Glottic and subglottic lesions: spread to the
these lateral tissues may involve spread into the cervical chain, paralaryngeal and paratracheal
parapharyngeal space LN
- histology of the majority of tumors in this region is
squamous cell carcinoma K. BENIGN CONDITIONS OF THE LARYNX
- (+) asymmetrical enlargement of the tonsils and tongue
base  think lymphoma Recurrent respiratory papillomatosis (RRP)
- Clinical findings: ulcerative lesion, exophytic mass, - (+)HPV 6 & 11
tumor fetor, muffled or "hot potato" voice (large tongue - larynx is the most frequently involved site
base tumors), Dysphagia, weight loss, Referred otalgia, - presents in early childhood, secondary to viral
(tympanic branches of CN IX & CN X), Trismus acquisition during vaginal delivery.
(involvement of the pterygoid musculature), ipsilateral - Sx: hoarseness, airway compromise
or bilateral nontender cervical lymphadenopathy - Diagnosis: endoscopy
- LN metastasis from oropharyngeal cancer most - Tx: operative microlaryngoscopy with excision or laser
commonly occurs in the subdigastric area of level II. ablation
Others - levels III, IV, & V, retropharyngeal & - High tendency to recur
parapharyngeal LN.
 Bilateral metastases: seen in tumors originating Laryngeal granulomas
from the tongue base and soft palate; if found in - typically occur in the posterior larynx on the arytenoid
these areas  associated with poor survival mucosa
- Tx: - risk factors: reflux, voice abuse, chronic throat clearing,
 Options: surgery, primary radiation alone, surgery endotracheal intubation, and vocal fold paralysis
with postoperative radiation, & combined - Sx: pain often with swallowing (less commonly: vocal
chemotherapy with radiation therapy. changes)
 If tongue base crossing middling: do total - Dx: fiber-optic laryngoscopy, voice analysis, laryngeal
glossectomy with possible total laryngectomy electromyography (EMG), and pH probe testing.
 Tumors of the oropharynx tend to be - Tx: voice rest, voice retraining therapy, and antireflux
radiosensitive. therapy.

I. TUMORS OF THE HYPOPHARYNX/CERVICAL Reinke's edema


ESOPHAGUS - located at the superficial lamina propria due to injury to
- Squamous cancers of the hypopharynx frequently the capillaries that exist in this layer, with subsequent
present at an advanced stage, hence are associated with extravasation of fluid.
poorer survival rates - Sx: rough, low-pitched voice.
- Clinical findings: neck mass, muffled or hoarse voice, - Risk factors: smoking, laryngopharyngeal reflux,
referred otalgia, progressive dysphagia to solids  hypothyroidism, and vocal hyperfunction.
liquids, weight loss.
- Invasion of the larynx by direct extension  vocal Vocal cord cyst
cord paralysis (if unilaterally affected)  airway - may occur under the laryngeal mucosa (in regions
compromise (if bilaterally affected) containing mucous-secreting glands)
- Diagnosis: - Cysts of the vocal cord may be difficult to distinguish
 flexible fiber-optic laryngoscopy from vocal polyps
 CT and/or MRI imaging: check for regional - Diagnosis: video stroboscopic laryngoscopy
metastases (paratracheal and upper mediastinal - Tx: Large cysts of the supraglottic larynx are treated by
lymph nodes) marsupialization with cold steel or a CO2 laser.
- Tx:
 T1: RT Vocal cord paralysis
 T2 & T3: chemoradiation - most commonly is iatrogenic (s/p thyroid,
 Larynx-preserving surgical procedures: only if the parathyroid, carotid, or cardiothoracic surgeries)
tumor must not involve the apex of the pyriform - can be secondary to malignant processes in the lungs,
sinus, vocal cord mobility must be unimpaired, and thyroid, esophagus, thoracic cavity, skull base, or neck.
the patient must have adequate pulmonary - Sx: presents with hoarseness and “breathy” voice
reserve.  If superior laryngeal nerve is affected 
demonstrate aspiration secondary to diminished
supraglottic sensation

TOPNOTCH MEDICAL BOARD PREP SURGERY SUPPLEMENT HANDOUT Page 45 of 83


For inquiries visit www.topnotchboardprep.com.ph or email us at topnotchmedicalboardprep@gmail.com
TOPNOTCH MEDICAL BOARD PREP SURGERY SUPPLEMENT HANDOUT - Jules Lopez,MD-MBA,Teddy Carpio,MD-MBA
For inquiries visit www.topnotchboardprep.com.ph or email us at topnotchmedicalboardprep@gmail.com
 left vocal cord is more commonly involved 2. Level II: upper jugular chain nodes
secondary to its longer course of the recurrent - Level IIa: jugulodigastric nodes; deep to
laryngeal nerve (RLN) on that side sternocleidomastoid (SCM) muscle, anterior to the
 if anterior surgical approaches to the cervical spine posterior border of the muscle, posterior to the
are performed  right RLN is at an increased risk posterior aspect of the posterior belly of digastric,
(courses more laterally to the tracheoesophageal superior to the level of the hyoid, inferior to spinal
complex) accessory nerve (CN XI)
- Level IIb: submuscular recess; superior to spinal
L. LARYNGEAL CARCINOMA accessory nerve to the level of the skull base
- Suspect if with (+) Hx of smoking & complaint of a
change in vocal quality 3. Level III: middle jugular chain nodes
- are primarily squamous cell carcinoma - inferior to the hyoid, superior to the level of the cricoid,
- sx: deep to SCM muscle from posterior border of the
 supraglottic larynx: chronic sore throat, muscle to the strap muscles medially
dysphonia ("hot potato" voice), dysphagia, or a
neck mass secondary to regional metastasis, 4. Level IV: lower jugular chain nodes
Referred otalgia or odynophagia is encountered - inferior to the level of the cricoid, superior to the
with advanced supraglottic cancers. clavicle, deep to SCM muscle from posterior border of
 Glottic larynx: hoarseness (early; because only a the muscle to the strap muscles medially
small degree of change is required to produce jaime is the best!!
hoarseness), Airway obstruction (late), Decreased
vocal cord mobility may be caused by direct muscle 5. Level V: posterior triangle nodes
invasion or involvement of the RLN. - Level Va: lateral to the posterior aspect of the SCM
 Subglottic larynx: vocal cord paralysis (usually muscle, inferior and medial to splenius capitis and
unilateral) and/or airway compromise (are trapezius, superior to the spinal accessory nerve
relatively uncommon). - Level Vb: lateral to the posterior aspect of SCM muscle,
- Lymphatic drainage: medial to trapezius, inferior to the spinal accessory
 Supraglottic larynx: subdigastric and superior nerve, superior to the clavicle
jugular nodes
 glottic and subglottic larynx: prelaryngeal node 6. Level VI: anterior compartment nodes
(the Delphian node), the paratracheal nodes, and - inferior to the hyoid, superior to suprasternal notch,
the deep cervical nodes medial to the lateral extent of the strap muscles
 glottic cancers have limited lymphatic access  bilaterally
regional nodal metastases is low
- treatment 7. Level VII: paratracheal nodes
 early stage glottis & supraglottic cancer: RT - inferior to the suprasternal notch in the upper
 small glottic cancers: Partial laryngectomy mediastinum
 supraglottic cancers w/o arytenoid or vocal cord
extension: supraglottic laryngectomy Patterns of spread from primary tumor sites:
 advanced tumors with extension : total - oral cavity and lip: levels I, II, and III
laryngectomy + postop RT  Skip metastases may occur with oral tongue
 Subglottic cancers: total laryngectomy. cancers such that involvement of nodes in level III
or IV may occur without involvement of higher
echelon nodes (levels I & II).
M. NECK AND ASSOCIATED CONDITIONS  - oropharynx, hypopharynx, and larynx: levels II, III, and
- differential diagnosis of neck masses is dependent on its IV.
location and patient’s age - nasopharynx and thyroid: level V nodes in addition to
 pediatric age: think congenital or inflammatory the jugular chain nodes.
conditions - nasopharynx, soft palate, and lateral and posterior walls
 adult + risk factors: rule out malignancy of the oropharynx and hypopharynx: Retropharyngeal
 in terms of location, think about patterns of lymph nodes
drainage - hypopharynx, cervical esophagus, and thyroid:
paratracheal nodal compartment + upper mediastinum
Lymphatic drainage of the neck is divided into 7 levels. nodes (level VII).
- advanced tumors of the glottis with subglottic spread:
Figure 30. levels of the neck bearing LN bearing regions
Delphian node

Neck dissections:

- Radical neck dissection (RND or CRILE method):


removes levels I to V of the cervical lymphatics + SCM +
internal jugular vein + CN XI

Modified radical neck dissection (MRND) or


-
functional neck dissection: Any modification of the
RND that preserves nonlymphatic structures (i.e., CN XI,
SCM muscle, or internal jugular vein)
 Comparable to RND in controlling regional
metastasis with superior functional results
- Selective neck dissection (SND): any modification of
1. Level I: submental & submandibular nodes
the RND that preserves lymphatic compartments
- Level Ia: the submental nodes; medial to the anterior
normally removed in RND
belly of the digastric muscle bilaterally, symphysis of
 Also comparable to RND in controlling regional
mandible superiorly, and hyoid inferiorly
metastasis with superior functional results
- Level Ib: the submandibular nodes and gland; posterior
 Types:
to the anterior belly of digastric, anterior to the
1. supraomohyoid neck dissection
posterior belly of digastric, and inferior to the body of
 used with oral cavity malignancies
the mandible
 removes lymph nodes in levels I to III
2. lateral neck dissection
TOPNOTCH MEDICAL BOARD PREP SURGERY SUPPLEMENT HANDOUT Page 46 of 83
For inquiries visit www.topnotchboardprep.com.ph or email us at topnotchmedicalboardprep@gmail.com
TOPNOTCH MEDICAL BOARD PREP SURGERY SUPPLEMENT HANDOUT - Jules Lopez,MD-MBA,Teddy Carpio,MD-MBA
For inquiries visit www.topnotchboardprep.com.ph or email us at topnotchmedicalboardprep@gmail.com
 used for laryngeal malignancies
 removes lymph nodes in levels II through IV N. SALIVARY GLAND TUMORS
3. posterolateral neck dissection - Majority of neoplasms are benign
 Used for thyroid cancer - Most common gland involved: parotid gland (85% of
 removes lymph nodes in levels II to V all salivary gland neoplasms)
- if clinically N(+) necks: do MRND or RND or SND ( only - Most common benign tumor of the salivary gland:
if limited N1 disease) pleomorphic adenoma
- if (+) extracapsular spread, perineural invasion, - Most common malignant epithelial neoplasm of
vascular invasion, and the presence of multiple involved salivary gland: mucoepidermoid carcinoma
lymph nodes are noted  neck dissection of choice + - 2nd most common malignant epithelial neoplasm of
Adjuvant RT +/- chemoRT salivary gland:Adenoid cystic carcinoma, which has a
propensity for neural invasion,
Parapharyngeal space masses - Risk of malignancy depending on location: minor
- Is a potential space, shaped like an inverted pyramid salivary gland > submandibular, sublingual >
spanning the skull base to the hyoid. parotid gland
- Contents of the prestyloid space: parotid, fat, and lymph - Symptoms suggestive of malignancy: pain, paresthesias,
nodes. facial nerve weakness, skin invasion, fixation to the
- Contents of poststyloid compartment: CNs IX to XII, the mastoid tip and trismus (invasion of the masseter or
carotid space contents, cervical sympathetic chain, fat, pterygoid muscles
and lymph nodes. - Tx:
- Tumors in this space can produce displacement of the  If benign neoplasm: do surgical excision
lateral pharyngeal wall medially into the oropharynx  If parotid: minimal surgical procedure for
, dysphagia, cranial nerve dysfunction, Horner's neoplasms of the parotid is superficial
syndrome, or vascular compression. parotidectomy with preservation of the facial
- Tumors found in the parapharyngeal space: nerve.
 40 to 50% of the tumors are of salivary gland  Most frequently injured nerve in
origin parotid surgery: greater auricular
 usually arising anterior to the styloid nerve (not facial nerve!); if transected,
process will produce numbness of the lower
 20 to 25% of tumors are of neurogenic origin portion of the auricle & periauricular skin
such as paragangliomas (glomus vagale, carotid  If the auriculotemporal nerve is
body tumor), schwannomas, and neurofibroma injured  Frey’s syndrome
 usually arising posterior to the styloid (postoperative gustatory sweating)
process
 angiography has a role if the tumor in question  If malignant: do en bloc removal of the involved
is located posterior to the styloid process gland with preservation of all nerves unless
 if a paraganglioma is suspected  request for directly invaded by tumor.
a 24-hour urinary catecholamine  if parotid tumor arising from the lateral
 15% represent LN metastases & 1o lymphoma lobe: superficial parotidectomy with
ecpreservation of CN VII is indicated.
Benign neck masses   If the tumor extends into the deep lobe of the
parotid: a total parotidectomy with nerve
Thyroglossal duct cyst preservation is performed
- most commonly encountered congenital cervical  If submandibular involvement: en bloc
anomalies resection of the gland and submental and
- represents the vestigial remainder of the tract of the submandibular lymph nodes.
descending thyroid gland from the foramen cecum, at  Nerves at risk for a submandibular
the tongue base, into the lower anterior neck during gland removal: lingual and hypoglossal
fetal development. nerve
- An embryological anomaly wherein there is failure of  Postoperative radiation treatment plays an
obliteration of the midline pharyngeal diverticulum important role in the treatment of salivary
during thyroid descent malignancies. The presence of
- present as a midline or paramedian cystic mass extraglandular disease, perineural
adjacent to the hyoid bone. invasion, direct invasion of regional
- After an upper respiratory infection, the cyst may structures, regional metastasis, and high-
enlarge or become infected. grade histology are all indications for
- Tx: removal of the cyst, the tract, and the central portion radiation treatment
of the hyoid bone (Sistrunk procedure) + portion of
the tongue base up to the foramen cecum. O. THYROID AND ASSOCIATED CONDITIONS
- Check 1st for normal thyroid tissue in the lower neck
area & if ensure that patient is euthyroid Important facts about Thyroid anatomy:
- 1% of thyroglossal duct cysts contain cancer (85% is
usually papillary) - Weight of a normal thyroid gland: 20 g 
- pyramidal lobe is present in about 50% of patients
Congenital branchial cleft anomalies:  in disorders resulting in thyroid hypertrophy (e.g.,
- remnants are derived from the branchial cleft apparatus Graves' disease, diffuse nodular goiter, or
that persists after fetal development. lymphocytic thyroiditis), the pyramidal lobe
 1st branchial cleft: EAC & parotid gland. usually is enlarged and palpable
 2nd branchial cleft: courses between the internal - enveloped by a loosely connecting fascia
and external carotid arteries and proceeds into the - thyroidea ima artery: arises directly from the aorta or
tonsillar fossa innominate in 1-4% of individuals
 3rd branchial cleft: courses posterior to the - ligament of berry: posteromedial suspensory
common carotid artery, ending in the pyriform ligament; has a close relationship with the recurrent
sinus region. laryngeal nerve
- inferior thyroid artery crosses recurrent laryngeal
Dermoid cysts nerve (RLN), necessitating identification of the RLN
- midline masses and represent trapped epithelium before ligation
originating from the embryonic closure of the midline.
TOPNOTCH MEDICAL BOARD PREP SURGERY SUPPLEMENT HANDOUT Page 47 of 83
For inquiries visit www.topnotchboardprep.com.ph or email us at topnotchmedicalboardprep@gmail.com
TOPNOTCH MEDICAL BOARD PREP SURGERY SUPPLEMENT HANDOUT - Jules Lopez,MD-MBA,Teddy Carpio,MD-MBA
For inquiries visit www.topnotchboardprep.com.ph or email us at topnotchmedicalboardprep@gmail.com
- RLNs innervate all the intrinsic muscles of the
larynx, except the cricothyroid muscles, which are 5. Free T3
innervated by the external laryngeal nerves  - normal: 3 – 9 pmol/L
 Injury to one RLN: paralysis of the ipsilateral - most useful in the diagnosis of early hyperthyroidism in
vocal cord (lie in the paramedian or the abducted which levels of free T3 and T4 rise before total T3 and T4
position)
 Injury to Bilateral RLN: airway obstruction, 6. Serum TRH
necessitating emergency tracheostomy, or loss of - used for the evaluation of pituitary TASH secretory
voice. function
 Most common position of right RLN: posterior
to the inferior thyroid artery 7. Thyroid antibodies
- Injury to the internal branch of the superior - include anti-Tg, antimicrosomal, or anti-TPO and TSI
- anti-Tg & anti-TPO antibody levels: elevated if with
laryngeal nerve  aspiration.
autoimmune thyroiditis
- Injury to the external branch of the superior
- can be elevated in Hashimoto’s, Graves’, multinodular
laryngeal nerve  inability to tense the ipsilateral goiter & thyroid neoplasms
vocal cord and hence difficulty "hitting high notes”
- Loop of galen: where the pharyngeal branches of the 8. Serum Thyroglobulin
recurrent laryngeal nerve communicate with the - amount is increased in destructive processes of the
branches of the superior laryngeal nerve. Maybe thyroid gland (thyroiditis) or overactive states (graves’
injured when dissecting or ligating the superior or toxic multinodular goiter)
thyroid artery - most important use is for the monitoring of
- Regional lymph nodes include pretracheal, differentiated thyroid cancer recurrence, after total
paratracheal, perithyroidal, RLN, superior mediastinal, thyroidectomy and RAI ablation
retropharyngeal, esophageal, and upper, middle, and  elevated anti-Tg antibodies can interfere with the
lower jugular chain nodes. accuracy of Tg levels and should always be
- Histology: measured when interpreting Tg levels.
 the thyroid is divided into lobules that contain 20
to 40 follicles 9. Serum Calcitonin
 Each follicle is lined by cuboidal epithelial cells - normal: 0-4 pg/mL basal
 C cells or parafollicular cells: secrete the - secreted by C cells
hormone calcitonin. - function: lower serum calcium
- sensitive marker for medullary thyroid cancer
Evaluation of patients with thyroid disease:
Thyroid Imaging:
Tests of thyroid function:
1. Radionuclide imaging
1. Serum TSH
- normal: 0.5 – 5μU/mL Types:
- only test necessary in most patients with thyroid 1. iodine 123 (123I)
nodules that clinically appear euthyroid - emits low dose radiation
- serum TSH levels reflect the ability of the anterior - t 1/2 : 12 – 14 hours
pituitary to detect free T4 levels - used to image lingual thyroids or goiter
- ultrasensitive TSH assay: most sensitive and most
specific test for the diagnosis of hyperthyroidism and 2. iodine 131 (131I)
hypothyroidism - higher dose radiation exposure because of longer t ½
- t ½ : 8 to 10 days
2. Total T4 - used to screen and treat patients with differentiated
- normal: T4: 55 – 150 nmol/L thyroid cancers for metastatic disease
- Total T4 levels reflect the output from the thyroid gland ** Cold: trap less radioactivity compared to the
- Not suitable as a general screening test surrounding gland, risk of malignancy is higher in cold
- Increased levels seen in hyperthyroid patients, elevated lesions (20%) compared to hot lesions (<5%)
Tg levels secondary to pregnancy, ** Hot: trap more radioactivity, therefore, with
estrogen/progesterone use or congenital diseases increased acitivity
- Decreased levels seen in hypothyroid patients,
decreased Tg levels secondary to anabolic steroid use 3. technetium Tc 99m pertechnetate (99mTc)
and protein losing disorders (i.e. nephrotic syndrome) - this isotope is taken up by the mitochondria
 These individuals maybe euthyroid if their free - shorter t ½, therefore, less radiation exposure
T4levels are normal - sensitive for nodal metastases

3. Total T3 4. F-fluorodeoxyglucose PET scan


- Normal: 1.5 – 3.5 nmol/L - used to screen for metastases in patients with
- Total T3 levels reflect peripheral thyroid hormone thyroid cancer in whom other imaging studies are
metabolism negative.
- Not suitable as a general screening test - May show clinically occult lesions
- Measurement of total T3 levels is important for
clinically hyperthyroid patients with normal T4 levels  2. Ultrasound
think T3 thyrotoxicosis - excellent noninvasive imaging study of thyroid gland
- no radiation exposure
4. Free T4 - useful for the evaluation of thyroid nodules,
- Normal: 12 – 28 pmol/L distinguinshing cystic from solid ones, size,
- Measures the biologically active hormone multicentricity and cervical lymphadenopathy
- Not performed as a routine screening test in thyroid
disease 3. CT/MRI
- Its utility is in detecting early hyperthyroidism in which - useful for the evaluation of extent of large, fixed or
total T4 levels maybe normal but free T4 levels are substernal goiters and their relationship to the
raised airway and vascular structures
- Refetoff syndrome: end-organ resistance to T4 wherein
free T4 are increased and TSH levels are normal Developmental abnormalities:
TOPNOTCH MEDICAL BOARD PREP SURGERY SUPPLEMENT HANDOUT Page 48 of 83
For inquiries visit www.topnotchboardprep.com.ph or email us at topnotchmedicalboardprep@gmail.com
TOPNOTCH MEDICAL BOARD PREP SURGERY SUPPLEMENT HANDOUT - Jules Lopez,MD-MBA,Teddy Carpio,MD-MBA
For inquiries visit www.topnotchboardprep.com.ph or email us at topnotchmedicalboardprep@gmail.com
 If eye signs are present, other tests are generally
Thyroglossal duct cyst (see Neck) not needed.
 123I uptake and scan: elevated uptake, with a
Lingual thyroid diffusely enlarged gland, confirms the diagnosis
- failure of the median thyroid anlage to descend - Treatment:
normally  Antihyroid drugs:
- may appear as reddish brown mass at the base of the  propylthiouracil (PTU, 100 to 300 mg three
tongue times daily) or methimazole (10 to 30 mg
- may be the only thyroid tissue present (hence, if three times daily, then once daily – because it
surgical tx is warranted, evaluation of normal has a longer half t ½ )
thyroid tissue in the neck must be carried out 1st)  MOA: inhibits the organic binding of
- Intervention becomes necessary for obstructive iodine and the coupling of iodotyrosines
symptoms such as choking, dysphagia, airway (mediated by TPO).
obstruction, or hemorrhage or if suspicious for  PTU also inhibits the peripheral
malignancy conversion of T4 to T3
- Tx: administration of exogenous ORAL thyroid  Most patients have improved
hormone to suppress thyroid-stimulating hormone symptoms in 2 weeks and become
(TSH) and radioactive iodine (RAI) ablation followed euthyroid in about 6 weeks.
by hormone replacement.  IMPORTANT SIDE EFFECT OF PTU:
AGRANULOCYTOSIS
Ectopic thyroid  Propranolol is the most commonly
- Normal thyroid tissue in aberrant locations (esophagus, prescribed medication in doses of about 20 to
trachea and anterior mediastinum) 40 mg four times daily for control of
adrenergic symptoms
Pyramidal lobe  RAI: most often used in older patients with small
- The distal end of the atrophied thyroglossal duct that or moderate-sized goiters, those who have
connects to the thyroid, projecting up to the isthmus, relapsed after medical or surgical therapy, and
lying just to the left or right of the midline. those in whom antithyroid drugs or surgery are
contraindicated.
Benign thyroid disorders  Absolute CI: women who are pregnant or
breastfeeding
Hyperthyroidism  Relative contraindications:
 young patients (i.e., especially children
Grave’s disease and adolescents)
- autoimmune disease with a strong familial
 those with thyroid nodules
predisposition, female preponderance (5:1), and
 those with ophthalmopathy
peak incidence between the ages of 40 to 60 years.
 Surgery:
- Most common cause of hyperthyroidism in North
 Patients should be rendered euthyroid before
America
operation
- characterized by thyrotoxicosis, diffuse goiter, and
 Lugol's iodide solution or saturated
extrathyroidal conditions including ophthalmopathy,
potassium iodide generally is administered
dermopathy (pretibial myxedema), thyroid acropachy
beginning 7 to 10 days preoperatively (three
- hallmark: thyroid-stimulating antibodies stimulate
drops twice daily) to reduce vascularity of
the thyrocytes to grow and synthesize excess
the gland and decrease the risk of
thyroid hormone
precipitating thyroid storm.
- associated with other autoimmune conditions (ex. type I
 Indications for Total or near-total
DM, Addison's disease, pernicious anemia, and
thyroidectomy: Patients with coexistent
myasthenia gravis)
thyroid cancer, and those who refuse RAI
- Macroscopic appearance: diffusely and smoothly
therapy or have severe ophthalmopathy or
enlarged, increase in vascularity
have life-threatening reactions to antithyroid
- Microscopic appearance: hyperplastic gland, minimal
medications (vasculitis, agranulocytosis, or
colloid present
liver failure)
- Clinical features:
 Hyperthyroid Sx: heat intolerance, ↑sweating,
↑thirst, ↑ weight loss despite adequate caloric
intake  REMEMBER: a
 adrenergic excess: palpitations, nervousness,
fatigue, emotional lability, hyperkinesis, and How would you know if there is an undiagnosed
tremors hyperthyroid problem intraoperatively?
 most common GI symptom: diarrhea
 can also develop amenorrhea, decreased fertility, Increased vascularity  increased bleeding in a sedated
and an increased incidence of miscarriages patient 
- PE: facial flushing, warm & moist skin, Tachycardia,
atrial fibrillation, fine tremor, muscle wasting, and
proximal muscle group weakness with hyperactive Toxic multinodular goiter
tendon reflexes - Symptoms and signs of hyperthyroidism are similar to
Graves' disease, but extrathyroidal manifestations
 50% of patients ophthalmopathy
are absent
 lid lag (von Graefe's sign)
- Possible presence of cervical compressive symptoms
 spasm of the upper eyelid
- Diagnosis:
 revealing the sclera above the corneoscleral
 suppressed TSH level and elevated free T4 or T3
limbus (Dalrymple's sign)
levels.
 prominent stare
 RAI uptake also is increased, showing multiple
 1 to 2% of patients  dermopathy (deposition of nodules with increased uptake
glycosaminoglycans leading to thickened skin in  Treatment: subtotal thyroidectomy
the pretibial region and dorsum of the foot)
- Diagnostic: suppressed TSH with or without an Toxic adenoma (Plummer’s disease)
elevated free T4 or T3 level. - Hyperthyroidism from a single hyperfunctioning nodule
- typically occurs in younger patients
TOPNOTCH MEDICAL BOARD PREP SURGERY SUPPLEMENT HANDOUT Page 49 of 83
For inquiries visit www.topnotchboardprep.com.ph or email us at topnotchmedicalboardprep@gmail.com
TOPNOTCH MEDICAL BOARD PREP SURGERY SUPPLEMENT HANDOUT - Jules Lopez,MD-MBA,Teddy Carpio,MD-MBA
For inquiries visit www.topnotchboardprep.com.ph or email us at topnotchmedicalboardprep@gmail.com
- PE: solitary thyroid nodule without palpable thyroid of compression, including dysphagia, dyspnea,
tissue on the contralateral side choking, and hoarseness.
- RAI: "hot" nodule - Can result to hypothyroidism
- rarely malignant. - Associated with retroperitoneal fibrosis and sclerosing
- Tx: Surgery (lobectomy and isthmusectomy on the mediastinitis
affected side) is preferred to treat young patients and - PE: hard, "woody" thyroid gland with fixation to
those with larger nodules. surrounding tissues.
- Tx: surgery
Thyroid storm  Goal of surgery: to decompress the trachea by
- hyperthyroidism + fever, central nervous system wedge excision of the thyroid isthmus and to make
agitation or depression, cardiovascular dysfunction due a tissue diagnosis
to infection, surgery, trauma or amiodarone
administration. Solitary thyroid nodule
- Tx: ICU, Beta blockers, Oxygen supplementation, Fever - History: time of onset ( usually slow and indolent),
reduction, fluids, hemodynamic support, PTU, change in size, and associated symptoms such as pain,
Corticosteroids (to prevent adrenal exhaustion and dysphagia, dyspnea, choking, hoarseness (secondary to
block hepatic thyroid hormone conversion) malignant involvement of the RLNs)
- Risk factors for malignancy:
Acute (suppurative) thyroiditis 1. exposure to ionizing radiation
- more common in children and often is preceded by an 2. (+) FH of thyroid and other malignancies
upper respiratory tract infection or otitis media. associated with thyroid cancer
- It is characterized by severe neck pain radiating to the 3. Men > women
jaws or ear, fever, chills, odynophagia, and dysphonia. 4. Children > adults
- Complications: systemic sepsis, tracheal or esophageal - PE: nodules that are hard, gritty, or fixed to surrounding
rupture, jugular vein thrombosis, laryngeal chondritis, structures (if malignant)
and perichondritis or sympathetic trunk paralysis - Diagnosis:
- Streptococcus and anaerobes account for about 70%
of cases Figure 31. Management of a solitary thyroid nodule 
- Diagnosis: leukocytosis on blood tests and FNAB for
Gram's stain, culture, and cytology.
- Tx: parenteral antibiotics & drainage of abscesses.

Subacute (de quervain’s) thyroiditis


- strong association with the HLA-B35 haplotype.
- Self-limiting painful thyroiditis most commonly occurs
in 30- to 40-year-old women
- characterized by the sudden or gradual onset of neck
pain, which may radiate toward the mandible or ear.
- History of a preceding upper respiratory tract
infection often can be elicited.
- The gland is enlarged, exquisitely tender, and firm.
- Diagnosis: TSH is decreased, and Tg, T4 , and T3 levels
are elevated (during the early phase)
- tx: symptomatic; NSAIDs are used for pain relief
(steroids may be indicated in more severe cases)

Hashimoto’s thyroiditis (chronic thyroiditis)


- autoimmune process leads to destruction of thyrocytes
by autoantibodies, which lead to complement fixation  FNAB: single most important test in the
and killing by natural killer cells evaluation of thyroid masses; 1st diagnostic test
- Antibodies directed against three main antigens—Tg ordered in a patient with a solitary thyroid
(60%), TPO (95%), the TSH-R (60%), and, less nodule
commonly, to the sodium/iodine symporter (25%)  Results: benign – cysts & colloid nodules
- more common in women (male:female ratio 1:10 to 20 ) (65%), suspicious – follicular or hurthle cell
between the ages of 30 and 50 years old. neoplasms (20%), malignant (5%), and
- The most common presentation is that of a nondiagnostic (10%)
minimally or moderately enlarged firm granular  false-positive results is about 1%
gland discovered on routine PE or the awareness of  false-negative results occur in approximately
a painless anterior neck mass 3%
- Gross appearance: mildly enlarged, pale, gray-tan cut  a negative FNAB does not rule out CA
surface that is granular, nodular, and firm.  if suspicious result, the diagnosis of
- microscopic examination: the gland is diffusely malignancy relies on demonstrating
infiltrated by small lymphocytes and plasma cells and capsular or vascular invasion, features
occasionally shows well-developed germinal centers, that cannot be determined via FNAB.
follicles are lined by Hürthle or Askanazy cells  RAI scan:
- Dx: elevated TSH and the presence of thyroid  Single, cold, solid nodule  malignant
autoantibodies usually confirm the diagnosis.  Multiply, Hot, cystic  benign
- Tx: Thyroid hormone replacement therapy or surgery  Labs:
(if with compressive symptoms)  TSH: expect euthyroid
 Tg levels: useful for patients who have
Reidel’s thyroiditis undergone total thyroidectomy for thyroid
- characterized by the replacement of all or part of the cancer & for serial evaluation of patients
thyroid parenchyma by fibrous tissue undergoing nonoperative management of
- primary autoimmune etiology thyroid nodules.
- occurs predominantly in women between the ages of 30  Serum calcitonin: obtained in patients with
to 60 years old. MTC or a family history of MTC or MEN2
- presents as a painless, hard anterior neck mass, which  RET oncogene mutations: All patients with
progresses over weeks to years to produce symptoms MTC should be tested for RET oncogene
mutations and have a 24-hour urine collection
TOPNOTCH MEDICAL BOARD PREP SURGERY SUPPLEMENT HANDOUT Page 50 of 83
For inquiries visit www.topnotchboardprep.com.ph or email us at topnotchmedicalboardprep@gmail.com
TOPNOTCH MEDICAL BOARD PREP SURGERY SUPPLEMENT HANDOUT - Jules Lopez,MD-MBA,Teddy Carpio,MD-MBA
For inquiries visit www.topnotchboardprep.com.ph or email us at topnotchmedicalboardprep@gmail.com
with measurement of levels of - In <1% of cases, follicular cancers may be
vanillylmandelic acid (VMA), metanephrine, hyperfunctioning, leading patients to present with signs
and catecholamine to rule out a coexisting and symptoms of thyrotoxicosis.
pheochromocytoma. - Spread via hematogenous route, hence their spread
 Ultrasound: helpful for detecting nonpalpable is more distant, than regional
thyroid nodules, differentiating solid from cystic - Most common site of distant metastasis: lung &
nodules, and identifying adjacent bone
lymphadenopathy - Diagnosis:
- Tx:  FNAB shows follicular type  must do lobectomy
 Malignant tumors are treated by thyroidectomy to demonstrate capsular or vascular invasion
 Simple thyroid cysts resolve with aspiration; if (criteria for malignancy)
persists after 3 attempts at aspiration  unilateral - Microscopically: follicles are present, but the lumen may
thyroid lobectomy is recommended. be devoid of colloid.
 Lobectomy is recommended for cysts >4 cm in - Tx:
diameter or complex cysts with solid and cystic  If follicular lesion  thyroid lobectomy +
components isthmusectomy because at least 80% of these
 Colloid nodule  observe with serial ultrasound patients will have benign adenomas.
and Tg measurements.  older patients with follicular lesions >4 cm: total
thyroidectomy
Thyroid cancer  if (+) thyroid CA: do Total thyroidectomy
 (+) cervical node mets: MDRD
Papillary CA  mortality from follicular thyroid cancer is
- 80% of all thyroid malignancies in iodine-sufficient approximately 15% at 10 years and 30% at 20
areas years.
- predominant thyroid cancer in children and  Poor long-term prognosis: age over 50 years
individuals exposed to external radiation. old at presentation, tumor size >4 cm, higher
- occurs more often in women, 2:1 tumor grade, marked vascular invasion,
- symptoms of locally advanced disease: Dysphagia, extrathyroidal invasion, and distant
dyspnea, and dysphonia metastases at the time of diagnosis.
- Diagnosis:
 FNAB of the thyroid mass or lymph node. Hurtle cell CA
 Complete neck UTZ: to evaluate the contralateral - account for approximately 3% of all thyroid
lobe and for LN metastases in the central and malignancies
lateral neck compartments. - considered to be a subtype of follicular thyroid
- The most common sites are lungs, followed by bone, cancer.
liver, and brain. - Cannot be diagnosed by FNAB.
- Spread via lymphatic route - Microscopically: hurthle cells (variable enlargement,
- Gross appearance: hard and whitish and remain flat on hyperchromatic nuclei and granular cytoplasm)
sectioning with a blade, macroscopic calcification, - Difference from follicular CA:
necrosis, or cystic change may be apparent.  multifocal
- Microscopically:  bilateral (about 30%)
 papillary projections -a mixed pattern of papillary  usually do not take up RAI (about 5%)
and follicular structures  more likely to metastasize to local nodes (25%)
 pure follicular pattern (follicular variant).  associated with a higher mortality rate (about 20%
 Cells are cuboidal with pale, abundant at 10 years)
cytoplasm, large nuclei that may demonstrate  higher recurrence rate
"grooving," and intranuclear cytoplasmic - Same management with follicular neoplasms
- If (+) for hurthle malignancy: perform total
inclusions (Orphan Annie nuclei)
thyroidectomy + routine central neck (level 6) node
 Psammoma bodies: microscopic, calcified removal or MDRD when lateral neck nodes are palpable.
deposits representing clumps of sloughed cells,
also may be present Medullary thyroid CA (MTC)
 Multifocality: associated with an increased risk of - accounts for about 5% of thyroid malignancies
cervical nodal metastases - arises from the parafollicular or C cells of the thyroid
 Other variants: tall cell, insular, columnar, diffuse usually located superolaterally in the thyroid lobes
sclerosing, clear cell, trabecular, and poorly (usual site of MTC)
differentiated types. - female-to-male ratio is 1.5:1
 are generally associated with a worse - Most patients present between 50 and 60 years old
prognosis. - Most MTCs occur sporadically.
- Tx:  Occur singly
 If less than 1.5 cm: lobectomy + isthmusectomy  unilateral (80%)
 If multicentric: near total or total thyroidectomy  no familial predisposition
 (+) cervical node mets: MDRD - approximately 25% occur within the spectrum familial
 patients with papillary thyroid CA have an MTC - MEN2A (pheochromocytoma + parathyroid
excellent prognosis with a >95% 10-year hyperplasia), and MEN2B (pheochromocytoma +
survival rate. neuromas)
 Age is the most important prognostic factor  due to germline mutations in the RET proto-
in determining long term survival oncogene
 encodes for tyrosine-kinase receptor in the
Follicular CA cell membrane
- account for 10% of thyroid cancers  RET protein is expressed in tissues derived
- occur more commonly in iodine-deficient areas. from embryonic nervous and excretory
- Women have a higher incidence of follicular cancer, systems
with a female-to-male ratio of 3:1  present at a younger age
- usually present as solitary thyroid nodules, occasionally  multicentric
with a history of rapid size increase, and long-standing  (+) C cell hyperplasia: premalignant lesion
goiter. - clinical features:

TOPNOTCH MEDICAL BOARD PREP SURGERY SUPPLEMENT HANDOUT Page 51 of 83


For inquiries visit www.topnotchboardprep.com.ph or email us at topnotchmedicalboardprep@gmail.com
TOPNOTCH MEDICAL BOARD PREP SURGERY SUPPLEMENT HANDOUT - Jules Lopez,MD-MBA,Teddy Carpio,MD-MBA
For inquiries visit www.topnotchboardprep.com.ph or email us at topnotchmedicalboardprep@gmail.com
 present with a neck mass that may be associated - present with a rapidly enlarging neck mass that is often
with palpable cervical lymphadenopathy (15 to painless.
20%). - may present with acute respiratory distress.
 Pain or aching is common - Tx:
 dysphagia, dyspnea, or dysphonia – already  CT(CHOP—cyclophosphamide, doxorubicin,
invasive vincristine, and prednisone) + RT
 diarrhea – indicates metastatic disease (due to  Thyroidectomy and nodal resection: for alleviation
increased intestinal motility and impaired of airway obstructive symptoms who do not
intestinal water and electrolyte flux) respond quickly to the above regimens or who
 2 to 4% of patients develop Cushing's syndrome as have completed the regimen before diagnosis.
a result of ectopic production of - The overall 5-year survival rate is about 50%; patients
adrenocorticotropic hormone (ACTH) with extrathyroidal disease have markedly lower
- tumor markers: calcitonin (diagnostic, most sensitive survival rates.
tumor marker), CEA (better predictor of prognosis),
calcitonin gene–related peptide, histaminadases, Thyroid surgeries:
prostaglandins E2 and F2 & serotonin.
- Microscopically: Total thyroidectomy: dissection and remonal of all visible
 Heterogenous thyroid tissue bilaterally, which usually reveals the entrance of
 Cells are polygonal or spindle shaped the recurrent laryngeal nerve as they enter the ligament of berry
 infiltrating neoplastic cells separated by collagen
and amyloid Near total thyroidectomy: complete hemithyroidectomy and
 presence of amyloid is a diagnostic finding isthmusectomy; most of the contralateral side is removed but a
- Diagnosis: remnant is left to prevent damage to parathyroid glands
 history, physical examination, raised serum
calcitonin, or CEA levels, and FNAB cytology of the Subtotal thyroidectomy: removes all visible thyroid tissue
thyroid mass except for a rim of thyroid tissue bilaterally to ensure
 all new patients with MTC should be screened parathyroid viability and avoids damage to the recurrent
for RET point mutations, pheochromocytoma, laryngeal nerve
and HPT. If (+) carrier, perform total
thyroidectomy REVIEW QUESTIONS
- Tx:
 Total thyroidectomy + bilateral central node 1. Regarding salivary gland tumors, which one of the
dissection (level 6) because of high incidence of following statements is true?
multicentricity
 If (+) pheochromocytoma  manage this 1st a. The majority of malignant salivary gland
 If with palpable cervical nodes or involved central tumors arise in the parotid gland
neck nodes: ipsilateral or bilateral MDRD b. Most parotid neoplasms are malignant
 If tumors >1 cm, ipsilateral prophylactic modified c. Fine needle aspiration biopsy is recommended
radical neck dissection is recommended because for all suspected salivary gland malignancies
>60% of these patients have nodal metastases.  d. Minor salivary gland tumors occur most
commonly in the floor of the mouth
if ipsilateral nodes are positive  do contralateral
node dissection Answer: A
- Postoperative Follow-Up: annual measurements of The likelihood of a given tumor’s being malignant is
calcitonin and CEA levels lowest in the parotid gland (approximately 20%),
- Prognosis: followed by the submandibular salivary gland
 10-year survival rate is approximately 80% but (approximately 50%) and sublingual glands (nearly
decreases to 45% in patients with lymph node 100%). However, because more than 75% of all salivary
involvement. gland tumors occur in the parotid gland, the parotid
 best in patients with non-MEN familial MTC, gland accounts for the majority of the malignant salivary
followed by those with MEN2A gland tumors. The diagnostic evaluation of a salivary
 Prognosis is the worst (survival 35% at 10 gland mass depends on the location and clinical
years) in patients with MEN2B. scenario. FNAB is not indicated for all parotid tumors,
since a tissue diagnosis does not change the treatment
Anaplastic CA plan for a patient with a small, mobile mass clearly
- approximately 1% of all thyroid malignancies within the gland. When the location is uncertain, the
- the most aggressive of thyroid malignancies history suggests the possibility of metastatic disease, or
- Women are more commonly affected the tumor size or location indicates a difficult facial
- present in the 7th & 8th decade of life nerve dissection, FNAB may be helpful. Biopsy, usually a
- Clinical features: punch or excisional biopsy, should be performed for
 Presents as a long-standing neck mass, which suspected minor salivary gland tumors, the most
rapidly enlarges and may be painful. Associated common site of which is the palate, usually at the
symptoms such as dysphonia, dysphagia, and junction of the hard and aoft palate. Like FNAB, imaging
dyspnea are common. studies (CT or MRI) should be used when they are likely
 Lymph nodes usually are palpable at presentation. to augment the clinical assessment of staging and affect
- Gross appearance: firm and whitish in appearance. treatment planning.
- Microscopically: characteristic giant and
multinucleated cells. with marked heterogeneity are 2. A 40 year-old woman comes to the clinician’s office
seen (spindle shaped, polygonal, or large, with a thyroid mass, which is confirmed on FNA
multinucleated cells) and UTZ to be unilateral, 3.2 cm follicular neoplasm.
- Tx: if resectable mass  surgery will only give small She has been completely asymptomatic. What will
improvement in survival the next intervention be?
- Prognosis: 6 months
a. Total thyroidectomy
Thyroid Lymphoma b. Hemithyroidectomy or isthmusectomy
- <1% of thyroid malignancies c. Excisional biopsy
- Most common: non-Hodgkin's B-cell type. d. Core-needle biopsy
- develop in patients with chronic lymphocytic e. Thyroid suppression via T3 or T4 analogues
thyroiditis.
TOPNOTCH MEDICAL BOARD PREP SURGERY SUPPLEMENT HANDOUT Page 52 of 83
For inquiries visit www.topnotchboardprep.com.ph or email us at topnotchmedicalboardprep@gmail.com
TOPNOTCH MEDICAL BOARD PREP SURGERY SUPPLEMENT HANDOUT - Jules Lopez,MD-MBA,Teddy Carpio,MD-MBA
For inquiries visit www.topnotchboardprep.com.ph or email us at topnotchmedicalboardprep@gmail.com
impregnanted marshmallow, barium-soaked bread or
Answer: A, B barium hamburger
The presence of a follicular neoplasm as confirmed  will bring out the functional disturbance in the
by FNA manadtes further evaluation, since FNA esophageal transport that can be missed when
does not provide enough information about tissie liquid barium is used.
architecture to differentiate between a benign
follicular adenoma and a follicular carcinoma. 2. endoscopic evaluation
Vascular or capsular invasion confirms the - endoscopy is indicated in patients complaining of
presence of carcinoma. The management of small, dysphagia even with a normal radiographic study
unilateral follicular lesions is controversial (total
versus hemi-thyroidectomy with frozen section). Tests to detect functional abnormalities
Hwoever, lesions larger than 4 cm should be 1. manometry
treated with total thyroidectomy, since - indicated when a motor abnormality of the esophagus is
multicentricity becomes more common as tumor considered on the basis of complaints (dysphagia,
size increases. Total thyroidectomy also facilitates odonyphagia, or noncardiac chest pain) and barium
the effectiveness of postoperative radioactive swallow and endoscopy does not show a structural
iodine, since no residual thyroid tissue remains to abnormality
serve as a sink for the radioisotope. - essential tool in preoperative evaluation of patients
before antireflux surgery
3. During a total thyroidectomy for papillary cancer,
the clinician observes an intact recurrent laryngeal Tests to detect increased exposure to gastric juice
nerve on the right side and a completely transected 1. 24 hour ambulatory pH monitoring
nerve on the left, with both ends in view. What - most direct method of measuring increased esophageal
should management of this patient at this point exposure to gastric juice (not reflux)
entail? - sensitivity and specificity of 96%
- gold standard for the dioagnosis of GERD
a. Complete the operation and evaluate the vocal
cords postoperatively via flexible 2. radiographic exposure of gastroesophageal reflux
bronchoscopy - radiographic demonstration of spontaneous
b. Perform intraoperative flexible bronchoscopy regurgitation of barium into the esophagus in the
to evaluate vocal cords upright position is a reliable indicator that reflux is
c. Repair the nerve using 8.0 monofilament present
sutures - note: failure to see this does not indicate absence of
d. None of the above disease

Answer: D B.
GERD
If the recurrent laryngeal nerve is injured or -
Clinical features:
transected during an otherwise uncomplicated 1. Heartburn: substernal burning-type discomfort,
operation, it should be repaired using loupes or an beginning in the epigastrium and radiating upward.
operating microscope to visualize the field, and 8.0 -It is often aggravated by meals, spicy or fatty
or 9.0 monofilament sutures to anstamose the cut foods, chocolate, alcohol, and coffee
ends of the nerves. There is no role for flexible -worse in the supine position
bronchoscopy either intraoperatively or 1. Regurgitation: effortless return of acid or bitter
postoperatively unless there is uncertainty about gastric contents into the chest, pharynx, or mouth;
the injury or the function of the contralateral nerve. highly suggestive of foregut pathology
-severe at night when supine or when bending over
-secondary to either an incompetent GEJ
ESOPHAGUS -explains the associated pulmonary symptoms,
including cough, hoarseness, asthma, and recurrent
pneumonia.
A. Diagnostic tests for esophageal function
2. Dysphagia: most specific symptom of foregut
B. GERD
disease; sensation of difficulty in the passage of
C. Diaphragmatic hernia
food from the mouth to the stomach
D. Schatzki’s ring
3. Chest pain
E. Scleroderma of esophagus
- primary cause of GERD: permanent attenuation of the
F. Zenker’s diverticulum
collar sling musculature, with a resultant opening of
G. Achalasia
the gastric cardia and loss of the high-pressure zone
H. Diffuse and segmental esophageal spasm
as measured with esophageal manometry
I. Nutcracker esophagus
 characteristics of a defective sphincter
J. Hypertensive LES
1. LES with a mean resting pressure of less than 6
K. Esophageal diverticulum
mmHg
L. Esophageal perforation
2. overall sphincter length of <2 cm
M. Mallory weiss syndrome
3. intra-abdominal sphincter length of <1 cm
N. Caustic injury
(most important consideration affecting the
O. Esophageal carcinoma
competence of the GE jxn)
- diagnosis:
 24 hour pH monitoring (gold standard): most
A. DIAGNOSTIC TESTS FOR ESOPHAGEAL FUNCTION sensitive for the detection of reflux
 Endoscopic examination: assessing anatomic
Tests to detect structural abnormalities damage produced by reflux (esophagitis, ulceration
and strictures) & for ruling out CA
1. Barium swallow  Grading of esophagitis
- 1st diagnostic test in patients with suspected esophageal Grade I: small circular nonconfluent erosions
disease (with full assessment of stomach and Grade II: presence of linear erosions lined with
duodenum) granulation tissue that bleeds easily when
- can reveal anatomic problems touched
- if patient complains of dysphagia and no obstructing Grade II: linear erosions coaslesce into a
lesion seen in barium swallow  use a barium- circumferential loss of the epithelium;
cobblestone mucosa
TOPNOTCH MEDICAL BOARD PREP SURGERY SUPPLEMENT HANDOUT Page 53 of 83
For inquiries visit www.topnotchboardprep.com.ph or email us at topnotchmedicalboardprep@gmail.com
TOPNOTCH MEDICAL BOARD PREP SURGERY SUPPLEMENT HANDOUT - Jules Lopez,MD-MBA,Teddy Carpio,MD-MBA
For inquiries visit www.topnotchboardprep.com.ph or email us at topnotchmedicalboardprep@gmail.com
Grade IV: (+) stricture  MUST KNOW a
Principles of surgical therapy in reflux disease 
**absence of esophagitis above a stricture
suggest chemical induced injury or neoplasm 1. the operation should restore the pressure of the distal
as a cause. esophageal sphincter to a level twice the resting gastric
i. pressure
 Manometric studies: rule out motility DO 2. the operation should place an adequate length of the
- Complications: distal esophageal sphincter in the positive-pressure
1. Metaplastic (Barrett's Esophagus) environment of the abdomen by a method that ensures
 condition whereby the tubular esophagus is its response to changes in intra-abdominal pressure
lined with columnar epithelium rather than 3. the operation should allow the reconstructed cardia to
squamous epithelium relax on deglutition
 occurs in 10 to 15% of patients with GERD 4. the fundoplication should not increase the resistance of
 end stage of natural Hx of GERD the relaxed sphincter to a level that exceeds the
 hallmark: presence of intestinal goblet peristaltic power of the body of the esophagus
cells in esophageal epithelium (intestinal 5. the operation should ensure that the fundoplication can
metaplasia) be placed in the abdomen without undue tension, and
 endoscopically: difficulty visualizing the maintained there by approximating the crura of the
squamocolumnar junction at its normal diaphragm above the repair
location & appearance of redder mucosa than
normally seen in lower esophagus
 earliest sign for malignant degeneration: C. DIAPHRAGMATIC HERNIA
severe dysplasia or intramucosal - Types:
adenocarcinoma 1. type I (sliding hernia)
 antireflux surgery is an excellent means of  upward dislocation of the cardia in the
long-term control for most patients posterior mediastinum
 one third of all patients with BE present with  the phrenoesophageal ligament is
malignancy stretched but intact
o should undergo surveillance with  most common
biopsy every 2 years  can evolve into a type III hernia
o if (+) low grade dysplasia, increase 2. type II (rolling or paraesophageal or giant
frequency to 6 months hiatal hernia)
2. Esophageal Adenocarcinoma  upward dislocation of the gastric fundus
 Most important etiologic factor in its alongside a normally positioned cardia
development is barrett’s esophagus  defect in the phrenoesophageal membrane
3. Respiratory symptoms  rare
 LERD  more likely to occur in women (4:1)
 Adult-onset asthma 3. type III (the combined sliding-rolling or mixed
 Idiopathic pulmonary fibrosis hernia)
- Treatment:  upward dislocation of both the cardia and the
 Medical: gastric fundus; therefore
 Uncomplicated GERD: 12 weeks of empiric  the esophagogastric junction is in the
treatment of antacid mediastinum
 Persistent sx: PPIs or H2 antagonists 4. type IV: colon, herniates as well (in some
 A structurally defective LES is the most classifications)
important factor predicting failure of 5. intrathoracic abdomen
medical therapy  the end stage of type I and type II hernias
 They don’t respond to medical therapy occurs when the whole stomach migrates up
well; candidates for anti-reflux surgery into the chest by rotating 180° around its
 Lifestyle changes: elevate the head of the bed longitudinal axis, with the cardia and pylorus
during sleep; avoid tight-fitting clothing; eat small, as fixed points
frequent meals; avoid eating the nighttime meal - most common complications:
immediately prior to bedtime; and avoid alcohol,  occult GI bleeding from gastritis
coffee, chocolate, and peppermint (which are  ulceration in the herniated portion of the stomach
known to reduce resting LES pressure)  gastric volvulus (surgical emergency): or
 Surgical Borchardt’s triad of pain, nausea with inability to
 Nissen fundiplication: a abdominal or vomit and inability to pass NGT
thoracic approach using a 360 degree - Diagnosis:
circumferential wrap of the gastric fundus  Barium esophagogram: for diagnosis of
 Belsey operation: difficult to learn, performed paraesophageal hiatal hernia
through the chestm, involves placement of 2  Fiber-optic esophagoscopy
layers of placating structures between the  Detection of pouch lined with gastric rugal
gastric fundus and lower esophagus with folds lying 2 cm or more above the margins of
subsequent creation of 280 degree anterior the diaphragmatic crura (identified by having
gastric wrap and posterior approximation of the patient sniff)
the crura - Treatment: surgical
 Hill operation: approach is through the  Important principles
abdomen, posterior approximation of the  Reduce the hernia contents
crura followed by anchoring of the posterior  After reduction, excise the sac
and anterior aspects of the GEJ to the median  The use of mesh can reduce recurrence rates
arcuate ligament adjacent to the aorta, of hernia is > than 8 cm
creating a 180 degree gastric wrap
 Collis gastroplasty: esophageal lengthening D. SCHATZKI’S RING
procedure - thin submucosal circumferential ring in the lower
 Angelchik prosthesis: horshe shoe shape esophagus at the squamocolumnar junction, often
silastic device placed around the distal associated with a hiatal hernia.
esophagus, keeping this segment in the - probably an acquired lesion that can lead to stenosis
abdomen from chemical-induced injury by pill lodgment in the

TOPNOTCH MEDICAL BOARD PREP SURGERY SUPPLEMENT HANDOUT Page 54 of 83


For inquiries visit www.topnotchboardprep.com.ph or email us at topnotchmedicalboardprep@gmail.com
TOPNOTCH MEDICAL BOARD PREP SURGERY SUPPLEMENT HANDOUT - Jules Lopez,MD-MBA,Teddy Carpio,MD-MBA
For inquiries visit www.topnotchboardprep.com.ph or email us at topnotchmedicalboardprep@gmail.com
distal esophagus, or from reflux-induced injury to the  Manometric studies: frequent occurrence of
lower esophageal mucosa simultaneous waveforms and multipeaked
- Symptoms: brief episodes of dysphagia during hurried esophageal contractions, which may be of
ingestion of solid foods. abnormally high amplitude or long duration.
- Treatment options: dilation alone, dilation with  Esophagogram: corkscrew esophagus or
antireflux measures, antireflux procedure alone, pseudodiverticulosis
incision, and excision of the ring
I. NUTCRACKER ESOPHAGUS
E. SCLERODERMA OF ESOPHAGUS - Other name: supersqueezer esophagus
- Scleroderma is a systemic disease accompanied by - most common of the primary esophageal motility
esophageal abnormalities in approximately 80% of disorders
patients. - characterized by peristaltic esophageal contractions
- onset of the disease is usually in the third or fourth with peak amplitudes greater than two SDs above the
decade of life, occurring twice as frequently in normal values (up to 400 mmHg)
women as in men. - Treatment in these patients should be aimed at the
- Small vessel inflammation appears to be an treatment of GERD
initiating event, with subsequent perivascular
deposition of normal collagen, which may lead to J. HYPERTENSIVE LES
vascular compromise. - This disorder is characterized by an elevated basal
- Muscle ischemia due to perivascular compression has pressure of the LES with normal relaxation and normal
been suggested as a possible mechanism for the motility propulsion in the esophageal body.
abnormality in scleroderma. - Treatment: Myotomy of the LES may be indicated in
- predominant feature at GI tract: smooth muscle patients not responding to medical therapy or dilation.
atrophy.
- Diagnosis: K. ESOPHAGEAL DIVERTICULUM
 Manometrically: observation of normal peristalsis - Classification:
in the proximal striated esophagus, with absent  Location: proximal, mid, distal
peristalsis in the distal smooth muscle portion  Pathology
 Barium swallow: dilated, barium-filled esophagus,  Pulsion: motor DO
stomach, and duodenum, or a hiatal hernia with  Traction: inflammatory DO
distal esophageal stricture and proximal dilatation - Epiphrenic diverticula:
 terminal 3rd of the thoracic esophagus & are usually
F. ZENKER’S DIVERTICULUM found adjacent to the diaphragm
- most common esophageal diverticulum  associated with distal esophageal muscular
- classified as false diverticulum hypertrophy, esophageal motility abnormalities,
- clinical features: dysphagia associated with the and increased luminal pressure
spontaneous regurgitation of undigested food,  considered as "pulsion" diverticula
halitosis, weight loss, chronic aspiration and repetitive  classified as false diverticulum (pouch of mucosa
respiratory infection that is protruding in the wall of the esophagus)
- Midesophageal or traction diverticula
- due to weakness of the cricopharyngeal muscle 
 Classified as true diverticulum (composed of all
weakness at the Killian’s area layers of the esophageal wall)
- Diagnosis: Barium swallow (to exclude neoplasia or  noted in patients who had mediastinal LN
ulceration) involvement with tuberculosis, mediastinal
- Treatment: lymphadenopathy, such as pulmonary fungal
 Pharyngomyotomy: 2 cm or less infections (e.g., aspergillosis), lymphoma, or
 Diverticulectomy/diverticuopexy: >2cm sarcoid
G. ACHALASIA L. ESOPHAGEAL PERFORATION
- Characterized by complete absence of peristalsis in the - true emergency.
esophageal body and failure of LES relaxation - It most commonly occurs following diagnostic or
- Classic triad of symptoms: dysphagia, regurgitation therapeutic procedures (endoscopy).
and weight loss; also associated with nocturnal asthma - Boerhaave's syndrome: spontaneous perforation,
and foul smelling esophageal contents - Clinical features: chest pain (very striking and
- pathogenesis of achalasia is presumed to be a consistent symptom), fever, tachycardia, subcutaneous
neurogenic degeneration, which is either idiopathic or emphysema, dysphagia, dysnea
due to infection. - Diagnosis:
- Can be caused by T. cruzi which demonstrates  contrast esophagogram with water soluble (like
destruction of smooth muscle myenteric auerbach’s gastrografin) medium: (+) extravasation
plexus (diagnostic)
- Diagnosis:  chest xray: air or effusion in pleural space,
 Barium Esophagogram: dilated esophagus with a mediastinal or cervical emphysema
tapering or other wise known as "bird's beak" - treatment:
 Manometric studies: failure of the LES to relax,  the incidence of mortality is related to the time
progressive peristalsis in proximal esophagus (if interval between perforation and treatment; hence
late disease) the key to optimum management is early
 Has a 10% chance of developing carcinoma due to diagnosis.
prolonged mucosal irritation  The most favorable outcome is obtained following
- Tx: primary closure of the perforation within 24
 heller’s myotomy (surgical myotomy of the LES) hours, resulting in 80 to 90% survival.
 goal of surgery: relieve functional obstruction at  The most common location for the injury is the left
the LES lateral wall of the esophagus, just above the GEJ
- non-operative management
H. DIFFUSE AND SEGMENTAL ESOPHAGEAL SPASM  usually follows an injury occruing during dilation
- characterized by substernal chest pain and/or of esophageal strictures or pneumatic dilations of
dysphagia. achalasia
- The LES in patients with DES usually shows a normal  indications
resting pressure and relaxation on swallowing
- Diagnosis:
TOPNOTCH MEDICAL BOARD PREP SURGERY SUPPLEMENT HANDOUT Page 55 of 83
For inquiries visit www.topnotchboardprep.com.ph or email us at topnotchmedicalboardprep@gmail.com
TOPNOTCH MEDICAL BOARD PREP SURGERY SUPPLEMENT HANDOUT - Jules Lopez,MD-MBA,Teddy Carpio,MD-MBA
For inquiries visit www.topnotchboardprep.com.ph or email us at topnotchmedicalboardprep@gmail.com
1. barium swallow must show the perforation to  To lessen the chance of perforation, the scope
be contained within the mediastinum and should not be introduced beyond the proximal
drain well back into the esophagus esophageal lesion.
2. mild symptoms - Treatment:
3. minimal evidence of clinical sepsis
- approach: Figure 32. algorithm for acute caustic injury
 hyperalimentation
 antibiotics
 cimetidine: to decreased acid secretion, diminish
pepsin activity

M. MALLORY WEISS SYNDROME


- Longitudinal tear in the mucosa of the GE junction
- Characterized by acute upper GI bleeding caused by
forceful vomiting and/or retching
- Commonly seen in alcoholics
- arterial  massive
- mechanism: an acute increase in intra-abdominal
pressure against a closed glottis in a patient with a
hiatal hernia.
- Diagnosis:
 requires a high index of suspicion (the pattern of
sudden upper GI bleeding following prolonged
vomiting or retching is indicative).
 Upper endoscopy: longitudinal fissures in the
mucosa of the herniated stomach as the source of
bleeding. O. ESOPHAGEAL CARCINOMA
- Treatment: - Squamous carcinoma accounts for the majority of
 bleeding will stop 90% of the time esophageal carcinomas worldwide.
spontaneously with nonoperative management. - Risk factors:
 Decompression  nitroso compounds in pickled vegetables and
 antiemetics smoked meats
 zinc & molybdenum deficiency
N. CAUSTIC INJURY  smoking (more squamous CA)
- Alkalies vs acids  alcohol consumption (more squamous CA)
 Alkalies dissolve tissue, and therefore  achalasia
penetrate more deeply (more serious)  lye strictures
 acids cause a coagulative necrosis that limits  tylosis (an autosomal dominant disorder
their penetration characterized by hyperkeratosis of the palms and
- The strength of esophageal contractions varies soles)
according to the level of the esophagus  human papillomavirus.
- weakest at the striated muscle–smooth muscle interface  Barrett’s esophagus (more adenocarcinoma)
 slower clearance  allowing caustic substances to - Most common presenting symptom: dysphagia
(already a late symptom)
remain in contact with the mucosa longer  explains
- Diagnosis:
why the esophagus is preferentially and more severely
affected at this level than in the lower portions.  barium esophagogram (if with lesion)  upper
- Phases of injury: endoscopy
 acute necrotic phase:  CT scan of chest and abdomen: delineate the tumor
 lasting 1 to 4 days after injury and detect distant pulmonary or hepatic metastasis
 coagulation of intracellular proteins results in - Characteristics based on tumor location and treatment:
cell necrosis  Cervical esophagus (proximal 1/3)
 living tissue surrounding the area of necrosis  Almost always squamous carcinoma
develops an intense inflammatory reaction.  Frequently unresectable because of early
 ulceration and granulation phase: invasion of larynx, great vessels or trachea
 3 to 5 days after injury  Tx: stereotactic radiation with concomitant
 Considered a quiescent period because chemotherapy
symptoms seem to disappear  thoracic esophagus (middle 1/3)
 the superficial necrotic tissue sloughs, leaving  almost always squamous carcinoma with LN
an ulcerated, acutely inflamed base, and metastasis
granulation tissue fills the defect left by the  tx: video assisted thoracic surgery (VATS) ±
sloughed mucosa. thoracotomy
 This phase lasts 10 to 12 days  distal 1/3 or near/at cardia
 period that the esophagus is the weakest  almost always adenocarcinoma
 cicatrization and scarring  tx: curative resection requires cervical division
 begins third week following injury. of esophagus + >50% proximal gastrectomy
 previously formed connective tissue begins to - Staging:
contract, resulting in narrowing of the Stage 0: in situ, high-grade dysplasia, no LN mets
esophagus Stage I: invaded lamina propria
 characterized by dysphagia Stage IIA: invaded esophageal wall but not surrounding
 It is during this period that efforts must be structures
made to reduce stricture formation. Stage IIB: LN (+); primary tumor has only invaded
- Clinical features: pain in the mouth and substernal submucosa or muscularis propria
region, hypersalivation, pain on swallowing, and Stage III: invaded the adventitia and surrounding
dysphagia, fever (strongly correlated with the structures (pericardium, pleura and aorta)
presence of an esophageal lesion) Stage IV: (+) metastasis
- Diagnosis: early esophagoscopy is advocated to - Clinical factors that indicate advanced stage (and
establish the presence of an esophageal injury therefore exclude surgery for curative intent):
 Horner’s syndrome
TOPNOTCH MEDICAL BOARD PREP SURGERY SUPPLEMENT HANDOUT Page 56 of 83
For inquiries visit www.topnotchboardprep.com.ph or email us at topnotchmedicalboardprep@gmail.com
TOPNOTCH MEDICAL BOARD PREP SURGERY SUPPLEMENT HANDOUT - Jules Lopez,MD-MBA,Teddy Carpio,MD-MBA
For inquiries visit www.topnotchboardprep.com.ph or email us at topnotchmedicalboardprep@gmail.com
 Persistent spinal pain Chest pain, fever, tachycardia, subcutaneous
 Paralysis of diaphragm emphysema, dysphagia and dyspnea are typical of
 Fistula formation esophageal perforation. Perforation may result
 Malignant pleural effusion from iatrogenic operations, external trauma,
- Treatment: primary esophageal disease or postemetic
 Surgery: (“spontaneous”) esophageal hypertension. The
 Ivor lewis procedure: incidence of mortality from esophageal perforation
 primarily for middle esophageal lesion; all is clearly related to the time interval between
LNs are removed en bloc with the lesser perforation and definitive treatment. Whenever
curvature of the stomach perforation is suspected, a contrast study should be
 most radical  highest number of performed with water-soluble contrast material.
complication rate However, if this study does not demonstrate the
 Left thoracoabdominal approach perforation, it should be repeated with barium.
 excellent exposure of distal esophagus Although barium is contraindicated in the presence
 Transhiatal blunt resection: resection of the of colonic injuries because of the harmful effects of
thoracic esophagus from abdomen with feces and barium, it does not cause a problem in the
subsequent pull-up of stomach and chest. Barium is more accurate than water in
esophagogastric anastomosis in the neck detecting esophageal leakage. Contrast studies are
 Goes against the principle of en-bloc important not just for verifying esophageal rupture
resection of cancer surgery but also for documenting the level of injury, which
 Minimized morbidity and mortality has important implications for treatment.
compared to the other procedures

REVIEW QUESTIONS a STOMACH

1. a 4 year old child is brought to the ER 15 minutes A. Anatomy


after accidentally ingesting a drain cleaner. The B. Diagnostic tests for stomach
child exhibits a hoarse voice and is stridorous. C. Peptic ulcer disease
Which of the following apply? D. Zollinger-Ellison syndrome
E. Gastritis
a. Laryngeal ulceration F. stress ulcer
b. Instillation of vinegar into the stomach G. Malignant neoplasms of the stomach
c. Immediate fiberoptic endoscopy H. Benign gastric neoplasms: polyps
d. tracheostomy I. Gastric volvulus
J. Postgastrectomy problems
Answer: A & D
This is a case of caustic ingestion. Since the child
already exhibits laryngeal and epiglottic edema,
preservation of the airway must be the priority. A. ANATOMY
Therefore, endoscopy is deferred. - Stomach is composed of 3 smooth muscle layers:
1. Outler longitudinal layer –greater and lesser
2. A 50 year old healthy man is brought to the ER with curvatures of the stomach
retching followed by hematemesis. 2. Middle circular – pylorus
3. Inner oblique
a. Treatment is by balloon tamponade - Majority of parietal cells are in the Body of the
b. Bleeding often stops spontaneously stomach
c. It is not caused by forceful vomiting - Largest artery to the stomach is the left gastric artery
d. There is air in the mediastinum (from the celiac trunk)
e. Diagnosis is not made by endoscopic
examination
- Gastric contraction is via the vagus nerve (primarily
due to parasympathetic fibers) 
Answer: B 1. The vagus nerves forms LARP (left:anterior &
This is a case of Mallory-weis tear. The mechanism right:posterior) at the esophageal hiatus as it
is similar to boerhave syndrome (postemetic descends from the mediastinum
esophageal rupture) in which there is associated 2. Anterior branch of vagus: nerves of Laterjet
perforation and vomiting against a closed cardia. It  they send segmental branches to the body of
is diagnosed by endoscopic examination, and the the stomach before they terminate near the
bleeding usually stops spontaneously. Because the angularis incisura as the "crow's foot”
bleeding is arterial, a pressure tamponade (i.e. 3. Posterior branch: Criminal nerve of Grassi
Sengstaken-blakemore tube) does not help and may (posterior fundus)
lead to esophageal disruption. If bleeding does not  easily missed during truncal or highly
stop, gastrotomy and oversewing of the bleeding selective vagotomy (HSV).
point is the proper therapy, although nonsurgical - Gastric relaxation is due to CCK, distention of
alternatives, such as endoscopic injection of duodenum and presence of glucose in the duodenum
epinephrine and cautery have been attempted.
 MUST KNOW a
3. After diagnostic esophagoscopy, a patient
complains of odynophagia and chest pain, but Atonic gastritis and abnormal distention and failure to
results of water-soluble contrast swallow are empty of the stomach can occur in the postoperative patient
negative. Which of the following apply? due to electrolyte disturbances, hyperglycemia and uremia.

a. Discharge the patient if ECG is normal. Gastric ulcers located in the PYLORUS are associated with
b. Use of barium in the chest is devastating increased gastric production (see below – Type II & III
c. Esophageal manometry should be performed ulcers)
immediately
d. Repeat swallow with barium
Answer: D

TOPNOTCH MEDICAL BOARD PREP SURGERY SUPPLEMENT HANDOUT Page 57 of 83


For inquiries visit www.topnotchboardprep.com.ph or email us at topnotchmedicalboardprep@gmail.com
TOPNOTCH MEDICAL BOARD PREP SURGERY SUPPLEMENT HANDOUT - Jules Lopez,MD-MBA,Teddy Carpio,MD-MBA
For inquiries visit www.topnotchboardprep.com.ph or email us at topnotchmedicalboardprep@gmail.com
B. DIAGNOSTIC TEST FOR STOMACH  H. Pylori: associated with both gastric and
duodenal ulcer but is a higher predictor of
1. EGD duonal ulcer formation
- patients with one or more of the alarm symptoms must  NSAID – patients taking NSAID and/or aspirin need
undergo immediate upper endoscopy acid suppressing medication if any of the ff are
present: age over 60 yo, hx of PUD, concomitant
Table 53. Alarm symptoms steroid/anticogualant/high dose NSAIDs intake
Alarm symptoms that indicate the need for upper endoscopy  Smoking – largest positive predictor of risk (also
Weight loss with alcoholic drinking)
Recurrent vomiting
 Stress – both physiologic and psychologic stress
Dysphagia
Bleeding
 Others
Anemia  More common in Type A personality
 Sex: duodenal ulcer is twice more common in
- requires an 8 hour fasting males; same incidence between sexes for
- more sensitive than double contrast upper GI series gastric ulcer
- most serious complication: esophageal perforation  Blood type:
 Type O: duodenal ulcer
2. Radiologic tests  Type A: gastric ulcer
- Types of ulcer based on location and
Plain abdominal xray pathophysiology
- helpful in the diagnosis of gastric perforation  Duodenal ulcers patients have ↑ daytime and
(pneumoperitoneum) or delayed gastric emptying nocturnal acid secretion, ↑ BAO and MAO, ↑ gastric
(large air-fluid level) emptying compared to gastric ulcer patients
 Gastric ulcers patients have variable patterns of
Double contrast upper GI series secretion
- better then EGD in detecting the ff: diverticula, fistula,
tortuisity or stricture location, and size of hiatal hernia Figure 33. Modified Johnson classification of gastric ulcer

3. CT and MRI
- is part of routine staging work-up for most patients
with a malignant gastric tumor

4. Gastric secretory analysis


- maybe useful in the evaluation of patients with
hypergastrinemia, including Zollinger-Ellison
syndrome, patients with refractory ulcer or GERD or
recurrent ulcer after operation

5. Tests for Helicobacter pylori

Serologic test for H. pylori


- a positive test is a presumptive evidence of active
infection if the patient has never been treated for H.
pylori infection

Histologic examination of antral mucosal biopsy (with


special stains)
- gold standard for H. pylori

Urease breath test Type I: located near the angularis incisura on the lesser
- standard test for to confirm eradication of H. pylori curvature; usually have normal or decreased acid secretion; most
post-treatment common
- basis: the patient ingests urea labeled with Type II: same with type I but with an associated active or
nonradioactive 13C  labeled urea is acted upon by the quiescent duodenal ulcer; associated with normal or increased
urease present in H. pylori  converts urea into gastric acid secretion
ammonia and carbon dioxide  radiolabeled carbon Type III: prepyloric ulcer disease; associated with normal or
dioxide is excreted from lungs and is detected in increased gastric acid secretion
expired air. Type IV: occur near the GE junction, and acid secretion is normal
 or below normal
Type V: NSAID induced, can occur anywhere in the stomch
 MUST KNOW a
 MUST KNOW a
H.pylori has the enzyme urease, which converts urea into
ammonia and bicarbonate, thus creating an environment Curling ulcers: peptic ulcers formed after severe burn injury
around the bacteria that buffers the acid secreted by the stomach. Cushing’s ulcers: peptic ulcers formed after severe brain
damage

H. pylori fecal antigen test - Pathophysiology, Clinical manifestations, diagnosis


- sensitive and specific for active H. pylori infection and treatment
- can also be used to confirm cure
Table 54: Comparison between gastric vs duodenal ulcer
C. PEPTIC ULCER DISEASE Gastric ulcer Duodenal ulcer
- focal defects in the gastric or duodenal mucosa that Pathophysiology H.pylori, overuse of ↑ acid production &
NSAIDS & steroids H.pylori
extend into the submucosa or deeper
Clinical Sharp burning pain in Severe epigastric pain
- caused by an imbalance between mucosal defenses and manifestation epigastrium shortly 2-3 hours after eating;
acid/peptic injury. after eating; nausea, epigastric pain can
- Etiology vomiting and anorexia also awaken them

TOPNOTCH MEDICAL BOARD PREP SURGERY SUPPLEMENT HANDOUT Page 58 of 83


For inquiries visit www.topnotchboardprep.com.ph or email us at topnotchmedicalboardprep@gmail.com
TOPNOTCH MEDICAL BOARD PREP SURGERY SUPPLEMENT HANDOUT - Jules Lopez,MD-MBA,Teddy Carpio,MD-MBA
For inquiries visit www.topnotchboardprep.com.ph or email us at topnotchmedicalboardprep@gmail.com
from sleep
Diagnosis Endoscopy and biopsy Endoscopy, history, - Indications for surgery in PUD are bleeding, perforation,
(must for all gastric PE, test for H pylori obstruction and intractability or nonhealing.
ulcers to rule out
- Most patient undergoing operation for PUD have simple
cancer; test for
H.pylori) oversewing of a bleeding ulcer, or simple patch of a
Best test to confirm eradication of H. pylori: perforated ulcer or distal gastrectomy
negative urea breath test - Surgical options for PUD
treatment Triple therapy; PPI, Triple therapy; stop 1. HSV or parietal cell vagotomy or proximal gastric
antacids and H2 smoking, alcohol vagotomy
blockers consumption - safe (mortality risk <0.5%) with minimal side effects
- done by severing the vagal nerve supply to the proximal
- More than 90% of patients with PUD complain of 2/3 of the stomach (where essentially all parietal cells
abdominal pain (non-radiating, burning in quality & are located) & preserves the vagal innervation to the
epigastriac in location) antrum and pylorus and remaining abdominal viscera.
- Indication for endoscopy in PUD: 2. Taylor procedure
 Any symptomatic patient 45 yo and up - posterior truncal vagotomy and anterior seromyotomy
 Any symptomatic patient regardless of age with - attractive to HSV with similar results
alarm symptoms (see table 54) 3. Vagotomy + drainage (V+D) procedures
- Medical treatment for PUD: PPIs are the mainstay of - Truncal vagotomy dennervates the antrapyloric
therapy for PUD. mechanism, therefore, some sort of procedure is needed
to bypass or ablate the pylorus
table 50. treatment regimens for H. pylori - Types:
PPI + clarithromycin 500 mg BID + amoxicillin 1000 mg BID 10-14 d  Truncal vagotomy and pyroplasty
PPI + clarithromycin 500 mg BID + metronidazole 500 mg 10-14 d  Pyroplasty – useful in patients who require
BID pyloroduodenotomy to deal with the ulcer
PPI + + amoxicillin 1000 mg BID, then 5d complication (i.e. posterior bleeding duodenal
PPI + clarithromycin 500 mg BID + tinidazole mg BID
ulcer), limited focal a scarring in the pyloric
Salvage regimens for patients who fail one of the above initial
regimens
region
Bismuth subsalicylate 525 mg qid + metronidazole 250 mg 10-14 d  Truncal vagotomy and gastrojejunostomy
qid + tetracycline 500 mg qid + PPI  Good choice in patients with gastric outlet
PPI + amoxicillin 1000 mg bid + levofloxacin 500 mg daily 10 d obstruction

- Indications for surgical treatment for PUD - disadvantage: 10% of significant dumping / diarrhea
 bleeding
 perforation 4. Vagotomy and distal gastrectomy
 obstruction
 intractability or nonhealing ulcers (with discretion) D. ZOLLINGER-ELLISON SYNDROME
- For nonhealing PUD - uncontrolled secretion of abnormal amounts of
 Rare indication for surgery gastrin by a duodenal or pancreatic neuroendocrine
 Consider possible differentials for nonhealing PUD tumor (i.e., gastrinoma) leading to excessive
first production of HCl by the parietal cells, further
 Surgical treatment is considered in patients with excacerbating PUD.
nonhealing or intractable PUD who have multiple - Most common symptoms are epigastric pain, GERD and
recurrences, large ulcers (>2 cm), complications diarrhea. More than 90% of patients have peptic ulcer,
(obstruction, perforation, or hemorrhage), or most are in the typical location (proximal duodenum)
suspected malignancy - The inherited or familial form of gastrinoma is
associated with multiple endocrine neoplasia type 1
- Complications of PUD: or MEN1 (parathyroid, pituitary, and pancreatic or
duodenal tumors)
Table 55: Comparison of complications of PUD 
Bleeding PUD Perforation Gastric outlet  PHYSIOLOGY a
Obstruction Gastrin
-most common -2nd most common -rare (5% of all PUD - produced by antral G cells
cause of ulcer complication of PUD complications) - major hormonal stimulant of acid secretion during the
related death -classic symptom: -usually due to gastric phase.
-most common patient can duodenal or - The biologically active pentapeptide sequence at the C-
cause of UGIB in remember the exact prepyloric disease
terminal end of gastrin is identical to that of CCK
admitted patients time of onset of -presents with bilous
-presents with abdominal pain vomiting, profound - Luminal peptides and amino acids are the most
melena, hematemesis, -presents acute hypochloremic, potent stimulants of gastrin release
shock abdomen with metabolic alkalosis - luminal acid is the most potent inhibitor of gastrin
-abdominal pain is peritoneal signs -tx: nasogastric secretion.
uncommon (+)pneumoperitoneum suction, IV hydration - principal mediator of gastrin-stimulated acid
-tx: acid suppression on upright chest xray and electrolyte production is histamine from mucosal ECL cells
and NPO, transfusion (80% of patients) repletion, and
and endocopic tx -Tx: analgesia, antisecretory
(electrocautery + epi) antibiotics, isotonic medication, OR
for high risk group fluid resuscitation, - rule out - Gastrinoma triangle (or Pasaro’s triangle) : where
immediate OR pancreatic, gastric & 90% of ZES tumors are found
duodenal CA as a  boundaries: jxn of cystic & CBD, confluence of 2nd &
cause of obstruction 3rd segments of the duodenum and jxn of body and
neck of pancreas
 MUST KNOW a - most common symptoms of ZES are epigastric pain,
GERD & diarrhea. Can also be associated with
High risk lesions for massive bleeding (based on location): steatorrhea and other symptoms of malabsorption.
-posterior duodenal ulcer with erosion of gastroduodenal - Diagnosis:
artery  Fasting gastrin of 1mg/L, BAO >15 mEq/h or >5
-lesser curvature gastric ulcer with erosion of left gastric mEq/h (if with previous procedure for peptic
artery or branch ulcer) are suggestive of ZES
 Confirmatory test: secretin stimulation test
TOPNOTCH MEDICAL BOARD PREP SURGERY SUPPLEMENT HANDOUT Page 59 of 83
For inquiries visit www.topnotchboardprep.com.ph or email us at topnotchmedicalboardprep@gmail.com
TOPNOTCH MEDICAL BOARD PREP SURGERY SUPPLEMENT HANDOUT - Jules Lopez,MD-MBA,Teddy Carpio,MD-MBA
For inquiries visit www.topnotchboardprep.com.ph or email us at topnotchmedicalboardprep@gmail.com
 (+) secretin stimulation test: paradoxical rise  protective factors: aspirin (Yes! Schwartz says so.
in gastrin levels (200 pg/mL or greater) upon You don’t believe me? Check p. 927, 9th edition),
administration of IV bolus of secretin (an vitamin C and diet high in fruits and vegetables
inhibitor of gastrin)  premalignant conditions:
 Should also check for serum calcium and PTH  polyps
levels to rule out MEN1.  hyperplastic and adenomas are the
 Preoperative imaging of choice for gastrinoma: types associated with carcinoma
somatostatin receptor scintigraphy (octreotide  inflammatory, hamartomatous and
scan) heterotropic polyps are considered
 Basis: Gastrinoma cells contain type 2 benign lesions
somatostatin receptors that bind the indium-  atrophic gastritis: most common
labeled somatostatin analogue (octreotide) precancerous lesion / precursor of gastric
with high affinity, making imaging with a cancer
gamma camera possible  intestinal metaplasia: can be caused by H.
pylori
 PHYSIOLOGY a - Pathology
Somatostatin  Gastric Dysplasia: universal precursor to
- produced by D cells located throughout the gastric gastric adenocarcinoma
mucosa.  Early gastric cancer: adenocarcinoma limited to
- major stimulus for somatostatin release is antral the mucosa and submucosa of the stomach,
acidification regardless of lymph node status.
- acetylcholine inhibits its release  4 forms of gastric cancer (Gross morphology):
- Somatostatin effects: inhibits acid secretion from 1. Polypoid: bulk of tumor is intraluminal, not ulcerated
parietal cells, inhibits gastrin release from G cells & 2. Fungating: bulk of tumor is intraluminal, ulcerated
decreases histamine release from ECL cells. 3. Ulcerative: bulk of tumor is within the stomach wall
- Octreotide is a somatostatin analogue 4. Scirrhous (linitis plastic): bulk of tumor is within the
stomach wall; infiltrate the entire thickness of stomch
and cover a large surface area, poor prognosis
- Treatment:  Location of primary tumor: 40% distal stomach,
 Surgical resection of gastrinoma 30% middle stomach and 30% proximal stomach
 If (+) MEN1, perform parathyroidectomy 1st  Most important prognosticating factors: lymph
before resection of gastrinoma node involvement and depth of tumor invasion
 PPI for symptomatic relief
- Clinical manifestations:
E. GASTRITIS  Most patients diagnosed with gastric CA have
- Definition: Mucosal inflammation advanced stage III or IV disease
- Most common cause: H. pylori  S/Sx:
 Other causes: alcohol, NSAIDs, Crohn's disease,  weight loss and decreased food intake due
tuberculosis, and bile reflux to anorexia and early satiety (most
- Pathophysiology: common)
 infectious and inflammatory causes: result in  Abdominal pain (usually not severe and often
immune cell infiltration and cytokine production ignored)
which damage mucosal cells.  nausea, vomiting, & bloating.
 chemical agents (alcohol, aspirin, and bile): disrupt  Acute GI bleeding (unusual)
the mucosal barrier, allowing mucosal damage by  chronic occult blood loss (iron deficiency
back diffusion of luminal hydrogen ions. anemia and heme+ stool)
 Dysphagia: if the tumor involves the cardia of
F. STRESS ULCER the stomach.
- Pathophysiology: due to inadequate gastric mucosal  Paraneoplastic syndromes - Trousseau's
blood flow during periods of intense physiologic syndrome (thrombophlebitis), acanthosis
stress. nigricans (hyperpigmentation of the axilla and
 Adequate mucosal blood flow is important to groin), or peripheral neuropathy can be
maintain the mucosal barrier, and to buffer any present.
back-diffused hydrogen ions. When blood flow is  Physical examination:
inadequate, these processes fail and mucosal  Enlarged Cervical, supraclavicular (on the left
breakdown occurs referred to as Virchow's node), and axillary
lymph nodes
G. MALIGNANT NEOPLASMS OF THE STOMACH  Sister Joseph’s nodule: palpable umbilical
- The three most common primary malignant gastric nodue; pathognomonic for advanced
neoplasms are adenocarcinoma (95%), lymphoma disease
(4%), and malignant GIST (1%)  Blumer nodes: palpable nodularity in the
pouch of douglas; evidence of drop
GASTRIC ADENOCARCINOMA metastasis
- Epidemiology & etiology
 Gastric adenoCA is a disease of the elderly - Diagnosis
 Risk factors:  Do endoscopy and biopsy
 Black race: twice more common in blacks  Pre-operative staging: abdominal/pelvic CT
compared to whites scanning with IV and oral contrast
 Pernicious anemia
 Blood group A - Treatment
 FH of gastric CA  Surgery is the only curative treatment for gastric
 Diet: starchy diet high in pickled, salted, or cancer (radical subtotal gastrectomy)
smoked food, nitrates increases risk  Goal in resecting gastric adenocarcinoma: grossly
 H. pylori negative margin of at least 5 cm to achieve R0
 Smoking resection
 EBV infections
 Remember: Alcohol has no role in gastric CA
GASTRIC LYMPHOMA

TOPNOTCH MEDICAL BOARD PREP SURGERY SUPPLEMENT HANDOUT Page 60 of 83


For inquiries visit www.topnotchboardprep.com.ph or email us at topnotchmedicalboardprep@gmail.com
TOPNOTCH MEDICAL BOARD PREP SURGERY SUPPLEMENT HANDOUT - Jules Lopez,MD-MBA,Teddy Carpio,MD-MBA
For inquiries visit www.topnotchboardprep.com.ph or email us at topnotchmedicalboardprep@gmail.com
- stomach is the most common site of primary GI load into the small bowel) leading to circumferential
lymphoma expansion, additional accumulation of fluids emptying
- over 95% are non-Hodgkin's type. from stomach to duodenum and sudden expulsion of
- Most are B-cell type, thought to arise in MALT food to GIT  possibly due to ablation of the pylorus or
- MALT lymphomas is a form of NHL arising from the B decreased gastric compliance with accelerated
cells in the marginal zone of MALT emptying of liquids (after highly selective vagotomy)
- Is associated with chronic inflammation due to H. pylori - clinical manifestation: tachycardia, crampy abdominal
- Diligent search for extragstric disease should be done pain and diarrhea, dizziness, lightheadedness,
before giving a diagnosis of primary gastric lymphoma diaphoresis, nausea and vomiting after ingestion of a
- Treatment: chemotx is equivalent to surgery fatty or carbohydrate laden meal
 due to sudden shift in electrolytes and fluids
GASTROINTESTINAL STROMAL TUMOR (GIST) combined with increased blood flow to small
- Are submucosal solitary slow growing tumors arising intestine
from interstitial cells of Cajal (ICC) - treatment:
- 2/3 of all GISTs occur in the stomach, occurring  decreasing fluid and food intake to small frequent
commonly in the body portions
- defining feature of GISTS is their gain of function  avoid fatty and simple sugars
mutation of protooncogene KIT, a receptor tyrosine
kinase (majority of GISTS have activated mutation in
the c-kit protooncogene, which causes KIT to be AFFERENT LIMB OBSTRUCTION (BLIND LOOP
constitutively activated, presumably leading to SYNDROME)
persistence of cellular growth or survival signals) - occurs usually after a Billroth II procedure (distal
- Epithelial cell stromal GIST: most common cell type gastric resection followed by gastrojejunal
arising in the stomach; cellular spindle type is the next anastomosis)
most common; glomus tumor type is seen only in the - location of obstruction: at the limb associated with the
stomach. gastric remnant going to the duodenum
- Markers: (+) c-KIT, a protooncogene; a characteristic - clinical manifestations: severe epigastric pain following
shared with ICC eating, bilous emesis without food
- Diagnosis: endoscopy and biopsy, - treatment: convert Billroth II to roux en-Y gastric
- Mode of metastasis: hematogenous route; most bypass (possible problem: can delay gastric emptying)
common sites: liver and lung
- Treatment:
 Wedge resection with clear margins is adequate GASTRIC OUTLET OBSTRUCTION (see complications
surgical treatment of PUD as well)
 Imatinib (Gleevec): a chemotherapeutic agent - presents with hypochloremic, hypokalemic
that blocks the activity of the tyrosine kinase metabolic alkalosis   dehydration
product of c-kit, is reserved for metastatic or - as a compensatory response due to worsening
unresectable GIST. benign gastric neoplasms dehydration, Na conservation occurs in the kidney,
leading to renal tubular acidosis with subsequent
H. BENIGN GASTRIC NEOPLASMS: POLYP (see also aciduria
premalignant conditions of gastric adenoCA)
- most common benign tumor of the stomach
- 5 types: POST-GASTRECTOMY PROBLEMS
1. Adenomatous: (+) malignant potential; 10-15% of all (1) Dumping Syndrome: phenomenon caused by the
gastric polyps destruction or bypass of the pyloric sphincter. Symptoms
2. hyperplastic (regenerative): most common gastric are thought to be the result of the abrupt delivery of a
polyp (75% of all gastric polyps); occurs in the hyperosmolar load into the small bowel. Medical therapy
setting of gastritis and has a low malignant potential consists of dietary management and somatostatin
3. hamartomatous: benign analogue (octreotide)
4. inflammatory: benign a. Early Dumping: diaphoresis, weakness,
5. heterotopic (e.g., ectopic pancreas): benign
lightheadedness, tachycardia, crampy abdominal
***Polyps that are symptomatic, >2 cm, large
pain and diarrhea
hyperplastic or adenomatous should be removed,
b. Late Dumping: hypoglycemia and
usually by endoscopic snare polypectomy.
hyperinsulinemia
I. GASTRIC VOLVULUS (2) Diarrhea: may be a result of truncal vagotomy, dumping or
- is a twist of the stomach that usually occurs in malabsorption
association with a large hiatal hernia or unusually (3) Gastric Stasis: may be due to a problem with gastric motor
mobile stomach without hiatal hernia. function or be caused by an obstruction.
- Gastric volvulus is a chronic condition that can be (4) Bile Reflux Gastritis
surprisingly asymptomatic. (5) Roux Syndrome: post gastrectomy with great difficulty
- Clinical manifestations: abdominal pain and pressure with gastric emptying in the absence of mechanical
related to the intermittently distending and poorly obstruction. These patients present with vomiting,
emptying twisted stomach, dyspnea (due to pressure on epigastric pain and weight loss. Endoscopy may show
the lung), palpitations (due to pressure on the bezoar formation, dilation of the gastric remnant and/or
pericardium) and dysphagia (pressure on the dilatation of the Roux limb.
esophagus) (6) Gallstones
- Management: (7) Weight loss
 Vomiting and passage of a NGT may relieve (8) Anemia
symptoms (9) Bone Disease
 Gastric infarction is a surgical emergency

J. POSTGASTRECTOMY PROBLEMS REVIEW QUESTIONS


DUMPING SYNDROME 1. A patient with a vagotomy and pyloroplasty
- occurs after bariatric surgery and PUD repair (after returns with a recurrent ulcer. The best method
pyloroplasty, pyloromyotomy or distal gastrectomy) for determining if there was an inadequate
- mechanism: there is accumulation of digested food in vagotomy performed is
the small intestine (or abrupt delivery of hyperosmolar
TOPNOTCH MEDICAL BOARD PREP SURGERY SUPPLEMENT HANDOUT Page 61 of 83
For inquiries visit www.topnotchboardprep.com.ph or email us at topnotchmedicalboardprep@gmail.com
TOPNOTCH MEDICAL BOARD PREP SURGERY SUPPLEMENT HANDOUT - Jules Lopez,MD-MBA,Teddy Carpio,MD-MBA
For inquiries visit www.topnotchboardprep.com.ph or email us at topnotchmedicalboardprep@gmail.com
- Mucosal folds: plicae circulares / valvulae
a. Direct vagal stimulation conniventes
b. Stimulated gastric analysis - Peyer’s patches: most commonly located in the ileum
c. Stimulated PPI (pancreatic polypeptide) levels which are aggregates of lymphoid follicles and is a local
d. None of the above – there is no good test to source of IgA
determine inadequate vagotomy - Difference between jejunum and ileum: jejunum has
larger circumference, thicker wall, less fatty
Answer: C mesentery, and longer vasa recta
Historically, gastric analysis was performed most - Calcium is primarily absorbed in the duodenum
commonly to test for the adequacy of vagotomy in through both transcellular transport and paracellular
postoperative patients with recurrent or persistent diffusion.
ulcer. Now this can be done by assessing peripheral
pancreatic polypeptide levels in response to sham
feeding. A 50% increase in pancreatic polypeptide  PHYSIOLOGY a
within 30 minutes of sham feeding suggests vagal
integrity. Representative Regulatory Peptides produced in the small
Intestine:
2. Which of the following procedures for PUD has the
highest incidence of postoperative diarrhea? Hormone Source Actions
Somatostatin D Cell Inhibits GI secretion, motility &
a. Graham patch splanchnic perfusion
b. Parietal cell vagotomy Secretin (1st S cell Stimulates exocrine pancreatic
c. Truncal vagotomy and pyloroplasty hormone discovered in secretion; stimulates intestinal
the human body)
d. Distal gastrectomy without vagotomy secretion
Cholecystokinin I cell Stimulates exocrine pancreatic
Answer: C secretion; Stimulates GB
Parietal Truncal Truncal
Cell vagotomy & vagotomy & emptying; Inhibits sphincter of
vagotomy pyloroplasty Antrectomy Oddi contraction
Operative 0 <1 1
Motilin M cell Stimulates intestinal motility
mortality Glucagon-like L cell Stimulates intestinal
rate (%)
Ulcer 5-15 5-15 <2
peptide 2 proliferation
recurrence Peptide YY L cell Inhibits intestinal motility &
(%)
Dumping(%)
secretion

Mild <5 10 10-15

Severe 0 1 1-2
B. SMALL BOWEL OBSTRUCTION
Diarrhea
- Epidemiology:
 most frequently encountered surgical disorder
Mild <5 25 20 of the small intestine.
 Lesions can be described as:
Severe 0 2 1-2
 Intraluminal: foreign bodies, gallstones,
meconium
 Intramural: tumors, Crohn’s disease
associated inflammatory strictures
 Extrinsic: adhesions, hernias, carcinomatosis
- Etiology:
 Intra-abdominal adhesions related to prior
SMALL INTESTINE abdominal surgery: most common cause (75%
of cases)
A. Gross Anatomy and Histology  Hernias
B. Small bowel obstruction  Malignancy: due to extrinsic compression or
C. Ileus & other disorders of intestinal motility invasion by advanced malignancies arising in
D. Crohn’s disease organs other than the small bowel
E. Intestinal fistulas  Crohn's disease.
F. Small bowel neoplasms  Congenital abnormalities (i.e. midgut volvulus and
G. Radiation enteritis intestinal malrotation) diagnosed at adulthood.
H. Meckel’s diverticulum  superior mesenteric artery syndrome: rare;
I. Acquired diverticulum compression of the 3rd portion of the duodenum by
J. Mesenteric Ischemia the superior mesenteric artery as it crosses over
K. Obscure GI bleeding this portion of the duodenum; seen in young
L. Intussuception asthenic individuals who have chronic symptoms
M. Short bowel syndrome suggestive of proximal small bowel obstruction.

- Pathophysiology
 Gas (usually from swallowed air) and fluid (from
A. GROSS ANATOMY AND HISTOLOGY swallowed liquids and GI secretions) accumulate
- raison d'être of the GI tract because it is the principle within the intestinal lumen proximal to the site of
site of nutrient digestion and absorption. obstruction  intestinal activity ↑ to overcome the
- Layers of the small intestine (from innermost to obstruction (seen as colicky pain and diarrhea) 
outermost layers): mucosa, submucosa, muscularis bowel distention  ↑ intraluminal and intramural
propria and serosa pressures rise  intestinal motility is eventually
 Contraction of the inner circular layer causes reduced with fewer contractions If intramural
results in luminal narrowing pressure becomes high enough  impaired
 Contraction of the outer longitudinal layer results intestinal microvascular perfusion  intestinal
in bowel shortening ischemia  necrosis (strangulated bowel
 Contraction of the muscularis mucosa contribute to obstruction)
mucosal or villus motility (but not peristalsis)
TOPNOTCH MEDICAL BOARD PREP SURGERY SUPPLEMENT HANDOUT Page 62 of 83
For inquiries visit www.topnotchboardprep.com.ph or email us at topnotchmedicalboardprep@gmail.com
TOPNOTCH MEDICAL BOARD PREP SURGERY SUPPLEMENT HANDOUT - Jules Lopez,MD-MBA,Teddy Carpio,MD-MBA
For inquiries visit www.topnotchboardprep.com.ph or email us at topnotchmedicalboardprep@gmail.com
 With obstruction, the luminal flora of the small  Obstruction presenting in the early postoperative
bowel (which is usually sterile) changes  period (particularly those undergoing pelvic
Translocation of these bacteria to regional lymph surgery, especially colorectal procedures) pose
nodes the greatest risk for developing early
postoperative small bowel obstruction.
Partial SBO: only a portion of the intestinal lumen is  obstruction should be considered if Sx of
occluded, allowing passage of some gas and fluid. intestinal obstruction occur after the initial
return of bowel function or if bowel function
Complete SBO: complete occlusion fails to return within the expected 3 to 5
days after abdominal surgery.
Closed loop obstruction: dangerous form of SBO, in  Regardless of etiology, the affected intestine should
which a segment of intestine is obstructed both be examined, and nonviable bowel resected.
proximally and distally (e.g., with volvulus). In such  Criteria for viability: normal color
cases, the accumulating gas and fluid cannot escape (pinkish), (+)peristalsis, and marginal
either proximally or distally from the obstructed segment, arterial pulsations.
leading to a rapid rise in luminal pressure, and a rapid
progression to strangulation. Ogilvie syndrome
- massive idiopathic non-obstructive dilatation of the
- Clinical presentation colon; acute colonic pseudo-obstruction
 Symptoms: colicky abdominal pain, nausea, - Distention of the abdomen leading to obstruction
vomiting (a more prominent symptom with - Tends to occur following non-abdominal procedures
proximal obstructions than distal; vomitus is (i.e. cardiac surgery)
usually feculent), and obstipation, continued - Due to a neurologic dysfunction, electrolyte
passage of flatus and/or stool beyond 6 to 12 hours abnormality and ↑age
after onset of symptoms (more for partial SBO than - Treatment: NGT, IV neostigmine, IV atropine (to
complete SBO) counter bradycardia as SE of neostigmine), exploratory
 Signs: abdominal distention (pronounced if the site laparotomy during worst case scenario)
of obstruction is distal ileum & absent if the site of
obstruction is in the proximal small intestine), C. ILEUS & OTHER DISORDERS OF INTESTINAL
initially hyperactive bowel sounds (maybe minimal MOTILITY
towards the late stages of bowel obstruction) - Ileus is a temporary motility disorder
 Lab findings: hemoconcentration and electrolyte - Postoperative ileus: most frequently implicated
abnormalities (reflect intravascular volume cause of delayed discharge following abdominal
depletion) & Mild leukocytosis operations
 Features of strangulated SBO: abdominal pain - Pathophysiology:
often disproportionate to the degree of  Common etiologies: abdominal operations,
abdominal findings ( suggestive of intestinal infection and inflammation, electrolyte
ischemia), tachycardia, localized abdominal abnormalities (↓K, ↓&↑Mg, ↓ Na) & drugs
tenderness, fever, marked leukocytosis, & acidosis. (anticholinergics, opiates, phenothiazine, CCB,
Tricyclic antidepressants)
- Diagnosis  Proposed mechanisms: surgical stress-induced
 Confirmatory test: abdominal series (radiograph sympathetic reflexes, inflammatory response
of the abdomen with the patient in a supine position, mediator release, and anesthetic/analgesic effects
upright position &radiograph of the chest with the  Normal temporal pattern of return of GI
patient in an upright position) motility : small intestinal motility (1st 24
 Sensitivity of abdominal radiographs for hours), gastric motility (48 hours) and colonic
detecting SBO is 70-80% motility (3 to 5 days)
 Triad of dilated small bowel loops (>3 cm in - Clinical presentation (usually resembles SBO):
diameter), air-fluid levels seen on upright Inability to tolerate liquids and solids by mouth, nausea,
films, and a paucity of air in the colon is and lack of flatus or bowel movements, vomiting,
MOST SPECIFIC abdominal distention & diminished or absent bowel
 CT scan sounds
 80 to 90% sensitive - diagnosis: If ileus persists beyond 3 to 5 days
 70 to 90% specific postoperatively  or occurs in the absence of
 Apperance of closed-loop obstruction in abdominal surgery, further investigation is warranted
CT: presence of U-shaped or C-shaped dilated to rule out possibility of mechanical obstruction
bowel loop associated with a radial
distribution of mesenteric vessels converging  CLINICAL PEARLS a
toward a torsion point.
 Appearance of strangulation in CT: Measures to REDUCE postoperative ileus:
thickening of the bowel wall, pneumatosis Intraoperative measures:
intestinalis (air in bowel wall), portal venous - minimize handling of bowel
gas, mesenteric haziness and poor uptake of IV - laparascopic approach, if possible
contrast into the wall of the affected bowel. - avoid excessive intraoperative fluid administration
- Treatment Postoperative measures
 Fluid resuscitation: isotonic replacement - early enteral feeding
 Broad spectrum antibiotics - epidural anesthesia, if indicated
 NGT placement for decompression - avoid excessive IV fluid administration
 If complete SBO, perform surgery - correct electrolyte abnormalities
 If partial SBO, may be approached conservatively - consider m-opiod antagonists (
given that there is no fever, tachycardia,
tenderness, or an increase in white cell count ***Remember, though often recommended, the use of early
(indicates perforation) ambulation and routine NG intubation has NOT been
 most patients with partial small obstruction demonstrated to be associated with earlier resolution of
whose symptoms do not improve within 48 hours postoperative ileus.
after initiation of nonoperative therapy should
undergo surgery.

TOPNOTCH MEDICAL BOARD PREP SURGERY SUPPLEMENT HANDOUT Page 63 of 83


For inquiries visit www.topnotchboardprep.com.ph or email us at topnotchmedicalboardprep@gmail.com
TOPNOTCH MEDICAL BOARD PREP SURGERY SUPPLEMENT HANDOUT - Jules Lopez,MD-MBA,Teddy Carpio,MD-MBA
For inquiries visit www.topnotchboardprep.com.ph or email us at topnotchmedicalboardprep@gmail.com
D. INFLAMMATORY BOWEL SYNDROME: CROHN’S  (+) drainage of enteric material through the
DISEASE VS ULCERATIVE COLITIS  abdominal wound or through existing drains:
associated with intra-abdominal abscesses.
table 56. Inflammatory bowel syndrome - Diagnosis
Crohn’s disease Ulcerative colitis  CT scan: most useful initial test
description chronic, idiopathic Chronic inflammatory  small bowel series or enteroclysis examination: can
transmural inflammatory disease affecting only the
be obtained to demonstrate the fistula's site of
disease with a propensity colonic mucosa and
to affect the distal ileum submucosa origin in the bowel.
Etiology & -more common in Higher chance of leading to  Fistulogram: greater sensitivity in localizing the
epidemiology Ashkenazi jews & colorectal cancer fistula origin.
caucasaians, females, has - Treatment
a bimodal age  Should follow orderly steps (done to maximize
distribution (3rd & 6th spontaneous closure)
decade), (+) strong  Stabilization: fluid & electrolyte resuscitation,
pattern of family
TPN, antibiotics,
inheritance, smokers &
higher Socio-eco status  Investigation: see diagnosis
Pathology Focal transmural Inflammation is limited to  Decision to do perform surgery or do
inflammation, mucosa and submucosa conservative treatment
aphthous ulcers only; lead pipe colon  Surgeons usually do 2 to 3 months of
(earliest lesion of (lacks haustral markings); conservative therapy before
Crohn’s), no granulomas considering surgical intervention.
non casseating  This approach is based on evidence that
granulomas,
90% of fistulas that are going to close,
cobblestoning,
**fat wrapping close within a 5-week interval
(encroachment of - Definitive management: surgery (if failure of
mesenteric fat onto the spontaneous closure during time period or with
serosal surface of the complications and risk factors)
bowel): pathognomonic - rehabilitation
of crohn’s
 MUST KNOW a
spares rectum,can occur
anywhere in the GI tract, Primarily affects the colon
skip lesions, targets & rectum and is continous; Remember FRIEND (factors that inhibit spontaneous closure of
terminal ileum can also manifest with fistulas):
backwash ileitis Foreign body within the fistula tract
fistula, Radiation enteritis
Infection/Inflammation at the fistula origin Epithelialization of
the fistula tract
S/Sx Inisiduous onset with bloody diarrhea and Neoplasm at the fistula origin
waxing and waning crampy abdominal pain.
Distal obstruction of the intestine
course of abdominal pain Proctitis may produce
(usually RLQ), nonbloody tenesmus; can proceed to
diarrhea & weight loss; fulminant colitis and toxic
megacolon F. SMALL BOWEL NEOPLASMS
(+) extraintestinal - Adenomas are the most common benign neoplasm of
manifestation: arthritis, the small intestine
uveitis, iritis, eythema - Most common location for primary adenocarcinoma
nodosum, pyoderma and adenomas of the small bowel is DUODENUM
gangrenosum, primary
sclerosing cholangitis,
(EXCEPT in patient’s with Crohn’s disease, which is
nephrolithiasis found mostly in the ileum)
Diagnosis Endoscopy (skip lesions, Endoscopy & proctoscopy - Primary small bowel cancers are rare; 1.1 to 2.4% of
cobblestoning, abscess (earliest manifestation is all GI malignancies
formation and fistulas); mucosal edema; mucosal  Adenocarcinomas: 35 - 50%
histology demonstrate friability ; ulceration; (+)  Carcinoid tumors: 20 to 40%
granulomas; (+) (pANCA) Pus and mucus)  Lymphomas: 10 to 15 %
and anti–Saccharomyces  GISTs: most common location is STOMACH (60-
cerevisiae antibody
70%), 2nd most common location is small intestine
(ASCA
Treatment Sulfasalazine + steroids; Similar to Crohn’s;
(25-35%)
surgery if unresponsive colectomy after 15 years of - Pathophysiology: proposed explanations for the low
to aggressive medical Tx symptoms frequency of small intestinal neoplasms
 dilution of environmental carcinogens in the liquid
E. INTESTINAL FISTULAS chyme present in the SI lumen
- abnormal communication between two epithelialized  rapid transit of chime (limiting the contact time
surfaces between carcinogens and the intestinal mucosa)
- can be internal (within GI tract or adjacent organs)or  relatively low concentration of bacteria in small
external (with communication to external environment) intestinal chime (therefore, low concentration of
- Kinds: carcinogenic products of bacterial metabolism)
 low output fistulas - drain less than 200 mL of  mucosal protection by secretory IgA and
fluid/day hydrolases such as benzpyrene hydroxylase 
 high output fistulas - drain more than 500 mL of render carcinogens less active
fluid/day  efficient epithelial cellular apoptotic mechanisms
- 80% of enterocutaneous fistulas are due to that serve to eliminate clones harboring genetic
iatrogenic complications mutations.
- Clinical presentation
 usually become clinically evident between the 5th &
10th postop - Clinical presentation
 initial signs: Fever, leukocytosis, prolonged ileus,  Partial SBO is the most common mode of
abdominal tenderness, and wound infection presentation
 Only becomes symptomatic when it becomes large

TOPNOTCH MEDICAL BOARD PREP SURGERY SUPPLEMENT HANDOUT Page 64 of 83


For inquiries visit www.topnotchboardprep.com.ph or email us at topnotchmedicalboardprep@gmail.com
TOPNOTCH MEDICAL BOARD PREP SURGERY SUPPLEMENT HANDOUT - Jules Lopez,MD-MBA,Teddy Carpio,MD-MBA
For inquiries visit www.topnotchboardprep.com.ph or email us at topnotchmedicalboardprep@gmail.com
- Diagnosis: Because of the absent or nonspecific common in pediatric age), intestinal
symptoms associated with most small intestinal obstruction (most common in adults), and
neoplasms, these lesions rarely are diagnosed diverticulitis
preoperatively - Diagnosis
- Treatment: surgical resection  Usually discovered incidentally on radiographic
imaging, during endoscopy, or at the time of
G. RADIATION ENTERITIS surgery.
- An undesired side effect of radiation therapy is  CT scan: low sensitivity and specificity
radiation-induced injury to the small intestine  Enteroclysis: has 75% accuracy but u not
- The SI is susceptible to radiation-induced injury applicable during acute presentations
because it has a high rate of rapidly proliferating  Radionuclide scans (99mTc-pertechnetate): positive
cells compared to the other portions of the GI tract only when the diverticulum contains associated
- Pathophysiology ectopic gastric mucosa that is capable of uptake of
 principal mechanism of radiation-induced cell the tracer
death is believed to be apoptosis resulting from - Treatment: surgical
free-radical–induced breaks in double-stranded  diverticulectomy
DNA  If the indication for diverticulectomy is bleeding,
 The intensity of injury is related to the dose of segmental resection of ileum that includes both the
radiation administered diverticulum and the adjacent ileal peptic ulcer
- Pathology should be performed.
 acute injury: villus blunting, dense infiltrate of  Segmental ileal resection may also be necessary if
leukocytes and plasma cells within the crypts, the diverticulum contains a tumor or if the base of
mucosal sloughing, ulceration, and hemorrhage the diverticulum is inflamed or perforated.
 chronic injury: progressive occlusive vasculitis that  The management of incidentally found
leads to chronic ischemia and fibrosis that affects (asymptomatic) Meckel's diverticula is
all layers of the intestinal wall, rather than the controversial.
mucosa alone  leading to strictures, abscesses,
and fistulas I. ACQUIRED DIVERTICULUM
- Clinical presentation - Considered as false diverticula (because their walls
 Acute: nausea, vomiting, diarrhea, and crampy consist of mucosa and submucosa but lack a complete
abdominal pain. muscularis)
 Chronic: becomes evident within 2 years of - more common in the duodenum, near the ampulla
radiation administration, most commonly presents (periampullary, juxtapapillary, or peri-Vaterian
with partial small bowel obstruction with diverticula)
nausea, vomiting, intermittent abdominal - Diverticula in the jejunum tend to be large and
distention, crampy abdominal pain, and weight loss accompanied by multiple other diverticula, whereas
The terminal ileum is the most frequently affected those in the ileum tend to be small and solitary.
segment - Pathophysiology
- Diagnosis  Due to acquired abnormalities of intestinal smooth
 Enteroclysis: most accurate imaging test for muscle or dysregulated motility  leading to
diagnosing chronic radiation enteritis, herniation of mucosa and submucosa through
 CT scan findings are neither very sensitive nor weakened areas of muscularis.
specific for chronic radiation enteritis; should be - Clinical presentation
obtained to rule out the presence of recurrent  Acquired diverticula are asymptomatic unless
cancer (because of overlap in clinical associated complications arise
manifestations)  Complications (6 to 10% of patients): intestinal
- Treatment: supportive therapy obstruction, diverticulitis, hemorrhage,
perforation, and malabsorption.
H. MECKEL’S DIVERTICULUM  - Diagnosis
- most prevalent congenital anomaly of the GI tract  Most acquired diverticula are discovered
- considered a true diverticula incidentally on radiographic imaging, during
- location is usually found in the ileum within 100 cm of endoscopy, or at the time of surgery.
the ileocecal valve  Enteroclysis is the most sensitive test for detecting
- 60% of Meckel's diverticula contain heterotopic jejunoileal diverticula
mucosa (most common: gastric mucosa – 60%; - Treatment
others: Pancreatic acini, Brunner's glands, pancreatic  If asymptomatic, observe
islets, colonic mucosa, endometriosis, and hepatobiliary  If (+)complications, such as bleeding and
tissues). diverticulitis: segmental intestinal resection for
diverticula located in the jejunum or ileum.
 MUST KNOW a
rule of TWOs of Meckel’s diverticulum: J. MESENTERIC ISCHEMIA

2% prevalence Table 57: comparison of acute vs chronic mesenteric ischemia


2:1 female predominance Acute mesenteric ischemia Chronic mesenteric ischemia
location 2 ft proximal to the ileocecal valve in adults Causes: - results from atherosclerotic
-arterial embolus: most lesions in the main splanchnic
one half of those who are symptomatic are under 2 years of age
common cause; most common arteries (celiac, superior
source: heart; most common mesenteric, and inferior
location: SMA mesenteric arteries
- Pathophysiology -arterial thrombosis: occur in -rarely leads to infarction
 Failure of the the omphalomesenteric proximal mesenteric arteries
(vitelline) duct to undergo obliteration during -vasospasm (nonocclusive -Postprandial abdominal pain
the 8th week of gestation mesenteric ischemia): is the most prevalent
- Littre’s hernia: Meckel's diverticula found in an diagnosed in critically ill symptom, producing a
patients receiving vasopressor characteristic aversion to food
inguinal or femoral hernia sacs; when incarcerated, agents. ("food-fear") and weight loss
can cause intestinal obstruction -venous thrombosis: involves (can be mistaken as a symptom
- Clinical presentation the superior mesenteric vein of malignancy)
 most common presentations associated with in 95% of cases; associated with
symptomatic Meckel's diverticula: bleeding (most heritable or acquired

TOPNOTCH MEDICAL BOARD PREP SURGERY SUPPLEMENT HANDOUT Page 65 of 83


For inquiries visit www.topnotchboardprep.com.ph or email us at topnotchmedicalboardprep@gmail.com
TOPNOTCH MEDICAL BOARD PREP SURGERY SUPPLEMENT HANDOUT - Jules Lopez,MD-MBA,Teddy Carpio,MD-MBA
For inquiries visit www.topnotchboardprep.com.ph or email us at topnotchmedicalboardprep@gmail.com
Acute mesenteric ischemia Chronic mesenteric ischemia
coagulation DO

Golden period: 3 hours –


intestinal sloughing; 6 hours:
full thickness intestinal
infarction

Hallmark of acute mesenteric


ischemia: Severe abdominal REVIEW QUESTIONS
pain, out of proportion to the
degree of tenderness on 1. Vitamin B12 deficiency can occur after
examination
a. Gastrectomy
SUPERIOR MESENTERIC ARTERY SYNDROME b. Gastric bypass
c. Ileal resection
Compression of the third, or transverse, portion of the d. ALL OF THE ABOVE
duodenum between the aorta and the superior mesenteric
artery Answer: D
Vitamin B12 (cobalamin) malabsorption can result
The superior mesenteric artery usually forms an angle of from a variety of surgical manipulations. The
approximately 45° (range, 38-56°) with the abdominal aorta, vitamin is initially bound by saliva-derived R
and the third part of the duodenum crosses caudal to the protein. In the duodenum, R protein is hydrolyzed
origin of the superior mesenteric artery, coursing between by pancreatic enzymes, allowing free cobalamin to
the superior mesenteric artery and aorta. Any factor that bind to gastric parietal cell-derived intrinsic factor.
sharply narrows the aortomesenteric angle to approximately The cobalamin-intrinsic factor complex is able to
6-25° can cause entrapment and compression of the third escape hydrolysis by pancreatic enzymes, allowing
part of the duodenum as it passes between the superior it to reach the terminal ileum, which expresses
mesenteric artery and aorta, resulting in superior specific receptors for intrinsic factor. Subsequent
mesenteric artery syndrome. events in cobalmin absorption are poorly
characterized, but the intact complex probably
In addition, the aortomesenteric distance in superior enters enterocytes through translocation. Because
mesenteric artery syndrome is decreased to 2-8 mm (normal each of these steps is necessary for cobalamin
is 10-20 mm). Alternatively, other causes implicated in assimilation, gastric resection, gastric bypass and
superior mesenteric artery syndrome include high insertion ileal resection can each result in Vitamin B 12
of the duodenum at the ligament of Treitz, a low origin of the insufficiency.
superior mesenteric artery, and compression of the
duodenum due to peritoneal adhesion. 2. Which of the following is the LAST to recover
from postoperative ileus?

K. INTUSSUCEPTION  a. Stomach
- refers to a condition where one segment of the intestine b. Small Bowel
becomes drawn in to the lumen of the proximal c. Colon
segment of the bowel d. NONE of the above – recovery is simultaneous
- usually is seen in the pediatric population
- Adult intussusceptions are rare; usually with distinct Answer: C
pathologic lead point (which can be malignant) The return of normal motility generally follows a
- commonly present with a history of intermittent characteristic temporal sequence, with small
abdominal pain and signs and symptoms of bowel intestinal motility returning to normal within the 1st
obstruction 24 hours after laparotomy and gastric and colonic
- CT scan: diagnostic of choice motility returning to normal by 48 hours and 3 to 5
 Finding: "target sign" days, respectively. Because small bowel motility is
- Treatment: surgical resection of the involved segment returned before colonic and gastric motility, listening
and the lead point, which needs to undergo pathologic for bowel sounds is not a reliable indicator that ileus
evaluation to rule out an underlying malignancy. has fully resolved. Functional evidence of coordinated
GI motility in the form of passing flatus or bowel
L. SHORT BOWEL SYNDROME movement is a more useful indicator.
- presence of less than 200 cm of residual small bowel in
adult patients
- usually acquired (s/p intestinal resection)
- can result to malabsorptive symptoms: diarrhea,
APPENDIX
dehydration, and malnutrition,
- most common etiologies:
 adults: acute mesenteric ischemia, malignancy, and A. Anatomy
Crohn's disease B. Acute appendicitis
 pedia: intestinal atresias, volvulus, and necrotizing C. Appendiceal tumors - Carcinoid
enterocolitis
- Pathophysiology
 Normal: Resection of less than 50% of the small A. ANATOMY
intestine is generally well tolerated. - Function: immunologic organ; a GALT tossie that
 Symptomatic when greater than 50 to 80% of the secrete immunoglobulins
small intestine has been resected. - The base of the appendix can always be found at the
 Malabsorption in patients who have undergone confluence of the taenia
massive small bowel resection is exacerbated by a - Tip of the appendix varies: retroceccal (most common),
characteristic hypergastrinemia-associated pelvic, subcecal, preileal, or right pericolic position
gastric acid hypersecretion that persists for 1 to  ***The location of the tip of the appendix
2 years postoperatively determine the location of physical findings
- Treatment: produced by irritation of parietal peritoneum
 TPN & enteral nutrition
 Pharmacotherapy
TOPNOTCH MEDICAL BOARD PREP SURGERY SUPPLEMENT HANDOUT Page 66 of 83
For inquiries visit www.topnotchboardprep.com.ph or email us at topnotchmedicalboardprep@gmail.com
TOPNOTCH MEDICAL BOARD PREP SURGERY SUPPLEMENT HANDOUT - Jules Lopez,MD-MBA,Teddy Carpio,MD-MBA
For inquiries visit www.topnotchboardprep.com.ph or email us at topnotchmedicalboardprep@gmail.com
- the luminal capacity if the normal appendix is 0.1 cc.  Diverticulitis: of cecum or perforated carcinoma
secretion of as little as 0.5 cc of fluid distal to the of cecum is difficult to distinguish clinically from
obstruction raises intraluminal pressure to 60 cm H20. appendicitis; diagnosis is usually done
intraoperatively
B. ACUTE APPENDICITIS  Epiploic apendagitis: infarction of the appendage
- Etiology due to torsion; pain shift is unusal and patient is
 Fecalith: most common cause of appendiceal usually not ill
obstruction; usually in adults  Gynecologic conditions: ruptured ectopic
 Lymphoid hyperplasia: most common cause of pregnancy, PID, ruptured grafian follicle, twisted
appendiceal obstruction in patients of pediatric age ovarian cyst
- Pathogenesis: Luminal obstruction  bacterial  Intussusception: patient’s age, type of pain,
overgrowth, active mucosal secretion & increased palpable mass in the lower quadrant and passage
luminal pressure of currant jelly stool may help with diagnosis;
- Natural history: rarely resolves; ultimately lead to barium enema offers both diagnostic and
gangrene and perforation therapeutic option for intussusception.
- Clinical presentation
 Hallmark of appendicitis: poorly localized pain (due - Special conditions:
to distension stimulates visceral afferent pain
fibers) that is referred to the periumbilical region AP IN THE YOUNG
- Diagnostic accuracy in these age group is lower
(via autonomic innervations)  Right lower
compared to adults due to imprecise history and
quadrant pain and tenderness (via somatic
nonspecific abdominal complaints
innervations due to involvement of the parietal
- Hx: vomiting, fever and diarrhea are common
peritoneum)
complaints
 Vomiting usually follows abdominal pain
- PE: abdominal distention, maximal tenderness in the
 Anorexia is a constant symptom; if not present,
right lower quadrant, the inability to walk or walking
question diagnosis; usually precedes abdominal
with a limp, and pain with percussion, coughing, and
pain
hopping
 Variations in the anatomic location of the
- Gangrene and rupture are more common in these age
appendiceal tip account for the different
group because of delays in diagnosis
manifestation of the abdominal pain
 Retrocecal: flank or back pain
 Pelvis: findings maybe absent; painful DRE AP IN THE ELDERLY
exam
- Usually with atypical presentation: fever, leukocytosis
- PE maneuvers:
and RLQ pain maybe minimal or absent
 Dumphy’s sign: increased pain during coughing or
- Have 60-90% rupture rate
jumping
- The atrophic omentum is less capable of walling off a
 Rovsing’s sign: pain in the RLQ when pressure is
applied on the LLQ; this suggests peritoneal perforated appendix  diffuse peritonitis or distant
irritation intra-abdominal abscess are expected
 Psoas sign: pain on extension of the right thigh - If patient is older than 60yo, always rule out cancer
with the patient lying on the left side; this is due to because the definitive treatment for that is right
the pain elicited by the stretched psoas muscle hemicolectomy (if affecting the cecum)
irritating the inflamed appendix
 Obturator’s sign: pain with passive rotation of the AP IN PREGNANCY
flexed right hip; suggests that the inflamed tip lies - Most common surgical emergency in pregnancy
in the appendix - In pregnancy, the gravid uterus pushes the appendix
- Diagnosis: usually based on history and physical superiorly and the tip medially
examination even in the absence of laboratories and - Most consistent sign of AP in pregnant women: pain in
imaging the right side of the abdomen
 Laboratory findings - Common occurrence of abdominal pain, nausea and
1. Moderate leukocytosis with leukocytosis in the normal course of pregnancy makes
polymorphonuclear predominance (if above diagnosis difficult
18,000 – suspect abscess or perforation) - Most cases occur during 2nd trimester
2. Can also have normal WBC count (1/3 of - Fetal mortality is 2-8.5%; increases to 35% with
patients) rupture
3. Minimal albuminuria, (+) WBC and RBC in
AP IN HIV OR AIDS PATIENTS
urine if appendix is retrocecal
4. Anemia in elderly should raise suspicion of - Similar presentation to non-infected patients
carcinoma of the cecum - Risk of appendiceal rupture is higher for these patients
 Imaging - DDx: CMV enteritis, typhilitis, fungal, protozoal and
1. Plain abdominal films: fecalith, localized mycobacterial infections
ileus on the RLQ & loss of peritoneal fat
Treatment: appendectomy
strip
2. UTZ: tubular, immobile and
C. APPENDICEAL TUMORS - CARCINOID
noncompressible appendix,wall thickness of
- most common location is appendix (50%), ileum (25%)
>2mm and outer diameter of at least 6 mm
then rectum (20%)
are indicative of appendicitis
- ileal carcinoid has the highest potential for metastasis
3. CT scan: thickened by more than 5-
(arounf 35%) vs appendiceal carcinoid which has
7mm&fluid filled, periappendiceal
lowest potential for metastasis (3%)
inflammation along with fat stranding, fluid
- Gross appearance: small, firm, circumscribed, yellowish
collections & phlegmons
tumor
- Differential diagnosis
- Treatment:
 Acute mesenteric adenitis: associated with URTI
 <2cm at distal appendix: appendectomy
and presents with a more diffused abdominal painl
 >2cm or at base: right hemicolectomy
also with generalized lymphadenopathy with
lymphocytosis
REVIEW QUESTIONS
 Acute gastroenteritis: crampy abdominal pain
with watery stools, nausea and vomiting
TOPNOTCH MEDICAL BOARD PREP SURGERY SUPPLEMENT HANDOUT Page 67 of 83
For inquiries visit www.topnotchboardprep.com.ph or email us at topnotchmedicalboardprep@gmail.com
TOPNOTCH MEDICAL BOARD PREP SURGERY SUPPLEMENT HANDOUT - Jules Lopez,MD-MBA,Teddy Carpio,MD-MBA
For inquiries visit www.topnotchboardprep.com.ph or email us at topnotchmedicalboardprep@gmail.com
1. A patient suspected of having appendicitis underwent - not attempted without anesthesia if patient complains
exploration, Crohn’s disease was found. Which of the of severe perianal pain and does not tolerate digital
following are true? rectal examination
a. The normal appendix should always be 2. Proctoscopy
removed - useful for the examination of the rectum and distal
b. All grossly involved bowel, including the sigmoid colon
appendix, should be resected. - can be both therapeutic and diagnostic
c. An inflamed appendix, cecum and terminal - length: 25 cm
ileum, should be resected - 15-19 mm diameter proctoscope is useful for diagnostic
d. Perforated bowel and advanced Crohn’s examination
disease with obstruction should be resected. - useful for polypectomy, electrocoagulation, detorsion of
sigmoid volvulus
Answer: D
If a normal appendix is found at the time of 3. Flexible sigmoidoscopy and colonoscopy
laparotomy, other causes should be sought. If - provides excellent visualization of colon and rectum
Crohn’s disease is encountered and the cecum and - can be both diagnostic and therapeutic
base of the appendix are normal, an appendectomy - length:
should be performed. If the base is involved with  60 cm: sigmoidoscope
Crohn’s disease and the appendix is normal,  100-160 cm: colonoscope
appendectomy should not be performed. If the - full length insertion:
finding of Crohn’s disease is uncomplicated by  may allow visualization as far as splenic flexure:
perforation or obstruction, ileal resection is not sigmoidoscope
indicated. However, in the case of perforation or  may allow visualization as far as terminal ileum:
Crohn’s disease with obstruction, the involved colonoscope
bowel should be resected.
IMAGING:

1. Plain x-ray and contrast studies


- plain x-rays of abdomen (upright, supine and
COLON, RECTUM, ANUS diaphragmatic views) are useful for detecting free intra-
abdominal air, bowel gas patterns suggestive of small or
A. Embryology large bowel obstruction and volvulus
B. Diagnostic evaluation of Colon, Rectum and Anus - contrast studies are useful for evaluationg obstructive
C. Evaluation of Common symptoms symptoms, delineating fistulous tracts and diagnosing
D. Diverticular disease small perforations or anastomotic leaks.
E. Colorectal adenocarcinoma - Gastrografin (water soluble contrast agent) is
F. Colorectal carcinoid tumors recommended if perforation or leak is suspected
G. Anal intraepithelial neoplasia (Bowen’s disease) - Double contrast barium enema is 70-90% sensitive
H. Volvulus for the detection of mass lesions greater than 1 cm in
I. Colonic pseudoobstruction (Ogilvie’s syndrome) diameter
J. Hemorrhoids  If a small, non obstructing lesion is considered,
K. Anal fissure colonoscopy is the preferred imaging modality of
L. Anorectal abcess choice
M. Fistula in ano
2. CT
- the utility of CT is in the detection of extraluminal
disease, such as intra-abdominal abscesses and
pericoloic inflammation and in staging colorectal
A. EMBRYOLOGY
carcinoma (because of its sensitivity in detecting
- Embryonic GI tract begins developing during 4th week
hepatic metastasis)
of gestation
 REMEMBER: a standard CT scan is INSENSITIVE
Table 57. Embryology of GI tract
for detection of intraluminal lesions
FOREGUT MIDGUT HINDGUT - If considering a perforation / anastomotic leak:
Esophagus, stomach, small intestine, distal transverse check for extravasation of oral or rectal contrast
pancreas, liver, ascending colon, colon, descending - Bowel wall thickening / mesenteric stranding
duodenum and proximal colon, rectum, and suggests inflammatory bowel disease, enteritis/colitis
transverse colon proximal anus or ischemia
Celiac artery SMA IMA
***distal anus is derived from the ectoderm; BS: internal pudendal 3. MRI
artey - the main use of MRI in colorectal DO is in the evaluation
of pelvic lesions
- The colon has 5 distinct layers: mucosa, submucosa,
- more sensitive than CT for detecting bony involvement
inner circular muscle, outer longitudinal muscle, and
or pelvic sidewall extension of rectal tumors.
serosa
- Can be useful in the detection and delineation of
complex fistulas in ano.
 MUST KNOW
4. Positron Emission Tomography
Most common bacterium within the colon is B. fragilis - useful for imaging tissues with high levels of anaerobic
followed by E. coli and Enterococcus sp. glycolysis, such as malignant tumors
- F-fluorodeoxyglucose is injected as a tracer  its
metabolism results in positron emission
B. DIAGNOSTIC EVALUATION OF COLON, RECTUM AND - Used as an adjunct to CT in staging colorectal cancer
ANUS
5. Angiography
ENDOSCOPY: - used for the detection of bleeding within the colon or
small bowel
1. Anoscopy - to visualize hemorrhage angiographically, bleeding
- useful instrument for the examination of the anal canal must be relatively brisk (0.5 to 1 cc per minute)

TOPNOTCH MEDICAL BOARD PREP SURGERY SUPPLEMENT HANDOUT Page 68 of 83


For inquiries visit www.topnotchboardprep.com.ph or email us at topnotchmedicalboardprep@gmail.com
TOPNOTCH MEDICAL BOARD PREP SURGERY SUPPLEMENT HANDOUT - Jules Lopez,MD-MBA,Teddy Carpio,MD-MBA
For inquiries visit www.topnotchboardprep.com.ph or email us at topnotchmedicalboardprep@gmail.com
- if extravasation of contrast is identified, infusion with
vasopressin or angiographic embolization can be 2. Lower GI bleeding
therapeutic. - first goal in managing a patient with GI hemorrhage:
ADEQUATE RESUSCITATION
6. Endorectal and Endoanal UTZ - insert NGT (1st test that should be performed) since the
- is used primarily to evaluate the depth of invasion of most common cause of bleeding can either be
neoplastic lesions in the rectum and detecting esophageal, gastric or duodenal
sphincter defects & outlining complex anal fistulas  if (+) return of bile  suggests that bleeding is
- normal rectal wall can be seen as a 5 layer structure distal to the ligament of Treitz
- UTZ can reliably differentiate benign polyps from  if bloody/non-bile secretions  suggests an upper
invasive tumors based upon the integrity of the intestinal source; do EGD right away
submucosal layer. - technetium-99-tagged RBC scan: highly sensitive (as
- Accuracy in detecting depth of mural invasion is 81- little as 0.1 cc/hour of bleeding can be detected);
94% however location is imprecise  perform angiography
to localize bleeding
PHYSIOLOGIC AND PELVIC FLOOR INVESTIGATIONS: - if sharp, knife-like pain + bright red rectal bleeding with
useful in the evaluation of patients with incontinence, bowel movements  anal fissure
constipation, rectal prolapse, obstructed defecation and - if painless, bright red rectal bleeding secondary to
other pelvic floor disorders bowel movements  internal hemorrhoids

1. Manometry 3. Constipation and obstructed defecation


- procedure: pressure-sensitive catheter is placed in the - A very common problem
lower rectum  catheter is withdrawn through the anal - rule out an underlying metabolic, pharmacologic,
canal and pressures recorded endocrine, psychological and neurologic causes first
- values: before work up
 resting pressure (normal: 40-80 mmHg): reflects - a stricture or mass lesion should be excluded by
the function of the internal anal sphincter colonoscopy or barium enema
 Squeeze pressure (normal: 40-80 mmHg above - once other causes have been ruled out, perform transit
resting pressure): maximum voluntary contraction studies
pressure minus resting pressure, reflects the - Medical management is the mainstay of treatment for
function of the external anal sphincter constipation (High fiber, increase fluids & laxatives)
 High pressure zone (normal: 2-4 cm): estimates
the kength of the anal canal 4. Diarrhea and irritable bowel syndrome
- Absence of rectoanal inhibitory reflex is - Acute bloody diarrhea and pain can be due to infection
characteristic of Hirschsprung’s disease or inflammation
- chronic diarrhea has a more difficult diagnostic
2. Neurophysiology dilemma since causes are myriad (ulcerative colitis,
- Neurophysiologic testing assesses function of the crohn’s colitis, malabsorption, short-gut syndrome,
pudendal nerve and recruitment of puborectalis muscle carcinoid, islet cell tumors, etc)
fibers
5. Incontinence
LABORATORY STUDIES: - ranges in severity from occasional leakage of gas and
liquid stool to daily loss of solid stool
1. Fecal Occult Blood testing (FOBT) - can be neurogenic or anatomic
- is a screening test for colonic neoplasms in  Neurogenic: diseases of CNS, spinal cord, pudendal
asymptomatic, average-risk individuals nerve injury
- occult bleeding from any GI source will produce a  Anatomic: congenital abnormalities, procidentia,
positive result (since it is a non specific test for overflow incontinence secondary to impaction,
peroxidase contained in hemoglobin) neoplasm or trauma
- any positive FOBT mandates further investigation, - Most common traumatic cause of incontinence is injury
usually by colonoscopy to the anal sphincter during vaginal delivery

2. Stool studies D.
DIVERTICULAR DISEASE
- helpful in the evaluation of etiology of diarrhea Diverticulosis: presence of diverticula without
-
- wet mount examination: (+) fecal leukocytes indicate inflammation.
colonic inflammation or presence of invasive organisms  Common in patients with low fiber diet
(such as E. coli or Shigella)  Majority tend to occur after the age of 85
- Sudan red stain to stool sample: to evaluate steatorrhea  Sigmoid colon: most common site of
diverticulosis
3. CEA: tumor marker  Common symptom: massive LGIB
- elevated in 60-90% of patients with colorectal cancer; - Diverticulitis: inflammation and infection associated
however, not an effective screening agent for colorectal with diverticula.
CA  5% of complicated diverticulitis develop a fistula to
- serial monitoring used after curative-intent surgery is an adjacent organ. Most common of which is a
done colovesical fistula.
C. EVALUATION OF COMMON SYMPTOMS - More common is false diverticula type
1. Pain  Only mucosa and muscularis mucosa have
- abdominal pain related to colon and rectum can result herniated (also called pulsion diverticula) through
from obstruction (inflammatory or neoplastic), the colonic wall, in between taenia coli (area of
inflammation, perforation or ischemia weakness)
- pelvic pain can originate from distal colon and rectum - True diverticula, comprises all layers of the bowel, is
or adjacent urogenital structures congenital and rare
 tenesmus: due to proctitis or from rectal or - Clinical manifestations of diverticulitis:
rectrorectal mass  Uncomplicated diverticulitis: left-sided abdominal
 cyclical pain + menses + rectal bleeding: pain, with or without fever, mass and leukocytosis
endometriosis  complicated diverticulitis: abscess, obstruction,
- anorectal pain is most often secondary to anal fissure, diffuse peritonitis (free perforation), or fistulas
perirectal abscess and/or fistula, or a thrombosed (most common is colovesical fistula)
hemorrhoids - Hinchey staging for complicated diverticulitis
TOPNOTCH MEDICAL BOARD PREP SURGERY SUPPLEMENT HANDOUT Page 69 of 83
For inquiries visit www.topnotchboardprep.com.ph or email us at topnotchmedicalboardprep@gmail.com
TOPNOTCH MEDICAL BOARD PREP SURGERY SUPPLEMENT HANDOUT - Jules Lopez,MD-MBA,Teddy Carpio,MD-MBA
For inquiries visit www.topnotchboardprep.com.ph or email us at topnotchmedicalboardprep@gmail.com
Stage I: colonic inflammation with an associated  Hyperplastic polyp: most common type of
pericolic abscess all polyps; usually small, multiple and sessile;
Stage II: colonic inflammation with a retroperitoneal or occur frequently in the rectosigmoid region
pelvic abscess  Pseudopolyps (or inflammatory polyps):
Stage III: purulent peritonitis occur most commonly in the context of
Stage IV: fecal peritonitis. inflammatory bowel disease, amebic colitis,
- Diagnosis: CT scan ischemic colitis, and schistosomal colitis; not
 Appears as pericolic soft tissue stranding, premalignant, but they cannot be
colonic wall thickening, and/or phlegmon distinguished from adenomatous polyps based
- Treatment: upon gross appearance & therefore should be
 Uncomplicated diverticulitis: outpatient therapy removed.
with broad-spectrum oral antibiotics for 7-10 days  Hamartomas: similar appearance to
& low-residue diet; failure to improve within 48-72 adenomatous polyps but is not considered to
hours indicates abscess formation be premalignant
 If 2nd episode of uncomplicated diverticulitis or 1st  Familial juvenile polyposis: autosomal
episode of complicated diverticulitis: elective dominant DO in which patients develop
sigmoid colectomy is recommended hundreds of polyps in the colon and
 Small abscesses (<2 cm diameter) may be treated rectum; degenerate into adenomas 
with parenteral antibiotics. carcinoma
 Larger abscesses are best treated with CT-guided  Peutz Jeghers syndrome: characterized
percutaneous drainage by polyposis of the small intestine and, to
a lesser extent, of the colon and rectum. ;
E. COLORECTAL ADENOCARCINOMA Characteristic melanin spots are noted on
- Most common malignancy of the GIT the buccal mucosa and lips of these
- Risk factors: patients.
1. Aging: dominant risk factor for colorectal cancer;  Cronkite-Canada syndrome: GI
incidence increases after 50 yo polyposis + alopecia + cutaneous
2. Known FH of cancer: accounts for 20% of cases pigmentation + atrophy of the fingernails
3. Diet: high in animal fat and low in fiber and toenails; SSx: Diarrhea, vomiting,
4. Inflammatory bowel syndrome malabsorption, and protein-losing
5. Cigarette smoking enteropathy
 Cowden syndrome: autosomal dominant
- Pathogenesis: genetic defects
disorder with hamartomas of all three
Figure 34. Schematic diagram showing progression from normal
embryonal cell layers; Facial
colonic mucosa to carcinoma of colon trichilemmomas, breast cancer, thyroid
disease, and GI polyps are typical of the
syndrome.
 Neoplastic polyps
 Tubular adenomas: most common type of
neoplastic polyps; asymptomatic,
pedunculated, less than 1 cm in size and occur
APC commonly in the rectosigmoid region
- Tumor suppressor gene located at chromosome 5 <1cm: rare chance for malignany
- Function: the protein product of APC is for maintain 1-2cm: 10% chance for malignancy
cellular adhesions and suppressing neoplastic growth >2 cm: 30% chance for malignancy
- APC inactivation leads to sporadic colorectal cancer  Tubulovillous adenoma: mixed; 22% chance
- Mutated in individuals with familial adenomatous for malignancy
polyposis (FAP)  Villous adenoma: sessile, larger and
symptomatic, can cause malignancy by 40-
K-RAS 50%; highest risk of cancer
- Proto-oncogene located in chromosome 12  Sessile adenomas are more likely to harbor
- Function: encodes for plasma membrane based protein malignancy compared to pedunculated ones
involved in transduction of growth and differential
signals - Inherited colorectal carcinoma
- Mutation leads to uncontrolled cell division  Familial adenomatosis polyposis
- K-RAS activation leads to colorectal cancer  rare autosomal dominant condition accounts
- for only about 1% of all colorectal
adenocarcinomas.
DCC  Due to mutation in the APC gene, located on
- Tumor suppressor gene located at chromosome 18 chromosome 5q
- Function: encodes for a protein responsible for cell to  Clinically, patients develop hundreds to
cell contact thousands of adenomatous polyps shortly
- Loss of DCC gene (or inactivation) tend to present in after puberty.
more advanced carcinomas  The lifetime risk of colorectal cancer in FAP
- Present in 70% of colorectal carcinomas patients approaches 100% by age 50 years.
 Flexible sigmoidoscopy of first-degree
P53 relatives of FAP patients beginning at age 10
- Tumor suppressor gene located at chromosome 17 to 15 years
- Mutations of this gene are the most common genetic  FAP may be associated with extraintestinal
abnormality found in human cancer genes manifestations such as congenital hypertrophy
- Function: crucial for initiating apoptosis in cells with of the retinal pigmented epithelium, desmoid
irreparable genetic damage. tumors, epidermoid cysts, mandibular
- Mutations in p53 are present in 75% of colorectal osteomas (Gardner's syndrome), and central
cancers nervous system tumors (Turcot's syndrome).
- p53 inactivation leads to colorectal cancer  HNPCC or Lynch syndrome
 Rare autosomal dominant disorder arising
- Polyps: from errors in mismatch repair
 Non-neoplastic polyps (no malignant potential)

TOPNOTCH MEDICAL BOARD PREP SURGERY SUPPLEMENT HANDOUT Page 70 of 83


For inquiries visit www.topnotchboardprep.com.ph or email us at topnotchmedicalboardprep@gmail.com
TOPNOTCH MEDICAL BOARD PREP SURGERY SUPPLEMENT HANDOUT - Jules Lopez,MD-MBA,Teddy Carpio,MD-MBA
For inquiries visit www.topnotchboardprep.com.ph or email us at topnotchmedicalboardprep@gmail.com
 is characterized by the development of - Result of a neuroendocrine tumor that secretes
colorectal carcinoma at an early age (average neurotransmitters (serotonin, ACTH, histamine,
age: 40 to 45 years). dopamine, tryptophan, substance P, bradykinin)
 The risk of synchronous or metachronous - occur most commonly in the GI tract
colorectal carcinoma is 40%. - most common location is appendix (50%), ileum (25%)
 HNPCC also may be associated with then rectum (20%)
extracolonic malignancies, including - ileal carcinoid has the highest potential for metastasis
endometrial (most common), ovarian, - appendiceal has lowest potential for metastasis
pancreas, stomach, small bowel, biliary, and - Carcinoid tumors in the proximal colon are less
urinary tract carcinomas. common and are more likely to be malignant.
 Diagnosis: Amsterdam criteria for clinical - Can be part of MEN type I
diagnosis of HNPCC are three affected - risk of malignancy increases with size (more than 60%
relatives with histologically verified of tumors greater than 2 cm in diameter are associated
adenocarcinoma of the large bowel (one must with distant metastases)
be a 1st degree relative of one of the others) in - clinical manifestation:
2 successive generations of a family with 1  triad of flushing (due to excess bradykinin),
patient diagnosed before age 50 years. diarrhea (due to excess serotonin) & valculhar
 Screening colonoscopy is recommended heart disease (primarily affects the mitral valve
annually for at-risk patients beginning at from excess serotonin)
either age 20 to 25 years or 10 years younger  others: hypotension, tachycardia, alcohol
than the youngest age at diagnosis in the intolerance
family, whichever comes first. - diagnosis:
 24 hour 5-HIAA collection (5-HIAA is a metabolite
 Familial colorectal cancer of serotonin)
 Nonsyndromic familial colorectal cancer - treatment:
accounts for 10 to 15% of patients with  Small carcinoids can be locally resected, either
colorectal cancer transanally or using transanal endoscopic
 Screening colonoscopy is recommended every microsurgery.
5 years beginning at age 40 years or beginning  Larger tumors or tumors with obvious invasion
10 years before the age of the earliest into the muscularis require more radical resection
diagnosed patient in the pedigree.  Medical: somatostatin analogues (octreotide)
 RT
- Routes of spread & natural history:
 Regional lymph node involvement is the most G. ANAL INTRAEPITHELIAL NEOPLASIA (BOWEN’S
common form of spread of colorectal carcinoma DISEASE)
and usually precedes distant metastasis - refers to squamous cell carcinoma in situ of the
 T stage (depth of invasion) is the single most anus.
significant predictor of lymph node spread - precursor to an invasive squamous cell carcinoma
 The number of lymph nodes with metastases (epidermoid carcinoma)
correlates with the presence of distant disease and - may appear as a plaque-like lesion, or may only be
inversely with survival: 4 or more involved lymph apparent with high-resolution anoscopy and
nodes predict a poor prognosis application of acetic acid or Lugol's iodine solution.
 most common site of distant metastasis from - associated with HPV infection types 16 and 18.
colorectal cancer is the liver (via hematogenous - Incidence has increased dramatically among HIV-
spread to the portal venous system) positive, homosexual men.
- Treatment:
- Screening:  Ablation
 annual DRE at age 40  Topical immunomodulators such as imiquimod,
 FOB at age 50 Topical 5-FU
 Flexible signoidoscopy every 5 years at age 50
 Colonoscopy if with risk factors H. VOLVULUS
- Clinical presentation: change in bowel habits, rectal - occurs when an air-filled segment of the colon twists
bleeding, melena, unexplained anemia, or weight about its mesentery
loss - clinical manifestations: similar to SBO; abdominal
- Staging and treatment distention, nausea, and vomiting; can rapidly progress
to generalized abdominal pain and tenderness; Fever
Table 58. Duke staging of colorectal cancer & treatment and leukocytosis are heralds of gangrene and/or
Stage description 5 year Treatment perforation
survival
Stage A Cancer limited 90% Wide resection Table 59. Comparison of sigmoid vs cecal volvulus
to mucosa & of colon with Sigmoid volvulus Cecal volvulus
submucosa sampling of LN -90% of cases -<20% of cases
Stage B Cancer invades 70% (to rule out ↑er -plain abdominal xray: bent inner -plain abdominal xray: kidney-
the muscularis disease stage); tube or coffee bean appearance shaped, air-filled structure in the
propria stage B can with the convexity of the loop lying LUQ
also employ in the RUQ (opposite site of
RT+CT obstruction)
Stage C Invasion of 30% Surgery + -gastrografin enema: bird’s beak
local LN chemotx (5- (pathognomonic)
fluorouracil + Initial management: fluid -surgical exploration once
leucovorin) + resuscitation followed by diagnosis is made; no room for
RT endoscopic detorsion (rigid endoscopic detorsion
Stage D Distant Limited palliative proctoscope); if suspecting
metastasis survival gangrene or perforation, perform
immediate surgical exploration
 LAR: for tumors located 5-10 cm from anal verge
 APR: tumors less than 5 cm from ananl verge and if
recurrent cancer at LAR site
 MUST KNOW a
F. COLORECTAL CARCINOID TUMORS
TOPNOTCH MEDICAL BOARD PREP SURGERY SUPPLEMENT HANDOUT Page 71 of 83
For inquiries visit www.topnotchboardprep.com.ph or email us at topnotchmedicalboardprep@gmail.com
TOPNOTCH MEDICAL BOARD PREP SURGERY SUPPLEMENT HANDOUT - Jules Lopez,MD-MBA,Teddy Carpio,MD-MBA
For inquiries visit www.topnotchboardprep.com.ph or email us at topnotchmedicalboardprep@gmail.com
Bird’s beak is also seen in barium esophagogram of  Initially, can be managed conservatively with
achalasia! lubricants, warm sitz bath and bulk laxatives
(treatment of choice)
 Surgery: lateral subQ partial internal
sphincterectomy
I. COLONIC PSEUDOOBSTRUCTION (OGILVIE’S  Posterior fissurectomy & sphincterectomy
SYNDROME) can lead to keyhole defect & constant
- Distention of the abdomen leading to colonic soiling
obstruction (even if there is no obvious signs of
obstruction) L. ANORECTAL ABCESS 
- most commonly occurs in hospitalized patients and is - Perianal abscess: most common manifestation and
associated with the use of narcotics, bedrest, and appears as a painful swelling at the anal verge
comorbid disease. - Ischiorectal abscess: happens when there is Spread
- Due to a neurologic dysfunction, electrolyte through the external sphincter below the level of the
abnormality and ↑age puborectalis; may become extremely large and may not
- Treatment: NGT, IV neostigmine, IV atropine (to be visible externally; DRE will reveal a painful swelling
counter bradycardia as SE of neostigmine), exploratory laterally in the ischiorectal fossa
laparotomy during worst case scenario) - Intersphincteric abscess: occur in the intersphincteric
space and are notoriously difficult to diagnose; causes
J. HEMORRHOIDS deep pain in the rectum without external manifestation
- are cushions of submucosal tissue containing venules, - Pelvic and superior levator abscess: rare; may result
arterioles, and smooth-muscle fibers that are located in from extension of an intersphincteric or ischiorectal
the anal canal abscess upward, or extension of an intraperitoneal
- Excessive straining, increased abdominal pressure, and abscess downward
hard stools lead to further prolapsed of hemorrhoids - Horseshoe abscess: bilateral ischiorectal, supralevator
- Difference between internal and external hemorrhoids or perianal abscesses that communicate; begins as a
posterior midline infection
- Treatment: drainage with local anesthesia
Table 60: comparison of internal and external hemorrhoids - Signs of (+) fistula: nonhealing of an abscess wounds or
Internal hemorrhoids External hemorrhoids recurrence of an abscess at the same location
-Exaggerated submucosal vascular - are dilated veins of the inferior
cushions normally located above hemorrhoidal plexus located
M. FISTULA IN ANO
dentate line; covered by below the dentate line and
insensate transitional mucosa of covered by anoderm; can cause - Classification (based on relationship to the anal
the anal canal and not by significant pain sphincter complex)
anoderm; only become painful 1. Intersphincteric (most common): tracks
when already thrombosed / through the distal internal sphincter and
necrosis intersphincteric space to an external opening near
the anal verge
-prolapsing hemorrhoids: are
internal hemorrhoids beyond the 2. Transsphincteric: often results from an ischiorectal
dentate line abscess and extends through both the internal and
external sphincters
3. Suprasphincteric: originates in the intersphincteric
- Types of internal hemorrhoids: graded according to
plane and tracks up and around the entire external
extent of prolapsed sphincter
4. Extrasphincteric: originates in the rectal wall and
1st degree: bulge into the anal canal and may prolapse tracks around both sphincters to exit laterally,
beyond the dentate line on straining usually in the ischiorectal fossa
2nd degree: prolapse through the anus but reduce - Goodsall’s rule : states that, if the external opening
spontaneously
is anterior to the imaginary line drawn between the
3rd degree: prolapse through the anal canal and require
ischial tuberosities, the fistula runs directly into the
manual reduction
anal canal. If the external opening is posterior, the
4th degree: prolapse but cannot be reduced and are at
tract curves to the posterior midline.
risk for strangulation
- Combined internal and external hemorrhoids:
***EXCEPTION: if an anterior external opening is
straddle the dentate line; mixed characteristics
greater than 3 cm from the anal margin, these
- Post partum hemorrhoids: result from straining
fistulas usually track to the posterior midline!!!
during labor, which results in edema, thrombosis,
and/or strangulation. Figure 35. Goodsall’s rule
- Treatment:
 Dietary fiber, stool softeners, ↑OFI, avoid straining:
for 1st & 2nd degree hemorrhoids
 Rubber band ligation: for persistent 1st & 2nd
degree hemorrhoids & selected 3rd degree
hemorrhoids
 If thrombosed hemorrhoids, perform excision
 Most common complication of
hemorrhoidectomy: urinary retention

K. ANAL FISSURE
- is a tear in the anoderm distal to the dentate line
- 90% of fissures are located at the posterior midline, - Treatment: fistulotomy with adequate drainage or seton
an area where the anoderm is least supported by the placement
sphincter
- Fissures located laterally should arouse suspicion of
Crohn’s, UC, syphilis, TB,leukemia REVIEW QUESTIONS
- Clinical manifestation: tearing pain with defecation
and hematochezia; often too tender to tolerate DRE 1. Which of the following is important in maintaining
- Treatment the integrity of the colonic mucosa?
TOPNOTCH MEDICAL BOARD PREP SURGERY SUPPLEMENT HANDOUT Page 72 of 83
For inquiries visit www.topnotchboardprep.com.ph or email us at topnotchmedicalboardprep@gmail.com
TOPNOTCH MEDICAL BOARD PREP SURGERY SUPPLEMENT HANDOUT - Jules Lopez,MD-MBA,Teddy Carpio,MD-MBA
For inquiries visit www.topnotchboardprep.com.ph or email us at topnotchmedicalboardprep@gmail.com

a. short-chain fatty acids


b. alanine
c. medium-chain fatty acids
d. glutamine

Answer: A
Short chain fatty acids are produced by bacterial
fermentation of dietary carbohydrates. Short chain
fatty acids are an important source of energy for
the colonic mucosa, and metabolism by colonocytes
provides energy for processes such as active
transport of sodium. Lack of a dietary source for
production of short chain fatty acids, or diversion of
the fecal stream by an ileostomy or colostomy, may
result in mucosal atrophy and diversion colitis.

2. Match the organs in the left hand column with the


location of their referred pain in the right hand A.Midline -are used because of the flexibility offered by this
incision approach in establishing adequate exposure.
column. (items in the right may be used more than -the incision in the fused midline aponeurotic
once) tissue (linea alba) is simple and requires no
division of skeletal muscle.
A. Gallbladder a. epigastrium B.paramedian -made longitudinally 3 cm off the midline,
B. Jejunum b. periumbilical incision through the rectus abdominis sheath structures,
C. Rectum c. hypogastrium and have largely been abandoned in favor of
D. Pancreas d. shoulder midline or nonlongitudinal access methods
E. Appendix C.right -Subcostal incisions on the right (Kocher incision
subcostal for cholecystectomy) or left (for splenectomy) are
incision archetypal muscle-dividing incisions that
Answer: A – a,d; B – b; C – c; D – a; E – b
D.bilateral generally result in the transaction of some or all
subcostal of the rectus abdominis muscle fibers and
The visceral peritoneum is innervated by C fibers investing aponeuroses.
coursing with the autonomic ganglia. C fibers are -These incisions generally are closed in two
unmyelinated, slow-conducting (0.5-5.0 m/s), layers (anterior aponeurotic sheath of the rectus
polymodal nociceptors that travel bilaterally with the muscle medially, transitioning to external oblique
sympathetic and parasympathetic fibers. Visceral pain is muscle and aponeurosis more laterally &
a response to injury of the visceral peritoneum. posterior, deeper layer consists of internal
oblique and 73ransverses abdominis muscle)
Distension, stretch, traction, compression, torsion,
E.Rocky davis Right lower quadrant incision or muscle splitting
ischemia and inflammation trigger visceral pain incision incision for appendectomy
fibers. Abdominal organs are insensate to heat, F.McBurney it begins 2 to 5 centimeters above the anterior
cutting and electrical stimulation. incision superior iliac spine and continues to a point one-
third of the way to the umbilicus (McBurney's
Visceral pain is typically vague and crampy and is point). Thus, the incision is parallel to the
perceived in the region of oprigin of the embryologically external oblique muscle
derived autonomic ganglia. Foregut organs (proximal G.Transverse Similar to kocher incision (subcostal incision).
to the ligament of treitz) refer pain to the celiac chain, incision Preferred for newborns and infants because more
abdominal exposure is gained per length of the
and the pain is felt in the epigastrium. The organs of the
incision compared to vertical exposure
midgut (small intestine, ascending colon) refer pain to H.Pfannenstiel Pfannenstiel incision, used commonly for pelvic
the superior mesenteric chain (periumbilical chain) and incision procedures, is distinguished by transverse skin
those of the hindgut (transverse and descending colon, and anterior rectus sheath incisions, followed by
sigmoid colon and rectum) to the inferior mesenteric rectus muscle retraction and longitudinal incision
ganglia and hypogastrium. of the peritoneum.

B. RECTUS ABDOMINIS DIASTASIS


ABDOMINAL WALL & HERNIAS - Other name: diastasis recti
- is a clinically evident separation of the rectus abdominis
A. Abdominal Incisions muscle pillars resulting to a characteristic bulging of
B. Rectus abdominis diastasis the abdominal wall in the epigastrium (sometimes
C. Rectus sheath hematoma mistaken for a ventral hernia)
D. Abdominal wall hernias - may be congenital
E. Incisional hernias - can be associated with advancing age, in obesity, or
F. Retroperitoneal fibrosis after pregnancy
G. Inguinal hernias - In the postpartum setting, rectus diastasis tends to
H. Femoral hernia occur in women who are of advanced maternal age, who
have a multiple or twin pregnancy, or who deliver a
high-birth-weight infant.
- Diagnosis:
 CT scan: can differentiate rectus diastasis from a
A. ABDOMINAL INCISIONS
true ventral hernia
Figure 36. Abdominal incisions - Treatment: surgery

C. RECTUS SHEATH HEMATOMA


- As a result of hemorrhage from any of the network of
collateralizing vessels (superior and inferior epigastric
arteries or veins) within the rectus sheath and muscles
- History: trauma, sudden contraction of the rectus
muscles with coughing, sneezing, or any vigorous
physical activity.

TOPNOTCH MEDICAL BOARD PREP SURGERY SUPPLEMENT HANDOUT Page 73 of 83


For inquiries visit www.topnotchboardprep.com.ph or email us at topnotchmedicalboardprep@gmail.com
TOPNOTCH MEDICAL BOARD PREP SURGERY SUPPLEMENT HANDOUT - Jules Lopez,MD-MBA,Teddy Carpio,MD-MBA
For inquiries visit www.topnotchboardprep.com.ph or email us at topnotchmedicalboardprep@gmail.com
- Clinical manifestations: sudden onset of unilateral - Treatment: corticosteroids with or without surgery
abdominal pain that increases with contraction of the (only indicated when renal function is compromised)
rectus muscles; palpable tender mass
 (+)Fothergill's sign: palpable abdominal mass F. INGUINAL HERNIAS
that remains unchanged with contraction of the
rectus muscles Table 61. comparison of Inguinal hernias
- Diagnosis: Indirect inguinal hernia Direct inguinal hernia
 Abdominal UTZ may show a solid or cystic mass etiology - usually congenital; due -Usually acquired;
within the abdominal wall to patent processus weakness in the
 CT scan: most definitive study to establish the vaginalis abdominal wall
musculature
correct diagnosis and to exclude other disorders
Risk Strenuous physical activity, obesity, ehler’s danlos,
- Treatment: nonoperative; surgery is indicated in factors: smoking
instances of expensing hematoma and hemodyanamic anatomy protrude lateral to the protrusions medial to the
instability inferior epigastric inferior epigastric
vessels, through the deep vessels, in Hesselbach's
D. ABDOMINAL WALL HERNIAS inguinal ring triangle
- This is due to defects in the parietal abdominal wall PE: cough impulse is Cough impulse is
fascia and muscle through which intra-abdominal or inguinal controlled; felt on the manifest; felt on the
preperitoneal contents can protrude occlusion dorsum of fingertip fingertip
test
- ACQUIRED HERNIAS
 may develop through slow architectural
deterioration of the muscular aponeuroses or they
may develop from failed healing of an anterior  MUST KNOW a
abdominal wall incision (incisional hernia).
 most common finding is a mass or bulge on the Hesselbach’s triangle:
anterior abdominal wall, which may increase in Inferior: inguinal ligament
size with a Valsalva maneuver Medial: rectus abdominis
 PE reveals a bulge on the anterior abdominal wall Superolateral border: inferior epigastric vessels
that may reduce spontaneously, with recumbency,
or with manual pressure
 Treatment: if incarcerated (cannot be reduced) or  MUST KNOW a
strangulated (BS is compromised)  do surgical
Femoral hernias are more prevalent in females compared to
correction
males but the most common type of groin hernia in females
- PRIMARY VENTRAL HERNIAS
is still indirect inguinal hernia.
 Non incisional or true ventral hernias
 Examples:

Epigastric hernias: congenital due to defective


- Clinical manifestations: groin pain
midline fusion of lateral abdominal wall; occurs in
- Diagnosis: usually employed for ambiguous diagnosis
multiples and are small; located in the midline
(i.e. obese patients)
between the xiphoid process and the umbilicus;
- Treatment: definitive treatment is surgical repair
found to contain omentum or a portion of the
falciform ligament.
Bassini repair: anterior approach, nonprosthetic,
hernia reduced and the defect oversewn, &
Umbilical hernias: due to a patent umbilical ring;
reconstruction the site of weakness; disadvantage: (+)
more common in premature infants;spontaneous
tension on the reconstructed tissue
closure can occur at age of 5, no closure by that
time, do elective surgical repair
Shouldice repair: anterior approach, nonprosthetic,
multilayer (4-layer suture repair) reconstruction
Spigelian hernias: occur anywhere along the
distributes the tension, effectively resulting in a
length of the Spigelian line or zone—an
tension-free repair; lowest recurrence rate
aponeurotic band of variable width at the lateral
border of the rectus abdominis.
Pott’s repair: high ligation of the sac only, with no
repair of the inguinal canal; used for indirect hernias
E. RETROPERITONEAL FIBROSIS
only
- class of disorders characterized by hyperproliferation of
fibrous tissue in the retroperitoneum
McVay repair: anterior approach, nonprosthetic; the
- if primary, it is known as Ormond disease
conjoined tendon is sutured to the cooper’s ligament
- may be secondary to inflammatory process, malignancy,
laterally; can be used for indirect, direct & femoral
or medication (methysergide, ergotamine,
hernias
hydralazine, methyldopa and B blockers)
***problem with anterior non prosthetic
- Men are twice as likely to be affected as women
approaches: high recurrence rates
- primarily affects individuals in the 4th-6th decades of
life.
Lichenstein tension free repair: addition of a mesh
- Clinical manifestations:
prosthesis effected a reconstruction of the posterior
 Sx: insidious onset of dull, poorly localized
inguinal canal, without placing tension on the floor itself
abdominal pain, unilateral leg swelling,
intermittent claudication, oliguria, hematuria, &
Read-rives repair: anterior preperitoneal approach
dysuria.
 PE: hypertension, the palpation of an abdominal or
Rives, stoppa, wanz repair: giant prosthetic
flank mass, lower extremity edema (unilateral or
reinforcement of the visceral sac; preperitoneal
bilateral), or diminished lower extremity pulses
approach
(unilateral or bilateral).
- Diagnosis:
Kugel repair: maximize on the preperitoneal approach
 ↑ESR, BUN & creatinine
while minimizing on the length of the skin and fascia
 Most definitive noninvasive diagnostic test:
incision
intravenous pyelography

TOPNOTCH MEDICAL BOARD PREP SURGERY SUPPLEMENT HANDOUT Page 74 of 83


For inquiries visit www.topnotchboardprep.com.ph or email us at topnotchmedicalboardprep@gmail.com
TOPNOTCH MEDICAL BOARD PREP SURGERY SUPPLEMENT HANDOUT - Jules Lopez,MD-MBA,Teddy Carpio,MD-MBA
For inquiries visit www.topnotchboardprep.com.ph or email us at topnotchmedicalboardprep@gmail.com
Laparoscopic repair: uses preperitoneal approach undergo laparoscopic inguinal hernia repair.
with small incisions; can asses and repair unilateral or Preoperatively, all patients underwent US and MRI.
bilateral inguinal hernias Laparoscopic confirmation of the presence of inguinal
Emergent inguinal hernia repair: reserved for hernia was deemed as gold standard. Physical
strangulated, incarcerated and sliding hernias examination was found to be the least sensitive. False
positives were low on physical examination and MRI
(one finding), but higher with US (four findings). With
Figure 37: treatment algorithm for hernia repair further refinement of technology, radiologic techniques
qill continue to improve sensitivity and specificity rates
of diagnosis, thereby serving a supplementary role in
cases of undertain diagnosis

LIVER, PORTAL VENOUS SYSTEM & GALLBLADDER


DIAGNOSTIC PERITONEAL LAVAGE
A. Anatomy
A catheter is placed infraumbilically and a 10mL syringe is B. Liver function tests
connected. The aspirate is considered to show positive findings C. Radiographic evaluation
if >10 mL of blood is aspirated. If <10 mL is withdrawn, a liter D. Liver cirrhosis
of normal saline is instilled. The effluent is withdrawn via E. Portal Hypertension
siphoning and sent to the laboratory for analysis F. Budd-Chiari syndrome
G. Infections of the liver
Anterior H. Benign neoplasms of the liver
Thoracoabdominal I. Malignant tumors
Abdominal Stab
Stab Wounds J. Gallstone disease
Wounds
RBC Count >100,000/mL >10,000/mL K. Acute cholecystitis
WBC Count >500/mL >500/mL L. Choledocholithiases
Amylase level >19 IU/L >19 IU/L M. Cholangitis
Alkaline N. Biliary pancreatitis
Phosphatase >2 IU/L >2 IU/L O. Acalculous cholecystitis
level P. Biliary or choledochal cysts
Q. Sclerosing cholangitis
Bilirubin level >0.01mg/dL >0.01 mg/dL

A. ANATOMY
REVIEW QUESTIONS
Liver
1. Which of the following is the most important initial
- Largest organ, weighing approximately 1500 g
therapy for a patient with portal hypertension,
ascites, and a tense umbilical hernia? - Hepatoduodenal ligament  contains the porta
hepatis (portal vein, hepatic artery and common bile
a. Primary repair with concurrent placement of a duct)
peritoneal venous shunt  Pringle maneuver: used to clamp this ligament in
b. Emergency primary repair to avoid hernia rupture the event of injury to the right hepatic artery
c. Medical therapy to control ascites during cholecystectomy
d. Transjugular intrahepatic portocaval shunt  Relationship: CBD is located at the right of the
followed by umbilical hernia repair hepatic artery and anterior to the portal vein
 From the right and deep to the porta hepatis is the
Answer: C foramen of winslow (or epiploic foramen)
Treatment and control of the ascited with diuretic, - Segmental anatomy
dietary management and paracentesis is the most  Cantlie’s line: plane from the gallbladder fossa to
appropriate initial therapy. Patients with refractory the IVC that separates the liver’s right and left
ascited may be candidates for transjugular intrahepatic lobes grossly.
portocaval shunting or eventual liver transplantation.  Falciform ligament: separates the left lateral and
Umbilical hernia repair should be deferred until after left medial segments along the umbilical fissure
the ascites is controlled. and anchors the liver to the anterior abdominal
wall; does not separate the liver to right and left
2. In the setting of an equivocal examination, which lobes
of the following has the greatest sensitivity in  Couinaud’s segments: divides liver into 8
diagnosing an inguinal hernia? segments, in clockwise direction with caudate lobe
as segment 1
a. Repeat examination by a second surgeon
b. Ultrasound
Figure 38. Segmental anatomy of liver
c. CT scan
d. MRI

Answer: D
Although Ct scan is useful in ambiguous clinical
presentations, little data exist to support its routine use
in diagnosis. The use of MRI in assessing groin hernias
was examined in a group of 41 patients scheduled to
TOPNOTCH MEDICAL BOARD PREP SURGERY SUPPLEMENT HANDOUT Page 75 of 83
For inquiries visit www.topnotchboardprep.com.ph or email us at topnotchmedicalboardprep@gmail.com
TOPNOTCH MEDICAL BOARD PREP SURGERY SUPPLEMENT HANDOUT - Jules Lopez,MD-MBA,Teddy Carpio,MD-MBA
For inquiries visit www.topnotchboardprep.com.ph or email us at topnotchmedicalboardprep@gmail.com
several days for the levels to normalize even
after resolution of biliary obstruction
GGTP -early marker and sensitive test for
hepatobiliary disease
-nonspecific; can also be elevated in overdose
of certain medications, alcohol abuse,
pancreatic disease, myocardial infarction,
renal failure, & obstructive pulmonary disease
- interpret GGTP elevations with other enzyme
abnormalities

Notes to figure
Segments part Corresponding side Venous drainage
BIOCHEMISTRY
Segment I Caudate lobe IVC
Segment II Left lateral superior segment Left lobe Left hepatic vein AST (aspartate transaminase): an enzyme in gluconeogenesis
Segment III Left lateral inferior segment Left lobe Left hepatic vein
Segment IV Left medial segment (quadate Left lobe Middle hepatic that transfers amino groups from aspartic acid to ketoglutaric
lobe – outdated) vein acid to produce oxaloacetate.
Segment V Right anterior inferior segment Right lobe Right & middle
hepatic vein
Segment VI Right posterior inferior segment Right lobe Right hepatic vein ALT (alanine transaminase): an enzyme in gluconeogenesis that
Segment
VII
Right posterior superior segment Right lobe Right hepatic vein transfers amino groups from alanine to ketoglutaric acid to
Segment Right anterior superior segment Right lobe Right & middle produce pyruvic acid
VIII hepatic vein

- The hepatic veins divides the liver into 4 sectors


- The liver has dual blood supply:
 hepatic actery: 25%
 branch of celiac artery
 most common variation: right hepatic artery
from SMA
 portal vein: 75% (majority)
 confluence of splenic vein and SMV C. RADIOGRAPHIC EVALUATION
- normal pressure: 3-5 mmHg
ULTRASOUND
- communication of portal vein and systemic circulation
(important for location of varices & bleeding in portal
Liver
hypertension): gastroesophageal junction, anal
- Useful initial test imaging test of the liver because it
canal, falciform ligament, splenic venous bed and
is inexpensive, involves no radiation exposure, and is
left renal vein, and retroperitoneum
well tolerated by patients
- Biliary tree
- It is excellent for diagnosing biliary pathology and liver
 Hepatic ducts follow arterial branching of the liver
lesions.
 Left hepatic duct has a longer extrahepatic course
- Limitations:
 Incomplete imaging: dome or beneath the ribs on
B. LIVER FUNCTION TESTS 
the surface, lesion boundaries are not as visualized
- Term used to frequently measure the levels of group of
 Obesity
serum markers for evaluation of liver dysfunction.
 Overlying gas bowels
- A misnomer because the panel measures cell damage,
- If a mass is detected, further evaluation by CT or MRI is
and not liver function
required since UTZ has lower sensitivity and specificity
Table 62. Different components of liver function tests - Intraoperative ultrasound:
Serum albumin, -Measures liver’s synthetic function  Gold standard for diagnosing liver lesions
prothrombin - prothrombin time and INR: best test  Useful for tumor staging, visualization of
time & clotting among the 3 to measure the liver’s synthetic intrahepatic vascular structures, guidance of
factors (except function resection plane by assessment of relationship of
factor VIII) - PT is prolonged with conditions such as mass to vessels, for biopsy of tumors and tumor
vitamin K deficiency or warfarin therapy ablation
(because vitamin K is involved in the Y-
carboxylation of factors used to measure
Gallbladder
prothrombin time)
***factor VIII: not synthesized exclusively in the - UTZ is the initial investigation used for any patient
liver; has the shortest half life; useful for suspected of disease in the biliary tree.
determining liver failure - UTZ will show gallbladder stones with sensitivity and
AST (formarly -Indicators of integrity of hepatocellular specificity of >90%
SGOT) & ALT membranes; increased levels reflect  Appearance of GB Stones: (+) acoustic shadow,
(formarly SGPT) hepatocellular injury with leakage move with changes in position (vs polyps: may
-AST: can also be found in liver, cardiac also have a shadow but does not move with
muscle, skeletal muscle, kidney, brain,
changes in position)
pancreas, lungs, and red blood cells and thus is
less specific  If a stone obstructs the neck of the GB: large GB but
-ALT: more specific for liver disease thin walled
-AST:ALT ratio of >2:1  alcoholic liver - if acute cholecystitis: (+) edema within the wall of the
disease GB or between the GB and liver in association with
moderate increases: viral hepatitis localized tenderness
-in the thousands ischemia, toxin ingestion - if chronic cholecystitis: contracted thick-walled GB
(acetaminophen), fulminant hepatitis - Extrahepatic ducts are well visualized using UTZ
Indirect -elevations point to intrahepatic cholestasis, (except for retroduodenal portion)
(unconjugated) hemolytic disorders (hemolytic anemia,
bilirubin resoprtion of hematomas), bilirubin defects in
- Dilation of the ducts + stones in the GB + jaundiced
hepatic uptake or conjugation (acquired or patient  think extrahepatic obstruction
inherited)  Periampullary tumors can be difficult to diagnose
Direct -elevations point to extrahepatic or obstructive on UTZ
(conjugated) cholestasis, inherited or acquired disorders of  UTZ is useful for evaluating tumor invasion and
bilirubin intrahepatic excretion or extrahepatic flow in the portal vein – an important guideline in
obstruction
the resectability of periampullary and pancreatic
Alkaline -found in liver and bones;
phosphatase -indicative of biliary obstruction head tumors
- since half life of AP is 7 days, it may take
TOPNOTCH MEDICAL BOARD PREP SURGERY SUPPLEMENT HANDOUT Page 76 of 83
For inquiries visit www.topnotchboardprep.com.ph or email us at topnotchmedicalboardprep@gmail.com
TOPNOTCH MEDICAL BOARD PREP SURGERY SUPPLEMENT HANDOUT - Jules Lopez,MD-MBA,Teddy Carpio,MD-MBA
For inquiries visit www.topnotchboardprep.com.ph or email us at topnotchmedicalboardprep@gmail.com
ORAL CHOLECYSTOGRAPHY
- Considered as a diagnostic procedure of choice for
gallstones but it largely replaced now by UTZ. MAGNETIC RESONANCE IMAGING
- Mechanism: oral administration of radiopaque
compound that is absorbed and excreted by the liver, Liver
passed into the GB  stones are noted on a film as a - Also uses contrast agent, just like in CT scan, to
filling defect in a visualized, opacified GB differentiate normal and pathologic lesion in the liver
- Types:
 Gadopentate dimeglumine – behaves in a manner
BILIARY RADIONUCLIDE SCANNING (HIDA SCAN) similar to iodine in CT
 Feruxomide – excretion of kuppfer cells
- Provides a noninvasive evaluation of the liver, GB, bile  Iminoacetic acid-derivative radionuclide –
ducts and duodenum with both anatomic and functional secretion in bile by hepatocytes
information
- Mechanism: Technetium-labeled derivatives of Gallbladder
dimethyl iminodiacetic acid (HIDA) are injected IV - MRI with MRCP (magnetic resonance
 cleared by Kuppfer cells in the liver  excreted in cholangiopancreatography)  offers a single
the bile noninvasive test for the diagnosis of bliary tract and
 10 minutes: time it takes for the liver to detect it pancreatic disease
 60 minutes: time it takes for the GB, bile ducts and
duodenum to detect it
- the primary use of biliary scintigraphy is the ENDOSCOPIC RETROGRADE CHOLANGIOGRAPHY (ERCP)
diagnosis of acute cholecystitis - It is the diagnostic and therapeutic procedure of
 appearance: nonvisualized GB, with prompt filling choice for stones in the CBD associated with
of the common bil duct and duodenum, biliary obstructive jaundice, cholangitis and gallstone
obstruction pancreatitis
 sensitivity & specificity: 95% - Provides direct visualization of the bilary and
- can also detect obstruction of the ampulla pancreatic ducts, particularly the ampullary region and
 appearance: filling of the GB and CBD with delayed distal common bile duct
and absent filling of the duodenum - Therapeutic interventions include sphincterotomy,
- can also be used for detection of biliary leaks as a stone extraction if indicated
complication of GB surgery
POSITRON EMISSION TOMOGRAPHY
COMPUTED TOMOGRAPHY liver
Liver - PET offers functional imaging of tissues with high
- Contrast medium is routinely used for liver evaluation metabolic activity, including most types of metastatic
because of the similar densities of most pathologic tumors
liver masses and normal hepatic parenchyma. - With high value for colorectal cancer with liver
 Uses dual or triple phase bolus of IV contrast metastases
 Exploits the dual blood supply of the liver: most  20% of patients with colorectal cancer present
liver tumors receive their blood supply from initially with liver metastasis
the hepatic artery and normal hepatic  presence of extrahepatic disease is a poor
parenchyma from portal vein prognosticator and precludes surgical intervention
 2 phases:  valuable tool for the diagnostic work up of
1. Arterial dominant phase (20 to 30 patient with potentially resectable hepatic
seconds after beginning of contrast disease
injection) – the phase where hepatic tumors  must be combined with CT to improve
and other hypervascular lesions are well diagnostic accuracy
delineated.
2. Venous or portal dominant phase (60 to D. LIVER CIRRHOSIS
70 seconds after contrast injection) – the - final sequela of chronic hepatic insult, is characterized
phase where there is optimal enhancement by the presence of fibrous septa (due to
of normal liver parenchyma and accumulation to ECM matrix or scar tissue)
hypovascular lesions (will appear throughout the liver subdividing the parenchyma into
attenuated in contrast with brighter normal hepatocellular nodules
liver parenchyma) - 2 consequences: hepatocellular failure and portal
hypertension
gallbladder - Classification
- It is the test of choice in evaluating patients with  Micronodular cirrhosis: characterized by thick
suspected malignancy of the GB, extrahepatic regular septa, small uniform regenerative nodules,
biliary system or nearby organs, in particular, the and involvement of virtually every hepatic lobule
head of pancreas  Macronodular cirrhosis: frequently has septa and
- Abdominal CT scan is inferior to UTZ in diagnosing regenerative nodules (irregularly sized
gallstones hepatocytes with large nuclei and cell plates of
varying thickness)
 Mixed cirrhosis: present when regeneration is
PERCUTANEOUS TRANSHEPATIC CHOLANGIOGRAPHY occurring in a micronodular liver and over time
- Useful in patients with bile duct strictures and converts to a macronodular pattern
tumors, as it defines the anatomy of the biliary tree - Etiology: viral, autoimmune, drug-induced, cholestatic,
proximal to the affected segment and metabolic diseases
- Mechanism: intrahepatic ducts are accessed - Clinical manifestation
percutaneously with a small needle under fluoroscopic  Fat stores and muscle mass are reduced
 resting energy expenditure is increased
guidance  catheter is placed  cholangiogram
 (+) Muscle cramps: respond to administration of
performed  can do therapeutic interventions as quinine sulfate and human albumin
well (biliary drain insertion, stent placement)  increased CO & HR
- Very little role in management of uncomplicated
gallstone disease
TOPNOTCH MEDICAL BOARD PREP SURGERY SUPPLEMENT HANDOUT Page 77 of 83
For inquiries visit www.topnotchboardprep.com.ph or email us at topnotchmedicalboardprep@gmail.com
TOPNOTCH MEDICAL BOARD PREP SURGERY SUPPLEMENT HANDOUT - Jules Lopez,MD-MBA,Teddy Carpio,MD-MBA
For inquiries visit www.topnotchboardprep.com.ph or email us at topnotchmedicalboardprep@gmail.com

 Prone to infections  due to impaired phagocytic  Octreotide/somatostatin for splanchnic


activity of the RES vasoconstriction
 Endoscopic variceal ligation (EVL)
- Diagnosis  Balloon tamponade using sengstaken-
 mild normocytic normochromic anemia. blakemore tube
 Decreased WBC & PC  Shunt therapy (surgical shunts or
 bone marrow: macronormoblastic TIPS)
 prothrombin time is prolonged & does not respond  Even with aggressive pharmacologic and
to vitamin K tx endoscopic therapy, 10-20% of patients
 serum albumin level is decreased with variceal bleeding will continue to
 serum levels of bilirubin, transaminases, and rebleed
alkaline phosphatase are all elevated  Shunt therapy (surgical shunt or TIPS),
on the other hand, has been shown to
- CHILD-TURCOTTE-PUGH SCORE: evaluate the risk of control refractory variceal bleeding in
portocaval shunt procedures secondary to portal >90% of treated individuals
hypertension and also useful in predicting surgical  Surgical shunt: CTP class A
risks of other intra-abdominal operations  TIPS: CTP class B & C
performed on cirrhotic patients  Balloon tamponade using sengstaken-
blakemore tube can control refractory
Table 63. Child-Turcotte-Pugh Score bleeding in >80% of patients
variable 1 point 2 points 3 points
 Complication: aspiration, esophageal
Bilirubin < 2 mg/dL 2-3 mg/dL >3 mg/dL
perforation
Albumin >3.5 g/dL 2.8-3.5 g/dL <2.8 g/dL
INR <1.7 1.7-2.2 >2.2
Table.64 Comparison of Surgical shunts vs TIPS
Encephalopathy none controlled uncontrolled
Surgical shunts (can be selective TIPS (Transjugular Intrahepatic
Ascites none controlled uncontrolled
or non selective shunts) Portosystemic Shunt)
Child-Turcotte-Pugh Class & overall surgical mortality rates
-aim: reduce portal venous -considered as a nonselective
Class A = 5-6 points  10%
pressure, maintain total hepatic shunt
Class B= 7-9 points  30%
and portal blood flow and avoid a -involves implantation of a metallic
Class C= 10-15 points  75-80%
high incidence of complicating stent between an intrahepatic
hepatic encephalopathy branch of the portal vein and a
E. PORTAL HYPERTENSION hepatic vein radical
- definition: direct portal venous pressure that is >5 -non-selective shunt (ex. -TIPS can control variceal
mmHg greater than the IVC pressure, a splenic pressure portacaval shunt or eck fistula: bleeding in >90% of cases
of >15 mmHg, or a portal venous pressure measured at joins the portal vein to the IVC in refractory to medical treatment
surgery of >20 mmHg an end-to-side fashion & disrupts -disadvantages: bleeding either
portal vein flow to the liver, or intra-abdominally or via the biliary
- normal portal venous pressure: 5 to 10 mmHg
joins it in a side-to-side fashion tree, infections, renal failure,
 at this pressure, very little blood is shunted from and maintains partial portal decreased hepatic function, and
the portal venous system into the systemic venous flow to the liver; non ↑er hepatic encephalopathy
circulation selective; rarely performed now (because it is a non selective
 as portal venous pressure increases, the because it has a higher incidence shunt)
communication with the systemic circulation dilate of hepatic encephalopathy and
 Large amount of blood is shunted around the decreased liver function resulting
from the reduction of portal
liver and into the systemic circulation  perfusion; controls bleeding
complications effectively
 A portal pressure of >12 mmHg is necessary for
varices to form and subsequently bleed -selective shunt (ex. Warren
- Etiology: most common cause is cirrhosis shunt – distal splenorenal & left
gastric caval shunt) have ↓er
(intrahepatic)
incidence of hepatic
- Clinical manifestation encephalopathy
 Most significant clinical finding:
gastroesophageal varices F. BUDD-CHIARI SYNDROME
 Major BS of GE varices: anterior branch of the - uncommon congestive hepatopathy characterized by
left gastric or coronary vein the obstruction of hepatic venous outflow due to
 May present with splenomegaly, hemorrhoids, endoluminal venous thrombosis (primary) or
ascites, caput medusa & upper GI bleeding due to compressive lesion external to the veins (secondary)
variceal bleeding (leading cause of morbidity - risk factors: coagulopathies, thrombotic disease
and mortality) - most patients are women
- Diagnosis: most accurate method of determining portal - mean age of diagnosis: 30 yo
hypertension is hepatic venography - clinical manifestations: abdominal pain (RUQ), ascites,
- Management and hepatomegaly or long standing portal hypertension
 Prevention of variceal bleeding: improve liver - diagnosis
function (avoid alcohol), avoid aspirin & NSAID, - abdominal UTZ: initial investigation of choice
beta blockers  check for absence of hepatic vein flow, spider web
 Management of acute variceal bleeding hepatic veins & collateral circulation
 Specifics: - definitive imaging: hepatic venography
 ICU admission: must! - initial treatment: anticoagulation
 Blood resuscitation: goal is Hgb of
8g/dL and above G. INFECTIONS OF THE LIVER
 FFP and platelets for patients with
severe coagulopathy PYOGENIC LIVER ABSCESS
 Short term prophylactic antibiotics: - most common liver abscesses seen in the United
ceftriaxone 1g/day (proven to decrease States.
the rate of bacterial infections and - Risk factors: IV drug abuse, teeth cleaning, diverticulitis,
increase survival) Crohn's disease, subacute bacterial endocarditis, (+)
 Vassopressin at 0.2 to 0.8 units/min IV infected indwelling catheters & immunocompromised
for vasoconstriction (most potent) states
- may be single or multiple
- more frequently found in the right lobe of the liver
TOPNOTCH MEDICAL BOARD PREP SURGERY SUPPLEMENT HANDOUT Page 78 of 83
For inquiries visit www.topnotchboardprep.com.ph or email us at topnotchmedicalboardprep@gmail.com
TOPNOTCH MEDICAL BOARD PREP SURGERY SUPPLEMENT HANDOUT - Jules Lopez,MD-MBA,Teddy Carpio,MD-MBA
For inquiries visit www.topnotchboardprep.com.ph or email us at topnotchmedicalboardprep@gmail.com
- causative organisms:  intermediate hosts: Humans, sheep, and cattle
 monomicrobial: 40% ; polymicrobial: 40%;  definitive host: dogs
culture negative: 20% - commonly involve the right lobe of the liver, usually
 most common: gram-negative organisms the anterior-inferior or posterior-inferior segments
(Escherichia coli – 2/3; Streptococcus faecalis, - clinical manifestations: dull RUQ or abdominal
Klebsiella, and Proteus vulgaris are also common) distention; can be clinically silent; if ruptured, may lead
 Anaerobic organisms (ex. Bacteroides fragilis) are to an allergic or anaphylactic reaction.
also seen frequently - Diagnosis:
 If (+) endocarditis / indwelling catheter: think  (+)ELISA for echinococcal antigens; maybe (-) if
Staphylococcus and Streptococcus cyst has not leaked or does not contain scolices, or
- Clinical manifestations: RUQ pain, fever & jaundice (1/3 if the parasite is no longer viable
of patients)  Eosinophilia of >7% is found is approximately 30%
- Diagnosis: of infected patients.
 Leucocytosis, ↑ESR & AP (most common  UTZ & CT scan of the abdomen: sensitive for
laboratory findings) detecting hydatid cysts.
 Blood cultures reveal the causative organism in  hydatid cysts: appear as well-defined
approximately 50% of cases. hypodense lesions with a distinct wall; (+)
 Liver UTZ: round or oval hypoechoic lesions Ring-like calcifications of the pericysts
with well-defined borders and a variable (present in 20 to 30% of cases); healing occurs
number of internal echoes.  the entire cyst calcifies densely, and a
 CT scan: highly sensitive in the localization; lesion with this appearance is usually dead or
appear as hypodense mass with air-fluid levels inactive. Daughter cysts: occur in a peripheral
(indicating a gas-producing organisms) & location & are slightly hypodense compared
peripheral enhancement with the mother cyst.
- Treatment: cornerstones of treatment include - Treatment:
correction of the underlying cause, percutaneous needle  Unless the cysts are small or the patient is not a
aspiration, and IV antibiotic therapy suitable candidate for surgery, treatment of
 Initial antibiotic therapy needs to cover gram- hydatid disease is surgically based (laparoscopic
negative as well as anaerobic organisms; must or open complete cyst removal + instillation of
be continued for at least 8 weeks. scolicidal agent)
 If aspiration and IV antibiotics fail, undergo  caution must be exercised to avoid rupture of
surgical therapy (either laparoscopic or open the cyst with release of protoscolices into the
drainage) peritoneal cavity.
 Anatomic surgical resection is reserved for patients  Peritoneal contamination can result in an acute
with recalcitrant abscesses. anaphylactic reaction or peritoneal
 Always rule out necrotic hepatic malignancy implantation of scolices with daughter cyst
formation and inevitable recurrence
 Medical treatment of choice: albendazole - initial
AMEBIC ABSCESS treatment for small, asymptomatic cysts.
- most common type of liver abscesses worldwide.
- Causative agent: Entamoeba histolytica H. BENIGN NEOPLASMS OF THE LIVER
- can be single or multiple
- most commonly located in the superior-anterior HEPATIC CYST
aspect of the right lobe of the liver near the diaphragm - most common benign lesion found in the liver is the
- Gross: necrotic central portion that contains a thick, congenital or simple cyst
reddish brown, pus-like material (anchovy paste or - female:male ratio is approximately 4:1
chocolate sauce) - Clinical manifestation: asymptomatic if small; Large
- Clinical manifestation: RUQ pain + fever + hepatomegaly simple cysts may cause abdominal pain, epigastric
+ travel to an endemic area fullness, and early satiety. Occasionally the affected
- Diagnosis: patient presents with an abdominal mass.
 most common biochemical abnormality: ↑ AP - Diagnosis: appear as thin-walled, homogeneous,
level. fluid-filled structures with few to no septations.
 (+) Leukocytosis - Treatment:
 ↑transaminase levels and jaundice are unusual.  Observation if asymptomatic
 (+) fluorescent antibody test for E. histolytica  If symptomatic, perform UTZ- or CT-guided
 Ultrasound and CT scanning: very sensitive but percutaneous cyst aspiration followed by
nonspecific for the detection of amebic abscesses sclerotherapy
 Appears to be as a well-defined low-density round  excised cyst wall is sent for pathologic analysis to
lesions that have enhancement of the wall, ragged rule out carcinoma, and the remaining cyst wall
in appearance with a peripheral zone of edema; has must be carefully inspected for evidence of
a central cavity with septations & fluid levels neoplastic change.
- Treatment
 Metronidazole 750 mg tid for 7 to 10 days is the HEMANGIOMA
treatment of choice and is successful in 95% of - consist of large endothelial-lined vascular spaces and
cases. represent congenital vascular lesions that contain
 Defervescence usually occurs in 3 to 5 days. fibrous tissue and small blood vessels which eventually
 Time of resolution of abscess: 30 to 300 days from grow
presentation - most common solid benign masses that occur in the
 Aspiration of the abscess is rarely needed and liver
should be reserved for patients with large - more common in women
abscesses, abscesses that do not respond to - clinical manifestation:
medical therapy, abscesses that appear to be  most common symptom is abdominal pain
superinfected, and abscesses of the left lobe of the  can be asymptomatic as well
liver that may rupture into the pericardium - diagnosis:
 biphasic contrast CT scan: asymmetrical nodular
peripheral enhancement that is isodense with
HYDATID DISEASE large vessels and exhibit progressive centripetal
- due to the larval or cyst stage of infection by the enhancement fill-in over time
tapeworm Echinococcus granulosus (causative agents)
TOPNOTCH MEDICAL BOARD PREP SURGERY SUPPLEMENT HANDOUT Page 79 of 83
For inquiries visit www.topnotchboardprep.com.ph or email us at topnotchmedicalboardprep@gmail.com
TOPNOTCH MEDICAL BOARD PREP SURGERY SUPPLEMENT HANDOUT - Jules Lopez,MD-MBA,Teddy Carpio,MD-MBA
For inquiries visit www.topnotchboardprep.com.ph or email us at topnotchmedicalboardprep@gmail.com
 MRI: hypointense on T1-weighted images and - Clinical manifestations: jaundice, pruritus,
hyperintense on T2-weighted images hepatosplenomegaly, bleeding diathesis, cachexia,
 Caution should be exercised in ordering a liver encephalopathy, asterixis, ascites and varices
biopsy if the suspected diagnosis is hemangioma - Diagnosis:
because of the risk of bleeding from the biopsy site  CT scan: appears hypervascular during the arterial
- treatment: Surgical resection (enucleation or phase of CT studies & relatively hypodense during
formal hepatic resection) only if symptomatic; the delayed phases due to early washout of the
observation if asymptomatic contrast medium by the arterial blood.
 MRI: HCC is variable on T1-weighted images and
usually hyperintense on T2-weighted images; HCC
HEPATIC ADENOMA enhances in the arterial phase after gadolinium
- benign solid neoplasms of the liver injection because of its hypervascularity and
- most commonly seen in young women (aged 20-40) becomes hypointense in the delayed phases due to
- typically solitary contrast washout
- risk factors: Prior or current use of estrogens (oral  (+) thrombus in portal vein is highly suggestive
contraceptives) of HCC
- Gross appearance: soft and encapsulated and are tan to  ↑AST,ALT,AFP
light brown. - treatment options for liver cancer
- Histology: does not contain Kuppfer cells  hepatic resection: reserved for patients without
- (+) risk of malignant transformation to a well- cirrhosis & Child's class A cirrhosis with preserved
differentiated HCC liver function and no portal hypertension
- Clinical manifestation:  liver transplantation: if with poor liver function
 carry a significant risk of spontaneous rupture and the HCC meets the Milan criteria (one nodule
with intraperitoneal bleeding. <5 cm, or two or three nodules all <3 cm, no gross
 The clinical presentation may be abdominal pain vascular invasion or extrahepatic spread)
- Diagnosis:  Chemoembolization can also be of benefit
 CT scan: with sharply defined borders; can be  5 year survival after complete resection: 30%
confused with metastatic tumors
 venous phase contrast: hypodense or isodense
(in comparison with background liver CHOLANGIOCARCINOMA
 arterial phase contrast: subtle hypervascular - 2nd most common primary malignancy within the liver
enhancement - It is the adenocarcinoma of the bile ducts that forms in
 MRI: hyperintense on T1-weighted images and the biliary epithelial cells
enhance early after gadolinium injection. - Most commonly occurs at the bifurcation of the
 nuclear imaging: "cold”; no uptake of common hepatic duct
radioisotope - Subclassification:
- Treatment: surgical resection  peripheral (intrahepatic) bile duct cancer
 tumor mass is within the lobe or peripheral of
the liver
FOCAL NODULAR HYPERPLASIA  less common that extrahepatic bile duct
- A benign, solid neoplasm of the liver Cancer
- more common in women of childbearing age  central (extrahepatic) bile duct cancer
- FNH lesions usually do not rupture spontaneously  if it is proximally located, referred to as a hilar
and have no significant risk of malignant cholangiocarcinoma (Klatskin's tumor).
transformation.  presents with obstructive and painless
- diagnosis: jaundice rather than an actual liver mass
 biphasic CT scan: well circumscribed with a - treatment:
typical central scar  surgical resection is the treatment of choice
 Arterial phase contrast: intense homogeneous  hilar cholangiocarcinoma + primary
enhancement sclerosing cholangitis: surgical resection has
 Venous phase contrast: isodense or invisible no role & transplantation provided dismal
 MRI scans: hypointense on T1-weighted images & results
isointense to hyperintense on T2-weighted images  neoadjuvant chemoradiation has a role
 After gadolinium administration, lesions are
hyperintense but become isointense on
delayed images. GALLBLADDER CANCER
 nuclear imaging: (+) uptake by Kupffer cells. - rare aggressive tumor with a very poor prognosis.
- Treatment: surgical resection only if symptomatic - Cholithiasis is the most important risk factor for
gallbladder carcinoma
BILE DUCT HAMARTOMA - 80-90% of gallbladder tumors are adenocarcinomas
- small liver lesions (2 - 4 mm) - signs and symptoms of GB carcinoma are
- usually visualized on the surface of the liver at indistinguishable from cholecystitis and cholelithiasis
laparotomy. - sensitivity of UTZ in detecting GB carcinoma ranges
- Gross appearance: firm, smooth, and whitish yellow in from 70-100%.
appearance. - Treatment: surgery is the only curative option for
- can be difficult to differentiate from small metastatic gallbladder cancer
lesions  reoperation for an incidental finding of gallbladder
- excisional biopsy often is required to establish the cancer after cholecystectomy (central liver
diagnosis resection, hilar lymphadenectomy, and evaluation
of cystic duct stump)
I. MALIGNANT TUMORS  reoperation should be considered for all patients
who have T2 or T3 tumors or for whom the
HEPATOCELLULAR CARCINOMA (HCC) accuracy of staging is in question
- 5th most common malignancy worldwide  radical resection in patients with advanced disease
- Risk factors: viral hepatitis (B or C), alcoholic cirrhosis,  usually with dismal results if already with (+)
hemochromatosis, and nonalcoholic steatohepatitis hilar LN
- HCCs are typically hypervascular with blood supplied
predominantly from the hepatic artery METASTATIC COLORECTAL CANCER
- Most common site of metastasis is lungs
TOPNOTCH MEDICAL BOARD PREP SURGERY SUPPLEMENT HANDOUT Page 80 of 83
For inquiries visit www.topnotchboardprep.com.ph or email us at topnotchmedicalboardprep@gmail.com
TOPNOTCH MEDICAL BOARD PREP SURGERY SUPPLEMENT HANDOUT - Jules Lopez,MD-MBA,Teddy Carpio,MD-MBA
For inquiries visit www.topnotchboardprep.com.ph or email us at topnotchmedicalboardprep@gmail.com
- Over 50% of patients diagnosed with colorectal cancer secrete mucus, and the gallbladder becomes
will develop hepatic metastases during their lifetime. distended with mucinous material)
- Resection is the preferred treatment for liver  Is usually an indication for cholecystectomy
metastases from colorectal CA, provided that patient - Diagnosis
has adequate liver reserve, no extrahepatic metastases,  Abdominal UTZ: standard diagnostic test for
total hepatic involvement and advanced crirhosis, vena gallstones
cava or portal vein invasion  Presence of hyperechoic intraluminal focus
- volume of future liver remnant and the health of the  Shadowing posterior to the focus
background liver, and not actual tumor number, as the  Movement of the focus with positional changes
primary determinants in selection for an operative of the patient
approach. - Management: Patients with symptomatic gallstones
should be advised to have elective laparoscopic
J. GALLSTONE DISEASE cholecystectomy
- Prevalence and incidence
 most common problems affecting the digestive K. ACUTE CHOLECYSTITIS
tract - Pathogenesis:
 Women are 3x more likely to develop gallstones  Acute cholecystitis is secondary to gallstones in 90
than men to 95% of cases
 risk factors: Obesity, pregnancy, dietary factors,  In <1% of acute cholecystitis, the cause is a tumor
Crohn's disease, terminal ileal resection, gastric obstructing the cystic duct (leads to gallbladder
surgery, hereditary spherocytosis, sickle cell distention, inflammation, and edema of the
disease, and thalassemia gallbladder wall)
- Natural history  Gross appearance: gallbladder wall is grossly
 Most patients will remain asymptomatic thickened & reddish with subserosal hemorrhages;
 prophylactic cholecystectomy in asymptomatic (+) pericholecystic fluid often; mucosal hyperemia
persons with gallstones is rarely indicated & patchy necrosis
 cholecystectomy is advisable for the ff  When the gallbladder remains obstructed and
asymptomatic patients: secondary bacterial infection supervenes  an
1. elderly patients with diabetes
acute gangrenous cholecystitis develops 
2. individuals isolated from medical care for
abscess or empyema forms within the gallbladder;
extended periods of time
can also lead to perforation of ischemic areas
3. in populations with increased risk of
 emphysematous gallbladder : (+) gas may be seen
gallbladder cancer (porcelain gallbladder
in the gallbladder lumen and in the wall of the
– premalignant lesion)
gallbladder on abdominal radiographs and CT
4. symptomatic Cholesterolosis:
scans due to gas-forming organisms as part of the
accumulation of cholesterol in macrophages
secondary bacterial infection
in the gallbladder mucosa, either locally or
- clinical manifestations:
as polyps; produces the classic macroscopic
 unremitting epigastric or RUQ pain, may persist for
appearance of a "strawberry gallbladder."
several days, may radiate to the right upper part of
5. symptomatic Adenomyomatosis or
the back or the interscapular area; febrile,
cholecystitis glandularis proliferans:
anorexia, nausea, and vomiting, reluctant to move,
characterized on microscopy by
(+) focal tenderness and guarding on the RUQ, (+)
hypertrophic smooth muscle bundles and
Murphy's sign (an inspiratory arrest with deep
by the ingrowths of mucosal glands into the
palpation in the right subcostal area) is
muscle layer (epithelial sinus formation)
characteristic
6. symptomatic granulomatous polyps
 mirizzi’s syndrome: Severe jaundice due to
- Gallstone formation
common bile duct stones or obstruction of the bile
 Cholesterol stones (80% of gallstones)
ducts by severe pericholecystic inflammation
 multiple, variable size, may be hard and
secondary to impaction of a stone in the
faceted or irregular, mulberry-shaped, and
infundibulum of the gallbladder that mechanically
soft; colors range from whitish yellow and
obstructs the bile duct
green to black
 in elderly patients and in those with diabetes
 Most cholesterol stones are radiolucent
mellitus, acute cholecystitis may have a subtle
 formation is due to supersaturation of bile
presentation resulting in a delay in diagnosis.
with cholesterol
- Laboratory diagnosis:
 Pigment stones (15-20% of gallstones)
 A mild to moderate leukocytosis (12,000 to 15,000
 dark because of the presence of calcium
cells/mm3)
bilirubinate
 if high WBC (above 20,000): suggests a
 Black pigment stones: small, brittle, black,
complicated form of cholecystitis such as
and sometimes speculated; In Asian countries
gangrenous cholecystitis, perforation, or
such as Japan, black stones account for a much
associated cholangitis.
higher percentage of gallstones than in the
 mild elevation of serum bilirubin, <4 mg/mL
Western hemisphere; typically occur in
 mild elevation of alkaline phosphatase,
patients with cirrhosis and hemolysis
transaminases, and amylase.
 Brown pigment stones: <1 cm in diameter,
- diagnosis:
brownish-yellow, soft, and often mushy; they
 UTZ: most useful radiologic test for diagnosing
are formed usually due to secondary to
acute cholecystitis
bacterial infection (ex. E. coli)caused by
 Is 95% sensitive and specific
bile stasis.; associated with stasis secondary
 Appears as thickening of the gallbladder wall
to parasite infection
and (+) pericholecystic fluid
- Clinical presentation
 (+) sonographic murphy’s sign
 Abdominal pain: epigastrium or RUQ, constant,
 Biliary radionuclide scanning (HIDA scan):
increasing in severity, episodic, usually after a fatty
most accurate in the diagnosis of acute
meal, nausea, vomiting
cholecystitis
 Hydrops of gallbladder: manifests as a palpable
- Treatment
nontender gallbladder
 IV fluids
 Usually due to impacted stone without
cholecystis (pathophysio: bile gets absorbed,
but the gallbladder epithelium continues to
TOPNOTCH MEDICAL BOARD PREP SURGERY SUPPLEMENT HANDOUT Page 81 of 83
For inquiries visit www.topnotchboardprep.com.ph or email us at topnotchmedicalboardprep@gmail.com
TOPNOTCH MEDICAL BOARD PREP SURGERY SUPPLEMENT HANDOUT - Jules Lopez,MD-MBA,Teddy Carpio,MD-MBA
For inquiries visit www.topnotchboardprep.com.ph or email us at topnotchmedicalboardprep@gmail.com
 Antibiotics: should cover Gram (-) aerobes +  UTZ: diagnostic test of choice; appears as distended
anaerobes - 3rd generation cephalosporin or 2nd gallbladder with thickened wall, biliary sludge,
generation cephalosporin + metronidazole pericholecystic fluid, and (+) abscess formation
 Analgesia - Treatment of choice: Percutaneous ultrasound- or CT-
 Cholecystectomy: definitive treatment guided cholecystostomy
 Laparoscopic cholecystectomy: procedure of choice
P. BILIARY or CHOLEDOCHAL CYSTS
L. CHOLEDOCHOLITHIASES - congenital cystic dilatations of the extrahepatic and/or
- Common bile duct stones intrahepatic biliary tree
- Common over the age of 60 - rare
- clinical manifestations: may be silent or incidental; if - more common in women
symptomatic, may cause pain, nausea and vomiting with - more frequently diagnosed during childhood
mild epigastric or RUQ tenderness + mild icterus - types:
- diagnosis: type I: cystic dilatation of the extrahepatic bile
 ↑ of serum bilirubin, alkaline phosphatase, and duct; most common type
transaminases type II: diverticulum of the CBD
 UTZ: dilated common bile duct (>8 mm in type III: a “choledochocele” extending from the distal
diameter) duct into the duodenum
 Endoscopic cholangiography: gold standard for type IV: combined intrahepatic and extrahepatic cysts
diagnosing CBD stones; can be therapeutic as well type V: cystic disease confined to intrahepatic ducts
 IOC can be done to evaluate CBD stones
- Treatment: sphincterotomy and ductal clearance of the - clinical manifestations: jaundice or cholangitis (for
stones is appropriate, followed by a laparoscopic adults);less than ½ of patients present with the classic
cholecystectomy clinical triad of abdominal pain, jaundice, and a mass
- diagnosis: Ultrasonography or CT scanning will confirm
M. CHOLANGITIS the diagnosis, but endoscopic, transhepatic, or MRC is
- Complication of choledochal stones required to assess the biliary anatomy and to plan the
- Gallstones are the most common cause of obstruction in appropriate surgical treatment
cholangitis - treatment: complete cyst excision with roux-en-Y
- Normal: bile is sterile hepaticojejunostomy
- Causative organisms: E. coli, Klebsiella pneumoniae,
Streptococcus faecalis, Enterobacter, and Q. SCLEROSING CHOLANGITIS
Bacteroidesfragilis - is a progressive disease that eventually results in
- Clinical manifestations : secondary biliary cirrhosis characterized by
 most common presentation is fever, epigastric or inflammatory strictures involving the intrahepatic and
right upper quadrant pain, and jaundice extrahepatic biliary tree
(Charcot's triad) - associated with ulcerative colitis, Riedel's thyroiditis
 charcot’s triad + septic shock + mental status and retroperitoneal fibrosis
- increased risk for developing cholangiocarcinoma.
changes  reynaud’s pentad
- mean age of presentation is 30 to 45 years
- diagnosis:
- men are affected twice as commonly as women
 Leukocytosis, hyperbilirubinemia, and elevation of
- clinical manifestations: jaundice, fatigue, weight loss,
alkaline phosphatase and transaminases are seen
pruritus, and abdominal pain; usually with cyclic
 UTZ: (+)gallbladder stones, dilated ducts
remissions and excacerbations
 ERC: Definitive diagnosis
- diagnosis:
- Treatment
 elevated ALP & bilirubin
 IV antibiotics: initial management; cover for
 ERCP: confirmatory test
gram (-)
 multiple dilatations and strictures
 Fluid resuscitation: initial management
(beading) of both the intra- and
 Emergency biliary decompression: if failed to
extrahepatic biliary tree
improve with IV antibiotics and resuscitation
measures
REVIEW QUESTIONS
N. BILIARY PANCREATITIS
- Obstruction of the pancreatic duct by an impacted stone
1. A patient presents with biliary colic. On ultrasound
or temporary obstruction by a stone passing through
there are multiple small gallstones in the
the ampulla leads to this condition
gallbladder and the common bile duct measures
- Diagnosis: UTZ of biliary tree
9mm in diameter. No stone is visualized in the
- Treatment: ERC with sphincterotomy and stone
common bile duct. Which of the following is the
extraction + cholecystectomy (upon resolution of
most reasonable next step?
pancreatitis during same admission)
a. Repeat UTZ in 24-48 hours
O. ACALCULOUS CHOLECYSTITIS
b. MRCP with contrast
- develops in critically ill patients in ICU (patients on
c. Percutaneous cholangiography
parenteral nutrition with extensive burns, sepsis, major
d. Laparoscopic cholecystectomy and intraoperative
operations, multiple trauma, or prolonged illness with
cholangiography
multiple organ system failure)
- histopathology: reveals edema of the serosa and Answer: D
muscular layers, with patchy thrombosis of arterioles For patients with symptomatic gallstones and
and venules suspected CBD stones, either preoperative endoscopic
- clinical manifestations: cholangiography or an intraoperative cholangiogram
 alert patient: right upper quadrant pain and will document the bile duct stones. If an endoscopic
tenderness, fever, and leukocytosis cholangiogram reveals stones, sphincterotomy and
 sedated or unconscious patient: fever and elevated ductal clearance of the stones is appropriate, followed
WBC count, as well as elevation of alkaline by a laparoscopic cholecystectomy. An intraoperative
phosphatase and bilirubin cholangiogram at the time of cholecystectomy will also
- diagnosis: document the presence or absence of bile duct stones.
Laparoscopic common bile duct exploration via the
cystic duct or with formal choledochotomy allows the
TOPNOTCH MEDICAL BOARD PREP SURGERY SUPPLEMENT HANDOUT Page 82 of 83
For inquiries visit www.topnotchboardprep.com.ph or email us at topnotchmedicalboardprep@gmail.com
TOPNOTCH MEDICAL BOARD PREP SURGERY SUPPLEMENT HANDOUT - Jules Lopez,MD-MBA,Teddy Carpio,MD-MBA
For inquiries visit www.topnotchboardprep.com.ph or email us at topnotchmedicalboardprep@gmail.com
stones to be retrieved in the same setting. If the
expertise and/or the instrumentation for laparoscopic
common bile duct exploration are not available, a drain
shuld be left adjacent to the cystic duct and the patient
scheduled for endoscopic sphincterotomy the following
day. An open common bile duct exploration is an option
of the endoscopic method has already been tried or is,
for some reason, not feasible.

2. Which hepatic cells provides the primary defense


against lipopolysaccharide (LPS)?

a. Hepatocytes
b. Kuppfer cells
c. Bile duct epithelial cells
d. Intrahepatic endothelial cells

Answer: B
The complications of Gram negative sepsis is
initiated by the endotoxin LPS. The liver is the
main organ in the clearance of LPS in the
bloodstream and plays a critical role in the
identification and processing of LPS. Kuppfer cells
are the resident macrophages in the liver and have
been shown to participate in LPS clearance.

TOPNOTCH MEDICAL BOARD PREP SURGERY SUPPLEMENT HANDOUT Page 83 of 83


For inquiries visit www.topnotchboardprep.com.ph or email us at topnotchmedicalboardprep@gmail.com

You might also like